You are on page 1of 350

12

GENERAL
CHEMISTRY 2

QUARTER 1

LEARNING ACTIVITY SHEET


Republic of the Philippines

Department of Education

COPYRIGHT PAGE
Learning Activity Sheet in EARTH SCIENCE
(Grade 12)

Copyright © 2020
DEPARTMENT OF EDUCATION
Regional Office No. 02 (Cagayan Valley)
Regional Government Center, Carig Sur, Tuguegarao City, 3500

“No copy of this material shall subsist in any work of the Government of the Philippines. However,
prior approval of the government agency or office wherein the work is created shall be necessary
for exploitation of such work for profit.”

This material has been developed for the implementation of K to 12 Curriculum through the
Curriculum and Learning Management Division (CLMD). It can be reproduced for educational
purposes and the source must be acknowledged. Derivatives of the work including creating an
edited version, an enhancement of supplementary work are permitted provided all original works
are acknowledged and the copyright is attributed. No work may be derived from this material for
commercial purposes and profit.

Consultants:
Regional Director : ESTELA L. CARIÑO, EdD., CESO IV
Assistant Regional Director : RHODA T. RAZON, EdD., CESO V
Schools Division Superintendent : ORLANDO E. MANUEL, PhD, CESO V
Asst. Schools Division Superintendent(s): WILMA C. BUMAGAT, PhD., CESE
CHELO C. TANGAN, PhD., CESE
Chief Education Supervisor, CLMD : OCTAVIO V. CABASAG, PhD
Chief Education Supervisor, CID : ROGELIO H. PASINOS, PhD.

Development Team
Writers : LESTERWIN UDARBE, FLORIE MAE UNCIANO, DIVINA S. RIBIACO,
JACKIE B. UBINA, ANGELIKA TORRES, SHAROLYN T. GALURA, IVON
ADDATU, LOVEJOICE AMBABAG, JENIFER LOU ABUZO, CATHERINE
PASCUAL, CHERRY JANE BASUG, JENEVIE VINAGRERA
Content Editor : CHRISTOPHER S. MASIRAG- SDO CAGAYAN, ,RITA CORPUZ-SDO
CAGAYAN, LEAH DELA CRUZ-SDO SANTIAGO, ROSELLE MENDOZA-
SDO NUEVA VIZCAYA
Language Editor : MARIBEL S. ARELLANO- SDO CAGAYAN
Illustrators : Name, School, SDO
Layout Artists : Name, School, SDO
Focal Persons : GERRY C. GOZE, PhD., Division Learning Area Supervisor
NICKOYE V. BUMANGALAG, PhD. Division LR Supervisor
ESTER T. GRAMAJE, Regional Learning Area Supervisor
RIZALINO CARONAN, PhD. Regional LR Supervisor
Printed by: DepEd Regional Office No. 02
Regional Center, Carig Sur, Tuguegarao City

NOTE: Practice personal hygiene protocols at all times


i
Table of Contents

Page
Compentency Code
number
Use the kinetic molecular model to explain
properties of liquids and solids
STEM_GC11IMFIIIa-c-99 1 - 13
Describe and differentiate the types of
intermolecular forces STEM_GC11IMFIIIa-c-100 14 – 29
Describe the following properties of liquids,
and explain the effect of intermolecular
forces on these propoerties: surface
tension, viscosity, vapor pressure, boiling
point, and molar heat of vaporization STEM_GC11IMFIIIa-c-102 30 – 50
Expplain the properties of water with its
molecular structure and intermolecular
forces STEM_GC11IMFIIIa-c-103 51 - 67
Describe the difference in structure of
crystalline and amorphous solids STEM_GC11IMFIIIa-c-104 68 – 91
Interpret the phase diagram of water and
carbon dioxide STEM_GC11IMFIIIa-c-107 92 – 110
Determine and explain the heating and
cooling curve of a substance STEM_GC11IMFIIIa-c-109 111- 119
Use different ways of expressing
concentration of solutions: percent by mass,
mole fraction, molarity, molality, percent by
volume, percent by mass, ppm STEM_GC11PPIIId-f-111 120 – 142
Perform stoichiometric calculations for
reactions in solution STEM_GC11PPIIId-f-112 143 – 159
Describe the effect of concentration on the
colligative proporties of solutions STEM_GC11PPIIId-f-115 160 – 170
Defferentiate the colligative properties of
nonelectrolyte solutions and of electrolyte
solutions STEM_GC11PPIIId-f-116 171 – 187
Calculate boiling point elevation and
freezong point depression from the
concentration of a solute in a solution STEM_GC11PPIIId-f-117 189- 198
Calculate molar mass from colligative
property data STEM_GC11PPIIId-f-118 199 – 209

NOTE: Practice personal hygiene protocols at all times ii


Describe laboratory procedures in
determining concentration of solutions STEM_GC11PPIIId-f-119 210 – 224
Explain the first law of thermodynamics STEM_GC11TCIIIg-i-124 225 – 233
Explain enthalpy of reaction STEM_GC11TCIIIg-i-125 234 – 244
Calculate the change in enthalpy of a given
reaction using Hess Law STEM_GC11TCIIIg-i-127 245 – 255
Describe how various factors influence the
rate of reaction STEM_GC11CKIIIi-j-130 256 – 268
Differentiate zero, first- , and second-order
reactions STEM_GC11CKIIIi-j-132 269 – 293
Explain reactions qualitatively in terms of
molecular collisions STEM_GC11CKIIIi-j-136 294 – 313
Expalin activation energy and how a catalyst
affects the reaction rate STEM_GC11CKIIIi-j-137 314 – 229
Cite and differentate the types of catalysts STEM_GC11CKIIIi-j-138 230 – 249

NOTE: Practice personal hygiene protocols at all times iii


GENERAL CHEMISTRY 2

Name: ____________________________ Grade Level: _________


Date: _____________________________ Score: ______________

LEARNING ACTIVITY SHEET


KINETIC MOLECULAR MODEL OF SOLIDS AND LIQUIDS

Background Information for the Learners (BIL)

In the previous lessons in General Chemistry 1, you have learned about


the kinetic molecular model of gases. This activity sheet will mainly focus on
the kinetic molecular model of solids and liquids.

Matter can exist in three main different states: namely, solid, liquid, and
gas. The most common example of which is water. You only have to think about
water to appreciate how different the
three states of matter are. Steam
bathing, drinking, and ice skating are all
done in contact with water in its various
forms. But how do these states of matter
differ with each other? Understanding the
kinetic molecular model of the three
states will answer this question.

What is Kinetic Molecular Theory?

The kinetic-molecular theory is a theory that explains the states of matter


and is based on the idea that matter is composed of tiny particles that are
always in motion. The theory helps explain observable properties and behaviors
of solids, liquids, and gases. An application of the theory is that it helps to
explain why matter exists in different phases (solid, liquid, and gas) and how
matter can change from one phase to another.

NOTE: Practice personal hygiene protocols at all times


1
The state of a substance depends on the balance between the kinetic
energy of the individual particles (molecules or atoms) and the intermolecular
forces. The kinetic energy keeps the molecules apart and moving around, and
is a function of the temperature of the substance. The intermolecular forces are
attractive forces that try to draw the particles together.

Postulates of Kinetic Molecular Theory

The Kinetic Molecular Theory (KMT) is based on a series of postulates. Some


of the postulates of KMT are as follows:

1. Matter is made of particles that are constantly in motion. This energy in


motion is called kinetic energy.
2. The amount of kinetic energy in a substance is related to its temperature.
Increased temperature means greater speed.
3. There is space between particles. The amount of space in between
particles is related to the substance's state of matter.
4. Phase changes happen when the temperature of the substance changes
sufficiently.
5. There are attractive forces in between particles called intermolecular
forces. The strength of these forces increase as particles get closer
together.

KMT of Liquids and Solids

The principal difference between the condensed states (liquids and


solids) and the gaseous state is the distance between molecules. In a liquid,
the molecules are so close together that there is very little empty space. Thus,
liquids are much more difficult to compress than gases, and they are also much
denser under normal conditions. Molecules in a liquid are held together by one
or more types of attractive forces. A liquid also has a definite volume, because
molecules in a liquid do not break away from the attractive forces. The
molecules can, however, move past one another freely, and so a liquid can
flow, can be poured, and assumes the shape of its container.

In a solid, molecules are held rigidly in position with virtually no freedom


of motion, so they only vibrate only about fixed positions. There is even less

NOTE: Practice personal hygiene protocols at all times


2
empty space in a solid than in a liquid because their particles are tightly packed.
Thus, solids are almost incompressible and possess definite shape and
volume. This is due to stronger intermolecular force of attraction compared to
liquids.

Change in
phase: The
figure shows
the orientation
of particles in
each state and
the direction of
phase change
due to addition
https://www.siyavula.com/read/science/grade-8/particle-model-of-matter/06- and removal of
particle-model-of-matter?id=toc-id-4
kinetic energy.

Learning Competency:

Use the kinetic molecular model to explain properties of liquids and solids
(STEM_GC11IMFIIIa-c-99)

Activity 1: Color Drop

Estimated time frame: 5 minutes


Materials:
• a glass of hot water
• a glass of cold water
• Liquid food color
Note: You can dissolve a pinch of powdered food color, if you don’t have a
liquid food color. You can use liquid detergent or fabric conditioner if the food
color is not available.

NOTE: Practice personal hygiene protocols at all times


3
Procedure:
1. Label the two glasses hot and cold with respect to their temperature to avoid
confusion.
2. Put a equal drops of food color into the two glasses at the same time. Do not
stir. Observe what happens and compare.

Safety Precaution: Be careful in handling hot water. Always ask the


assistance of your parents if necessary.

Illustration of the experiment:

COLD HOT

Source: https://www.youtube.com/watch?v=Vwkw_Z6luIg

Questions:
1. What happens after dropping the food color into the water? Why did this
happen?
______________________________________________________________
______________________________________________________________
____________________________________________

2. How can you compare the rate of diffusion in the two glasses? What is the
reason for this difference?
______________________________________________________________
______________________________________________________________
____________________________________________

3. How can you relate the kinetic molecular theory with the experiment?
______________________________________________________________
______________________________________________________________
____________________________________________

4
NOTE: Practice personal hygiene protocols at all times
4. What do you think will happen if the substances does not follow kinetic
molecular theory?
______________________________________________________________
______________________________________________________________
____________________________________________

Activity 2: What’s the Matter?

Directions: Based on the picture below, identify what state of matter is


being represented. Write a brief description based on the arrangement of
their particles and give three examples for each state.

State: ________ State: ________ State: ________

Arrangement of Arrangement of Arrangement of


Particles: Particles: Particles:
_______________ _______________ _______________
_______________ _______________ _______________
_______________ _______________ _______________

Examples: Examples: Examples:

• ____________ • ____________ • ____________


• ____________ • ____________ • ____________
• ____________ • ____________ • ____________

5
NOTE: Practice personal hygiene protocols at all times
Activity 3: Describe Me

Directions: Compare the properties of solids and liquids by completing the table
based on their kinetic molecular model.

Characteristic Solid Liquid


Intermolecular force
Shape
Volume
Density
Compressibility
Arrangement of
particles
Motion of molecules
Fluidity

ACTIVITY 4: Be a Science Detective!

Directions: Investigate and analyze the given situation. Provide an explanation for the
phenomenon.
Based on the kinetic molecular model, solids usually have higher densities
compared to liquids. However, ice, a solid form of H2O floats on liquid water which
means ice is less dense than water. What do you think is the reason for this
phenomenon? What is the biological significance of this concept?
___________________________________________________________________
___________________________________________________________________
___________________________________________________________________
___________________________________________________________________
___________________________________________________________________
___________________________________________________________________
____________________________________________________________.

6
NOTE: Practice personal hygiene protocols at all times
Activity 5: Let’s Test Your Understanding
Directions: Read and analyze each item carefully. Encircle the letter of the
correct answer.
1. In the broadest sense, what was the Kinetic Molecular Theory designed to
explain?
A. Phase changes
B. The behavior of matter
C. The types of intermolecular force
D. The types of crystalline solids.

2. What happens to the kinetic energy of particles when there is an increase in


temperature?
A. Increases
B. Decreases
C. Reduce by half
D. Does not change

3. Why are solids and liquids considered as condensed states?


A. Their intermolecular force is weaker compared to gas.
B. They have smaller spaces between molecules compared to gas
C. They have lower densities.
D. Their particles vibrate at fix position.

4. Which of the following properties pertains to solid only?


A. They expand slightly when heated.
B. The particles are able to slide past one another.
C. The kinetic energy is sufficient to overcome the attractive forces.
D. They diffuse extremely slow because the particles are tightly packed.

7
NOTE: Practice personal hygiene protocols at all times
5. Rank the matter based on decreasing the relative strength of attractive forces
between particles.
A. liquid, solid, gas
B. solid, liquid, gas
C. gas, liquid, solid
D. liquid, gas, solid

6. Rank the matter based on decreasing amount of space between particles.


A. liquid, solid, gas
B. solid, liquid, gas
C. gas, liquid, solid
D. liquid, gas, solid

7. Which of the following is common to both solids and liquids?


A. They are slightly compressible.
B. They have fixed shape ad volume.
C. They have sufficient kinetic energy to overcome their attractive forces.
D. They have a strong intermolecular forces of attraction and vibrate in
fixed position.

8. Which of the following explains why gases are less dense than solids?

A. Gases have faster moving particles that are very close together.
B. Gases have faster moving particles that are far apart.
C. Gases have slow moving particles that are very close together.
D. Gases have slow moving particles that are close together.

8
NOTE: Practice personal hygiene protocols at all times
9. What happens to water molecules when cooled?
A. The water molecules become excited.
B. The water molecules slide past each other.
C. The water molecules become fixed in position.
D. The forces between molecules breaks.

10. Which of the following has the strongest intermolecular force?


A. Ice
B. Steam
C. Water
D. All have the same strength of IMF

REFLECTION:

1. I learned that _____________________________________________________


_________________________________________________________________
_______________________________________________________

2. I enjoyed most on _________________________________________________


_________________________________________________________________
_______________________________________________________

3. I want to learn more on _____________________________________________


_________________________________________________________________
_______________________________________________________

9
NOTE: Practice personal hygiene protocols at all times
References:
The Kinetic Molecular Theory: Properties of Solids and Liquids. (2015, January 21).
Retrieved from https://study.com/academy/lesson/the-kinetic-molecular-theory-
properties-of-solids-and-liquids.html.
Chang, Raymond. 2010. Intermolecular Forces and Liquids and Solids. Chemistry,
10th edition (pp. 460-462). New York: McGraw-Hill.

10
NOTE: Practice personal hygiene protocols at all times
ANSWER KEY
Activity 1: COLOR DROP
Q1. The color slowly goes down the glass of water, spreads out and ultimately color
the entire water blue even without stirring. This happened because there is a constant
motion of water molecules due to their kinetic energy.
Q2. The rate of diffusion in hot water is faster than in cold water. This is because hot
water has higher kinetic energy than in cold water, thus, molecules move faster and
the food color will diffuse quicker. As stated in the kinetic molecular theory, the higher
the temperature, the higher the kinetic energy.
Q3. Kinetic Molecular Theory explains this experiment. In the water, there are spaces
between their molecules. This results to a constant movement of molecules. If the
kinetic energy increases due to temperature increase, the speed of molecules also
increases. That is why when you put a liquid substance in water, it will eventually
spread-out even without stirring.
Q4. If the water and food color does not follow kinetic molecular theory, the food color
will not spread as it should be.

11
NOTE: Practice personal hygiene protocols at all times
ACTIVITY 2: WHAT’S THE MATTER?

Directions: Based on the picture below, identify what state of matter is


being represented. Write a brief description based on the arrangement of
their particles and give three examples for each state.

State: _GAS__ State: _SOLID___ State: LIQUID

Arrangement of Arrangement of Arrangement of


Particles: The Particles: The Particles: The
particles are very particles are tightly particle are far from
distant from one packed with each each other and slide
another. other. past each other.

Examples: Examples: Examples:

• Oxygen______ • Wood______ • Water_____


• Air__________ • Steel_______ • Vinegar____
• Carbon dioxide • Rock_______ • Oil_______

ACTIVITY 3: DESCRIBE ME

Characteristic Solid Liquid


Intermolecular force Strong Strong but weaker than
solid
Shape Definite Indefinite
Volume Definite Definite
Density Have higher density than High density
liquids
Compressibility Virtually incompressible Slightly compressible

12
NOTE: Practice personal hygiene protocols at all times
Arrangement of Have ordered arrangement Have disordered
particles that is closed to each other clusters of particles that
are quite close together
Motion of molecules Vibrates in fixed position Have random motion
Fluidity Does not flow/Not fluid Flows freely/ Fluid

Activity 4: SCIENCE DETECTIVE!


Solids have higher densities compared to their liquid form since the particles in
solid are more compact. However, ice is less dense than water because the volume
of ice is greater than in liquid with the same amount of matter. This is due to the
anomalous behavior of water when it reaches 4 degrees Celsius. Normally, when a
matter is cooled, it contracts. But water does not obey this when it reaches 4 degrees
Celsius. That is why ice is less dense than liquid.
This phenomenon has a biological significance. In places like Antarctica, where
temperature is below the freezing point, fishes and other marine life managed to
survive. Since ice has lesser densities than water, it will float and it will remain at the
surface preventing the bottom to freeze. This will enable marine life to thrive in these
regions.
Activity 5: MULTIPLE CHOICE
1. B
2. A
3. B
4. D
5. B
6. C
7. A
8. B
9. C
10. A

Prepared by:

LESTERWIN UDARBE
Gonzaga National High School

13
NOTE: Practice personal hygiene protocols at all times
GENERAL CHEMISTRY 2

Name: ____________________________ Grade Level: _________


Date: _____________________________ Score: ______________

LEARNING ACTIVITY SHEET


INTERMOLECULAR FORCES

Background Information for the Learners (BIL)

In the preceding lesson, we have noted the differences in properties of matter


in gas phase from those in the liquid and solid phases can be attributed to the attractive
forces in solid and liquid molecules while gas molecules have negligible or no
attractions at all.

The condensation of gaseous substance to form liquids which in turn form


solids could be explained by the attractive forces called intermolecular forces.

Intermolecular forces vs. Intramolecular forces

It is important to note the difference between


intermolecular forces and intramolecular forces. As
discussed in General Chemistry 1, atoms can form
stable units called molecules by sharing electrons.
This is called the intramolecular bonding.
Intramolecular force in water molecule
which hold hydrogen and oxygen atom.
Intramolecular (within molecules) forces holds atoms
together in a molecule. Intramolecular forces stabilize
individual molecules. Generally, these forces are simply the chemical bonds such as
ionic and covalent bonding.

On the other hand,


Intermolecular forces are
attractive forces between
molecules. Intermolecular forces
are responsible for the non-ideal Intermolecular force in water molecule which hold two
molecules together.
behavior of gases, but they exert more influence in the condensed phases of matter

14
NOTE: Practice personal hygiene protocols at all times
which are liquids and solids. Intermolecular forces are collectively known as van der
Waals forces named after Dutch chemist, Johannes van der Waal. Van der Waals
forces are electrical in nature; that is, they result in the attraction between centers of
opposite charge in two molecules close to each other.

It is important to recognize that when a substance such as water changes from


solid to liquid to gas, the molecules remain intact. The changes in states are due to
changes in the forces among the molecules rather than in those within the molecules.
In ice, the molecules are virtually locked in place, although they can vibrate about their
positions. If energy is added, the motions of the molecules increase, and they
eventually achieve the greater movement and disorder characteristic of liquid water.
The ice has melted. As more energy is added, the gaseous state is eventually reached,
with the individual molecules far apart and interacting relatively little. However, the gas
still consists of water molecules. It would take much energy to overcome the covalent
bonds and decompose the water molecules into their component atoms. This can be
seen by comparing the energy needed to vaporize 1 mole of liquid water (40.7 kJ) with
that needed to break the -OOH bonds in 1 mole of water molecules (934 kJ).

Types of Intermolecular Forces

The intermolecular forces of attraction in substances includes Dipole-dipole,


London dispersion forces, hydrogen bonding and ion-dipole forces.

London Dispersion forces

London dispersion forces, or simply dispersion forces, are intermolecular forces


of attraction that exist between all atoms and molecules. In addition, dispersion forces
are the only kind of intermolecular forces present among symmetrical nonpolar
substances such as O2 and CO2 and monoatomic species such as noble gases.
Without dispersion forces, such substances could not condense to form liquids or
solidify to form solids.

15
NOTE: Practice personal hygiene protocols at all times
Dispersion forces are weak
attractive forces that results from
the continuous movement of
electrons in particles. Nonpolar
molecules have zero dipole
moment because their electron
density is uniform and symmetrical.
Nevertheless, the electrons have
some freedom to move around the molecule. This induces temporary dipoles
(instantaneous dipoles) in neighboring atoms or molecules. As electron clouds
become larger and more diffuse, they are attracted less strongly by their own positive
nuclei. Thus, they are more easily distorted, or polarized by the adjacent/nearby nuclei.

Polarization increases with increasing numbers of electrons and therefore with


increasing size of molecules. Therefore, dispersion forces are generally stronger for
molecules that are larger or have more electrons. As an example, between helium
and argon, two argon atoms will have greater dispersion force because they are bigger
than helium atoms.

Dipole-dipole Forces
Dipole-dipole forces are attractive forces between polar molecules, that is,
between molecules that possess dipole moments. Their origin is electrostatic, and they
can be understood in terms of Coulomb’s law. The larger the dipole moment, the
greater the force. Dipole-dipole forces are the attraction between the positive end of
one molecule and the negative end of
another.

Two molecules of HCl interacts. HCl has both positive and


Dipoles form when there is a large
negative end so it is a polar molecule and exhibits a dipole
moment. The solid lines represent intramolecular force while difference in electronegativity between
the broken lines represent intermolecular force.
two atoms joined by a covalent bond.

Hydrogen bonding

16
NOTE: Practice personal hygiene protocols at all times
Hydrogen bond is a special case of very
strong dipole-dipole interaction. They are not
chemical bonds in formal sense. Strong
hydrogen bonding occurs among polar
covalent molecules containing H and one of the
three small, highly electronegative elements – F,
O, or N.
Like ordinary dipole-dipole interactions,
hydrogen bonds result from the attractions
between + (partial positive) atoms of one molecule, in this case H atoms and the
− (partial negative) atoms of another molecule. The + H is attracted to a lone pair of
electrons on an F, O, or N atom. Typically, a hydrogen bond is about five to ten times
stronger than other dipole-dipole interactions.

The image shows


the unusual high
boiling points of
NH3, H2O and HF
compared with
those other
hydrides of the
same group
because of
hydrogen
bonding

Ion-Dipole Forces
Ion-dipole force acts between an ion (either cation or anion) and a polar
molecule. When an ionic compound is placed in a solution of water, the positive end
of the ionic compound becomes surrounded with the partial negative end of the ionic
compound, in turn, becomes surrounded by the partial positive hydrogen ion in water.
In short, the positive pole is attracted to the negative ion (anion), while the negative
pole is attracted to a positive ion (cation).

17
NOTE: Practice personal hygiene protocols at all times
Ion- dipole
interactions are involved in
solution process like in the
case of sodium chloride
(table salt) dissolving in
water. In this case, Na+ and
Cl- ions are dispersed
amidst water molecules. The
Na+ will be surround by the partial negative oxygen of water molecule while the Cl- will
be surround by the partial positive H of water molecule.

The strength of this interaction depends on the


charge and size of the ion and on the magnitude of the
dipole moment and size of the molecule. The charges
on cations are generally more concentrated because
cations are usually smaller than anions. Therefore, a
cation interacts more strongly with dipoles than does an
anion having a charge of the same magnitude.

These four intermolecular forces vary in strength. Ion dipole forces is the
strongest of the four, followed by hydrogen bonding being a special type of dipole-
dipole. Dipole-dipole is weaker than the ion-dipole and hydrogen bonding while
London dispersion forces is the weakest.

18
NOTE: Practice personal hygiene protocols at all times
Learning Competency:
Describe and differentiate the types of intermolecular forces (STEM_GC11IMFIIIa-c-
100)

Activity 1: OPERATION: Crossword Puzzle


Directions: Complete the crossword by filling in the boxes to form a word that fits
each clue.

Across:
1. This is a special case of very strong dipole-dipole interaction.
4. This holds atoms together in a molecule.
6. Intermolecular force present among symmetrical nonpolar substance
8. These are attractive forces between molecules

Down:
2. This is an intermolecular force that acts between an ion and a polar
molecule.
3. A collective term used to define attraction of intermolecular forces.
5. These are attractive forces between polar molecules.
7. This is an element that forms hydrogen bonding.

19
NOTE: Practice personal hygiene protocols at all times
Activity 2: Go with the Flow
Directions: Analyze the flow chart carefully and answer the missing parts based on
the words given at the box below.

Hydrogen Bonding Dipole-dipole Ionic Boding


Van der Waals forces London Dispersion Ion-dipole forces
Ne H2S KCl NaCl KBr in water
H2O CH3Cl H2 NH3 KCl in water

Activity 3: Classify Me
Directions: Determine what type of
intermolecular force/s is/are present in each compound.
1. Sulfur dioxide (SO2) ____________________________
2. Nitrogen gas (N2) _______________________________

20
NOTE: Practice personal hygiene protocols at all times
3. Lithium chloride (LiCl) dissolved in water __________________
4. Carbon tetrachloride (CCl4) ______________________________
5. Ethanol (C2H5OH) _____________________________
6. Hydrogen peroxide (H2O2) _______________________
7. Hydrogen iodide (HI) __________________
8. Ammonium bromide (NH4Br) solution _____________________
9. Ethylene (C2H4) __________________________
10. Phosphorus trichloride (PCl3) ____________________

Activity 4: Describe Me
Directions: Below are five chemical compounds. In the first column, determine the
type of intramolecular force for each compound while on the second column,
determine the strongest type of intermolecular force that each compound have. Justify
your reason in identifying their intermolecular force in the third column. An example is
provided below.

Compound Type of Strongest type Reason for classifying


Intramolecular of
force/ chemical intermolecular
bond force present
Example: The compound exhibits
Covalent bond London London dispersion force
Methane dispersion because it is a nonpolar
CH4 covalent compound.

1. Hydrogen
fluoride
HF

2. Magnesium
chloride
(MgCl2) in
Water

21
NOTE: Practice personal hygiene protocols at all times
3. Hydrogen
bromide
HBr

4. Silicon
dioxide
SiO2

5. Fluorine
molecule
F2

22
NOTE: Practice personal hygiene protocols at all times
Activity 5: WHAT IF?
Directions: Investigate and analyze the given situation. Provide a detailed
explanation of the situation.
Water is present in almost every living thing including human beings. It
was discussed that water possesses a unique bond known as hydrogen bond.
What do you think will happen if hydrogen bonding in water does not exist at all?
___________________________________________________________________
___________________________________________________________________
___________________________________________________________________
___________________________________________________________________
___________________________________________________________________
___________________________________________________________________
___________________________________________________________________
___________________________________________________________________
___________________________________________________________________
___________________________________________________________________
___________________________________________________________________
___________________________________________________________________
___________________________________________________________________
___________________________________________________________________
______________________________________________________

23
NOTE: Practice personal hygiene protocols at all times
Reflection:
1. I learned that _______________________________________________
_________________________________________________________________
_______________________________________________________

2. I enjoyed most on ____________________________________________


_________________________________________________________________
_______________________________________________________

3. I want to learn more on ________________________________________


_________________________________________________________________
_______________________________________________________

24
NOTE: Practice personal hygiene protocols at all times
References:
Chang, Raymond. Chemistry, 10th edition. New York: McGraw-Hill, 2010.
Whitten, Kennet W., et al. General Chemistry, 7th edition. USA: Brooks/Cole, 2004.
Zumdahl, Steven S., et al. Chemistry, 7th edition. USA: Houghton Mifflin Company,
2007
Bayquen, Aristea., et al. General Chemistry 2. Quezon City: Phoenix Publishing
House, Inc., 2016.
Ilao, Luciana V., et al. General Chemistry 2. Manila: Rex Bookstore, Inc., 2017
Intermolecular Forces. (2020, May 25). Retrieved from
https://chem.libretexts.org/Bookshelves/General_Chemistry/Map%3A_Chemistry_Th
e_Central_Science_(Brown_et_al.)/11%3A_Liquids_and_Intermolecular_Forces/11.
2%3A_Intermolecular_Forces

25
NOTE: Practice personal hygiene protocols at all times
Answer key:
Activity 1: OPERATION: Crossword Puzzle

26
NOTE: Practice personal hygiene protocols at all times
Activity 2: Go With the Flow

Activity 3:
Classify Me
Directions: Determine what type of intermolecular force/s is/are present in each
compound.
1. Dipole-dipole force and London Dispersion
2. London dispersion force
3. Ion-dipole force and London Dispersion
4. London Dispersion force
5. Hydrogen Bonding and London Dispersion
6. Hydrogen bonding and London Dispersion
7. Dipole-Dipole interaction and London Dispersion
8. Ion-dipole force and London Dispersion
9. London Dispersion force
10. Dipole-dipole force and London Dispersion

27
NOTE: Practice personal hygiene protocols at all times
Activity 4: Describe Me
Compound Type of Strongest type Reason for classifying
Intramolecul of
ar force/ intermolecular
chemical force present
bond
Example: The compound exhibits
Covalent London London dispersion force
Methane bond dispersion because it is a nonpolar
CH4 covalent compound.

1. Hydrogen Covalent Hydrogen The compound exhibits


bond Bonding hydrogen bonding
fluoride
because hydrogen is
HF bonded to highly
electronegative fluorine.

2. Magnesium MgCl2 – ionic Ion-dipole force The compound exhibits


compound ion-dipole force because
chloride
there is an attraction
(MgCl2) in Water – between ions and polar
Covalent molecule.
Water
compound
3. Hydrogen Dipole-dipole The compound exhibits
Covalent force dipole-dipole interaction
bromide
compound because the molecule is
HBr polar.

4. Silicon Covalent London The compound exhibits


compound Dispersion London dispersion force
dioxide
forces because it is a nonpolar
SiO2 covalent compound.

Covalent London The compound exhibits


compound Dispersion London dispersion force
5. Fluorine molecule
forces because it is a nonpolar
F2 covalent compound.

28
NOTE: Practice personal hygiene protocols at all times
Activity 5: What If?
Directions: Investigate and analyze the given situation. Provide a detailed
explanation of the situation.
Water is present in almost every living thing including human beings. It
was discussed that water possesses a unique bond known as hydrogen bond.
What do you think will happen if hydrogen bonding in water does not exist at all?
Possible Answer:
Our body is made up of more than 60% water. No one can live with the absence
of water. Water has many unusual and unique properties that is very important to
human life. One example is the specific heat. Water has a very high specific heat
capacity. This means that it takes a lot of energy to heat water up. Because of this
unique property, water is vital for body temperature regulation. Moreover, water has
an extremely high boiling point, compared to other molecules of similar size. This
means that at normal temperatures, water is a liquid. Therefore, this means that we
can be able to drink water. It also means that many organisms can live in water which
is a very stable and hospitable environment to live in. Water has so many hydrogen
bonds that it ‘sticks’ to itself and even other surfaces. It is this ‘sticking’ that allows
capillary action to occur. This is exploited by plants to draw water up the xylem. Without
the cohesive properties of water, plants would probably only be able to grow to a few
centimeters high. Lastly, because water is polar and exhibits hydrogen bonding, water
is a very good solvent and referred to as the universal solvent. These unique properties
of water can be attributed to hydrogen bonding. Without hydrogen bonding in water,
we could not take advantage of these properties and no one could survive.

Prepared by:

LESTERWIN UDARBE
Gonzaga National High School

29
NOTE: Practice personal hygiene protocols at all times
GENERAL CHEMISTRY 2
Name: ____________________________ Grade Level: _________
Date: _____________________________ Score: ______________

LEARNING ACTIVITY SHEET

PROPERTIES OF LIQUIDS AND INTERMOLECULAR FORCES

Background Information for the Learners (BIL)

Liquids are made up of particles that are close to each other and have kinetic
energy.
The particles are not confined to a rigid position and they move, but they can
only travel at a short distance before they collide with each other and change the
direction of motion. They roll and slide on top of one another and flow. Since the
molecules flow, they take the shape of their container and diffuse moderately to a fixed
volume.
Liquids have moderately high density since they occupy a fixed volume and the
particles are attracted to each other. They also have low compressibility and thermal
expansion.
The kinetic energy of the molecules break away from their neighbor and thus,
the particles are joined by intermolecular forces.
Most liquids exist as molecules at room temperature. The presence of the
intermolecular forces results in special properties. The physical properties of liquids
depends on the type of the different intermolecular forces.

➢ SURFACE TENSION

Surface tension is the force that causes the surface of a liquid to contract. It is
the property of the surface of a liquid that allows it to resist an external force, due to
the cohesive nature of its molecules.
Phenomena such as insects walking on the surface of water, droplets of liquid
being spherical in shape, and needle remaining suspended on the surface of water
can all be explained in terms of surface tension.

30
NOTE: Practice personal hygiene protocols at all times
The strength of surface tension depends on the intermolecular force of
attraction. If the intermolecular force of attraction of a liquid is strong, then there is a
greater force needed to break through the surface and the greater the surface tension
is. Since the intermolecular forces vary in nature and strength, surface tension is
different for different form of liquids. Water has a high surface tension because of its
ability to form a hydrogen bond.
Temperature affects surface tension, an increase in the liquid’s temperature
causes water molecules at the surface to evaporate resulting to the weakening of the
force of attraction. Therefore, there is less force needed to break through the surface
of the molecules, this decreases surface tension.

Fig.1. The forces of attraction between all the


molecules in the water pull molecules at the
surface together. The molecules look like thin
elastic membrane of slightly denser
molecules. The geometry of the legs of the
water strider allows the insect to cross the
water without sinking.

Image Source: http://


hyperphysics.phy-astr.gsu.edu/hbase/surten.html

Fig.2 examples of how surface tension is manifested.

31
NOTE: Practice personal hygiene protocols at all times
Molecules within a liquid are pulled in all directions by intermolecular forces.
Molecules at the surface are pulled downward and sideways by other molecules, not
upward away from the surface (shown in the diagram below).

These intermolecular forces tend to pull the molecules into the liquid and cause
the surface to tighten like an elastic film or “skin”.

Fig. 3 Intermolecular forces


that acts on a liquid.

(Image Source:
www.bville.org/.../AP%20Chapter%2011%20Intermolecular%20Forces)
Capillary action is the tendency of a liquid to rise in narrow tubes or be drawn
into small openings such as those between grains of a rock. Capillary action, also
known as capillarity, is a result of intermolecular attraction between the liquid and solid
materials.
Capillary action is shown by water rising spontaneously in capillary tubes. A
thin film of water adheres to the wall of the glass tube as water molecules are attracted
to atoms making up the glass (SiO2). Surface tension causes the film of water to
contract and pulls the water up the tube.

Fig.4 Colored water rising


up in tubes.

Image Source: http://


water.usgs.gov/edu/capillaryaction.html

32
NOTE: Practice personal hygiene protocols at all times
Two types of forces are involved in capillary action:

Cohesion is the intermolecular attraction between like molecules (the liquid


molecules).

Adhesion is an attraction between unlike molecules (such as those in water


and in the particles that make up the glass tube).

These forces also define the shape of the surface of a liquid in a cylindrical
container (the meniscus!)

➢ VISCOSITY

Viscosity is the resistance of fluids to flow. A liquid’s resistance (friction) to


flow exists between the molecules of liquid when they move past each other. The
greater the resistance in flowing, the more viscous the liquid is.
Maple syrup is usually made from the xylem sap of sugar maple, red maple
or black maple trees. It is boiled down so it becomes more concentrated and
viscous liquid. Maple syrup is more viscous than water. The difference in viscosity
between the two liquids is a measure of their intermolecular force of attraction. In
order to flow, molecules must move, roll and slide over one another. A liquid with
low intermolecular force allows its molecules to move freely and, therefore, has
lower viscosity.

An increase in temperature causes kinetic energy to increase. Heat breaks


the intermolecular forces causing the liquid molecules to move faster. This makes
the molecules flow more readily. Therefore, an increase in temperature decreases
viscosity.

33
NOTE: Practice personal hygiene protocols at all times
Since the structure of maple syrup contains a lot of O-H bond compared to
water, more H-bonds are formed in maple syrup. The greater the number of H-
bonds, the stronger the intermolecular force of attraction is, and the higher the
viscosity of the liquid.

Fig.5 The structure of maple syrup


shows the O-H bonds present. This
allows the formation of many
hydrogen bonds.

Viscosity is expressed in units of centipoise. The table below gives viscosities of


liquids of some pure substances. Water has viscosity of 1 centipoise or 0.001 Pa/s at
20 0C. Substances with lower viscosities include carbon tetrachloride and benzene.
Glycerol has a resistance to flow of more than a thousand times greater than water.
Liquids that have strong intermolecular forces have higher viscosities than those
that have weak intermolecular forces.

34
NOTE: Practice personal hygiene protocols at all times
Viscosity decreases as temperature increases: hot molasses flows much faster
than cold molasses.

➢ VAPOR PRESSURE

Vaporization is a change of state from liquid to gas, while the opposite process
(gas to liquid) is condensation. When liquid molecules break free from their neighbors
and escape into the gas phase, the process is called evaporation.
Vaporization is a broader term that includes evaporation and boiling. Gas and
vapor are similar but not the same. Vapor is used to refer to the gaseous state of a
substance which is normally a liquid or solid at room temperature. The average kinetic
energy of the liquid molecules of a substance depends on temperature. Most liquid
particles have higher kinetic energy and some others move at a slower pace.
Substances that evaporate readily are volatile. They have weak intermolecular
forces of attraction. Some examples of volatile liquids are alcohol, gasoline, paint
thinner and dry-cleaning solvents. Volatile substances burn more readily since they
easily combine with oxygen. As evaporation continues, the slow -moving particles are
left behind. Since the average kinetic energy is directly proportional to temperature,
temperature decreases. Therefore, evaporation is accompanied by cooling.
In a closed container half-filled with liquid, the fast-moving molecules also escape
into the gas phase forming vapor at the space above the liquid.
Gas molecules move in random directions, collide with other gas particles and the
walls of the container. Some will strike the liquid surface and condense back into it. In
the closed flask, none of the gas particles are able to get out of the container, and
eventually, the number of molecules that goes into the gaseous state would equal the
number of molecules that condenses back.
When the rate of condensation of the gas becomes equal to the rate of
evaporation of the liquid, the gas in the container is said to be in equilibrium with the
liquid.

35
NOTE: Practice personal hygiene protocols at all times
liquid ⇋ vapor (gas)
In this condition, the amount of gas and liquid no longer changes.

Fig.6
Equilibrium
between liquid
and gas.

Image Source:
http://wpscms.pearsoncmg.com/wps/media/objects/3662/3750037/Aus_content_10/Fig10-20.jpg

Like any gas sample, the molecules in the gaseous state over its liquid create a
pressure. The greater the number of gaseous particles, the greater the pressure
exerted by the gas. The pressure exerted by the gas in equilibrium with a liquid in a
closed container at a given temperature is called the equilibrium vapor pressure or
simply vapor pressure of the liquid.
The equilibrium vapor pressure is the maximum vapor pressure of a liquid at a
given temperature and that it is constant at a constant temperature. It increases with
temperature.
Vapor pressure is independent of the amount of liquid as well as the surface area
of the liquid in contact with the gas.
When temperature is high, more molecules have enough energy to escape from
the liquid. At a lower temperature, fewer molecules have sufficient energy to escape
from the liquid.

Consider the vapor pressure of the following substances;

36
NOTE: Practice personal hygiene protocols at all times
Ethyl alcohol and water have very low vapor pressures. Both liquids have the
strong dipole-dipole interaction called hydrogen bonding. Acetone is polar but does
not have H-bonding. Its vapor pressure is of intermediate value. Pentane is a nonpolar
substance, and its vapor pressure is high compared to those of water and ethyl
alcohol.
When liquids evaporate, the molecules have to have sufficient energy to break
the attractive forces that hold them in the liquid state. The stronger these
intermolecular forces are, the greater the amount of energy needed to break them.
For some substances with weak intermolecular forces, the energy requirement
is easily obtained from collisions with other molecules and absorption of energy from
the surroundings. Many molecules can vaporize, resulting in a high vapor pressure.
For molecules with strong intermolecular forces, gathering enough energy may not be
as easy, and register low vapor pressures.
The stronger the intermolecular forces of attraction, the lower the vapor pressure
of a liquid.

➢ MOLAR HEAT OF VAPORIZATION

The relationship between vapor pressure and strength of intermolecular forces is


consistent with the trends in two other properties of liquids, the enthalpy or molar heat
of vaporization, and the boiling point of the liquid.
The molar heat of vaporization (ΔHvap) is the energy required to vaporize 1 mole
of a liquid at a given temperature. H is the symbol for enthalpy, which means heat
content at a given standard condition.

Molar heat of vaporization and boiling points of selected substances.

37
NOTE: Practice personal hygiene protocols at all times
The heat of vaporization may be considered a measure of the strength of
intermolecular forces in a liquid. If the intermolecular attraction is strong, it takes a lot
of energy to free the molecules from the liquid phase and the heat of vaporization will
be high.
It is easier to vaporize acetone (lower Hvap) than water (higher Hvap) at a given
temperature, and more acetone escapes into the vapor phase at a given temperature.
Acetone is a polar substance but has no H-bonding. It has weaker intermolecular
forces than water, and therefore acetone molecules are held less tightly to one another
in the liquid phase.
A practical way to demonstrate differences in the molar heat of vaporization is
by rubbing acetone on your hands. Compare what is felt when water is used. Acetone
has a lower ΔHvap than water so that heat from our hands is enough to increase the
kinetic energy of the these molecules and provide additional heat to vaporize them.
As a result of the loss of heat from the skin, our hands feel cool.

➢ BOILING POINT
The boiling point of a liquid is the temperature at which the liquid converts into
a gas.
A liquid boils when its vapor pressure equals the pressure acting on the surface
of the liquid. The boiling point is the temperature at which the vapor pressure of a liquid
is equal to the external pressure.
The normal boiling point is the temperature at which the liquid converts to a gas
when the external pressure is 1 atm. The normal boiling point of water is 100 OC. The
boiling point of a liquid depends on the external pressure. For example, at 1 atm, water
boils at 100 OC, but if the pressure is reduced to 0.5 atm, water boils at only 82 OC.
The boiling point is related to molar heat of vaporization: the higher ΔHvap, the
higher the boiling point.

Learning Competency:
Describe the following properties of liquid, and explain the effect of intermolecular
forces on these properties; surface tension, viscosity, vapor pressure, boiling point,
and molar heat of vaporization (STEM_GC11IMFIIIa-c-102

38
NOTE: Practice personal hygiene protocols at all times
Activity 1: KNOW ME!
Directions: Determine which of the following terms refer to each statement.
a. Boiling point
b. Heat of vaporization
c. Surface tension
d. Viscosity

________1. It is responsible for the spherical drops of water and is the reason why
some insects can walk on the surface of water.
________2. It is the measure of a liquid’s resistance to flow.
________3. The temperature at which vapour pressure of the liquid is equal to the
atmospheric pressure.
________4. The heat needed to vaporize a certain amount of liquid.
________5. A certain substance A has a boiling point of 78 degrees Celsius and
substance B has a boiling point of 110 degrees Celsius. Answer the following
questions.
a. Which substance has a higher vapour pressure?
b. Which substance has a stronger force of attraction?
c. Which substance has a higher heat of vaporization?
d. Which one will have a higher viscosity?

Activity 2: I CAN DO IT!


Directions: Identify the words that are being described on the statement below.
Choose the correct answer from the given words below.

Surface tension Viscosity


Vapour pressure Capillary action
Molar heat of vaporization Boiling point
Vapor Liquid flow
Fluid Vaporization

______1. The measure of the elastic force in the surface of a liquid. It is the amount
of energy required to stretch or increase the surface of a liquid by a unit area.

39
NOTE: Practice personal hygiene protocols at all times
______2. A gas or a liquid; a substance that can flow.
______3. The tendency of a liquid to rise in narrow tubes or to be drawn into small
openings.
______4. A measure of a fluid’s resistance to flow.
______5. A gaseous substance that exist naturally as a liquid or solid at normal
temperature.
______6. The change of phase from liquid to vapor (gaseous phase).
______7. The equilibrium pressure of a vapor above its liquid; that is, the pressure
exerted by the vapor above the surface of the liquid in a closed container.
______8. The temperature at which a liquid boils. The boiling point of a liquid when
the external pressure is 1 atm is called the normal boiling point.
______9. The energy (usually in kilojoules) required to vaporize 1 mole of a liquid at
a given temperature.

Activity 3: PICTURE SHOWS WHAT I KNOW!


Directions: Describe what is happening to the water molecules in the two flasks
shown in the picture.

Image Source: http://


boomeria.org/physicslectures/heat/equilibrium.jpg

40
NOTE: Practice personal hygiene protocols at all times
Guide questions:
➢ What happens to the molecules of water in the container when the
temperature increases?
➢ Container B shows equilibrium; how does it manifest such balance?
1. _________________________________________________________
_________________________________________________________
__________________________________________
2. _________________________________________________________
_________________________________________________________
__________________________________________

Activity 4: MIND THE GRAPH

Direction: Explain what the plot presents.

a. Plot of vapour pressure of water as it varies with temperature.

➢ How does temperature affects vapour pressure based on the given graph?

Answer:
___________________________________________________________________
_________________________________________________________

41
NOTE: Practice personal hygiene protocols at all times
b.

Image Source: http://wps.prenhall.com/wps/media/objects/ 3311/3391416/


blb1105.html
➢ The graph shows the four common liquids, what happens to their vapour
pressure as temperature increases?
Answer:
___________________________________________________________________
___________________________________________________________________
____________________________________________________

Activity 5: CRITICAL THINKER

Directions: Analyze the problem and answer the questions below.

1. At 50. ° C the vapor pressure of ethanol is 0.30 atm, acetic acid is 0.08 atm,
water is 0.12 atm, and acetone is 0.84 atm.

a. Arrange these substances in order of increasing rates of evaporation.

b. Arrange these substances in order of increasing boiling point temperature

c. Arrange these substances in order of increasing intermolecular forces.

42
NOTE: Practice personal hygiene protocols at all times
2. Given molecular structures of water and glycerol, can you tell why glycerol
has a higher viscosity than water?

Answer:_____________________________________________________________
___________________________________________________________________
___________________________________________________

43
NOTE: Practice personal hygiene protocols at all times
Reflection:

1. I learned that _______________________________________________________


___________________________________________________________________
_________________________________________________________

2. I enjoyed most on ___________________________________________________


___________________________________________________________________
___________________________________________________________________.

3. I want to learn more on _______________________________________________


___________________________________________________________________
___________________________________________________________________.

44
NOTE: Practice personal hygiene protocols at all times
REFERENCES
General Chemistry 2 textbook by Ayson, Marissa F, et.al
General Chemistry 2 Teaching Guide

Mott, V. (n.d.). Introduction to Chemistry. Retrieved August 27, 2020, from


https://courses.lumenlearning.com/introchem/chapter/the-structure-and-
properties-of-water/

(n.d.). Retrieved August 27, 2020, from http://www.sci.sdsu.edu/

Wilkin, D., & Brainard, J. (2020, February 07). Structure and Properties of
Water. Retrieved August 03, 2020, from https://www.ck12.org/biology/structure-
and-properties-of-water/lesson/Biochemical-Properties-of-Water-Advanced-
BIO-ADV/

2. Water Has Excellent Solvent Action. (n.d.). Retrieved August 03, 2020, from
https://www.cropsreview.com/solvent.html

45
NOTE: Practice personal hygiene protocols at all times
ANSWER KEY

Activity 1

Directions: Determine which of the following terms refer to each statement.


a. Surface tension
b. Viscosity
c. Boiling point
d. Heat of vaporization

1. It is responsible for the spherical drops of water and is the reason why some
insects can walk on the surface of water.

➢ Surface Tension

2. It is the measure of a liquid’s resistance to flow.

➢ Viscosity

3. The temperature at which vapour pressure of the liquid is equal to the


atmospheric pressure.

➢ Boiling point

4. The heat needed to vaporize a certain amount of liquid.

➢ Heat of Vaporization

5. A certain substance A has a boiling point of 78 degrees Celsius and


substance B has a boiling point of 110 degrees Celsius. Answer the following
questions.
a. Which substance has a higher vapour pressure?
➢ A
b. Which substance has a stronger force of attraction?
➢ B
c. Which substance has a higher heat of vaporization?
➢ B
d. Which one will have a higher viscosity?
➢ B
As the strength of the IMFs in a series of liquids increases, the boiling points of the
liquids increase. Substance B with higher boiling point has a stronger IMFs (force of
attraction).
The strength of the IMFs in the liquid is a direct indication of the enthalpy of
vaporization (heat of vaporization). The IMFs in substance B are stronger, so the
enthalpy of vaporization is greater than that of substance A.

46
NOTE: Practice personal hygiene protocols at all times
As the strength of the IMFs in a series of liquids increases, the vapor pressures of
the liquids decrease. That’s why substance A has a higher vapor pressure than that
of substance B.
Strong IMFs result in high viscosity for a liquid. Substance B will have a higher
viscosity.

Activity 2

Directions: Identify the words that are being described on the statement below.

1. The measure of the elastic force in the surface of a liquid. It is the amount of
energy required to stretch or increase the surface of a liquid by a unit area.

➢ Surface tension

2. A gas or a liquid; a substance that can flow.

➢ Fluid

3. The tendency of a liquid to rise in narrow tubes or to be drawn into small


openings.

➢ Capillary action

4. A measure of a fluid’s resistance to flow.

➢ Viscosity

5. A gaseous substance that exist naturally as a liquid or solid at normal


temperature.

➢ Vapor

6. The change of phase from liquid to vapor (gaseous phase).

➢ Vaporization

7. The equilibrium pressure of a vapor above its liquid; that is, the pressure
exerted by the vapor above the surface of the liquid in a closed container.

➢ Vapour pressure

8. The temperature at which a liquid boils The boiling point of a liquid when the
external pressure is 1 atm is called the normal boiling point.

➢ Boiling point

47
NOTE: Practice personal hygiene protocols at all times
9. The energy (usually in kilojoules) required to vaporize 1 mole of a liquid at a
given temperature.

➢ Molar heat of vaporization

Activity 3

Directions: Describe what is happening to the water molecules in the two flasks
shown in the picture.

1. The water molecules in the liquid evaporate and go into the vapor phase. In the
open flask, some of the water molecules in the vapor phase find their way out of
the flask are lost to the atmosphere.

2. When a liquid evaporates to a gas in a closed container, the molecules cannot


escape.

48
NOTE: Practice personal hygiene protocols at all times
Activity 4

Directions: Explain what the plot presents.

a. Plot of vapour pressure of water as it varies with temperature.

Answer:
As the temperature increases, the vapor pressure of water also increases.

b.

Answer:
Given in the graph are the vapor pressures for four common liquids: diethyl
ether, ethyl alcohol , water and ethylene glycol, as a function of temperature.
For all four liquids, the vapor pressure increases as temperature increases

Activity 5

49
NOTE: Practice personal hygiene protocols at all times
Directions: Analyze the problem and answer the questions below.

1. At 50. ° C the vapor pressure of ethanol is 0.30 atm, acetic acid is 0.08
atm, water is 0.12 atm, and acetone is 0.84 atm.

a. Arrange these substances in order of increasing rates of


evaporation.

Acetic acid ---- water ---- ethanol ---- acetone

b. Arrange these substances in order of increasing boiling point


temperature

Acetone ---- Ethanol ---- Water ---- Acetic acid

c. Arrange these substances in order of increasing


intermolecular forces.
Acetone ---- Ethanol ---- Water ---- Acetic acid

2. Given molecular structures of water and glycerol, can you tell why glycerol
has a higher viscosity than water?

Answer:
The larger number of –OH groups allow glycerol to form more H-bonds with
other glycerol molecules, making its intermolecular forces stronger than those
of water, and its resistance to flow greater.

Prepared by:

FLORIE MAE UNCIANO


Lallo National High School

50
NOTE: Practice personal hygiene protocols at all times
GENERAL CHEMISTRY 2

Name: ____________________________ Grade Level: _________


Date: _____________________________ Score: ______________

LEARNING ACTIVITY SHEET


PROPERTIES OF WATER AND INTERMOLECULAR FORCES

Background Information for the Learners (BIL)


Water makes up a large proportion of the entire biosphere and of these, 95%
is saltwater and the remaining 5% is freshwater.
Water is locked up in ice and glaciers, deep and shallow underground lakes,
soil, atmosphere, and in rivers. The human body consists of 50-75% water. Water
serves important purposes for life on earth. Water’s unique properties result from the
strong intermolecular force of attraction characterized by the hydrogen bond.
Some substances, like common table salt, NaCl, dissolve in water very easily.
When placed in water, sodium chloride molecules fall apart. The positively charged
sodium ion (Na+) binds to oxygen, while the negatively charged chloride ion (Cl-)
attaches to hydrogen. This property of water allows for the transport of nutrients vital
to life in animals and plants. A drop of rainwater falling through the air dissolves
atmospheric gases. When rain reaches the earth, it affects the quality of the land, lakes
and rivers.

The following are properties of water:


1. Boiling point and freezing point. The high boiling point of water is a
consequence of its strong intermolecular forces of attraction caused by the
formation of the H-bond. It also explains why water is liquid at room
temperature. Due to hydrogen bonding, water molecules cling to each other
(cohesion) and remain in liquid state under temperatures that are favorable to
plants and other living organisms.
Pure water at sea level boils at 100 0C and freezes at 0 0C, but extra
energy is needed to push water molecules into the air. This is called latent
heat—the heat required to change water from one phase to another. At higher

51
NOTE: Practice personal hygiene protocols at all times
elevations (lower atmospheric pressure) water’s boiling temperature
decreases. This is why it takes longer to boil and egg at higher altitudes. The
temperature does not get high enough to cook the egg properly. If a substance
is dissolved in water, then the freezing point is lowered. Energy is lost when
water freezes. A great deal of heat is released into the environment when liquid
water changes to ice. It is lost when the high energy phase of liquid water moves
to the low energy phase of ice. Nights when ice freezes often feel warmer than
nights when ice melts.

2. Specific heat. Specific heat refers to the amount of heat needed to change the
temperature of 1 gram of a substance by 1 oC. For water, its specific heat is
1cal/g oC. It means that water can absorb and release large quantities of heat
without change in temperature. This is the reason why body temperature
remains at 37 oC even when there’s a change in the surrounding. This also
explains why oceans and lakes exert an influence on the climate. If there were
no large bodies of water, the earth would experience great temperature
variations. Water has a high specific heat. The amount of energy required to
raise the temperature of water by one degree Celsius is quite large. Because
so much heat loss or heat input is required to lower or raise the temperature of
water, the oceans and other large bodies of water have relatively constant
temperatures. Thus, many organisms living in the oceans are provided with a
relatively constant environmental temperature. The high water content of plants
and animals living on land helps them to maintain a relatively constant internal
temperature. The specific heat of water is 5 times greater than of sand.

3. Density in its liquid form. Water is the only substance that contracts when
cooled. For most substances, their solid form is denser than their liquid form.
This is because the H-bond is more extensive in its solid state than in its liquid
state. Ice has an open structure because the hydrogen bonds could not get
inside the hexagonal ring structure. This more open structure of the solid form
of water causes the ice to have smaller number of molecules packed in a given
volume. This causes the mass to be lower, hence, the density of ice is lesser
than the liquid water, and, as a result, ice floats on water. This also causes
water in lakes to freeze from the top down.

52
NOTE: Practice personal hygiene protocols at all times
Water is most dense at 4 0C and then begins to expand again (becoming less
dense) as the temperature decreases further. This expansion occurs because
its hydrogen bonds become more rigid and ordered. As a result, frozen water
(ice floats) upon the denser cold water. The expansion of water takes place
even before it actually freezes. This explains why a pond freezes from the
surface down, rather than from the bottom up. As water temperature drops, the
colder water (0-4 0C) where it is less dense— rises to the pond surface. It
freezes to form a lid of ice. This ice insulates the water below from the wintry
chill so that it is less likely to freeze. Organisms that inhabit the pond are able
to survive the frigid winter below the icy surface.

For most substances, solids are denser than liquids. But the special properties of
water make it less dense as a solid. Ice floats on water! Strong hydrogen bonds formed
at freezing 0 0C lock water molecules away from each other. When ice melts, the
structure collapses and molecules move closer together. Liquid water at 4 0C is about
9% denser than ice. This property plays an important role in lake and ocean
ecosystems. Floating ice often insulates and protects animals and plants living in the
water below.

4. Surface tension. The hydrogen bond formation among water molecules


causes water to have high surface tension, as described earlier. This high
surface tension causes water to move from the roots of a tree to the top of very
tall trees and explains why water moves into the fibers of a towel. This
phenomenon is called capillarity.

53
NOTE: Practice personal hygiene protocols at all times
Water molecules at the surface (next to air) hold closely together, forming an
invisible film. Water’s surface tension can hold weight that would normally sink.
You can carefully float a paper clip on top of the water. Some aquatic insects
such as the water strider or pond skater rely on surface tension to walk on
water. Surface tension is essential for the transfer of energy from wind to water
to create waves. Waves are necessary for rapid oxygen diffusion in lakes and
seas. Next to mercury, water has the highest surface tension of all commonly
occurring liquids.
Cohesion—Water molecules stick to each other. This is due to the hydrogen
bonds among the molecules. Water molecules at the surface have a much
greater attraction for each other than for molecules in the air. This cohesiveness
creates a high surface tension at the surface of the water. The water molecules
at the surface crowd together, producing a strong layer as they are pulled
downward by the attraction of other water molecules beneath them.
Adhesion—Water molecules stick to other substances. You can see this
property when water creeps up the inside of a drinking glass. Think of a sponge
or a paper towel used to “soak up” spilled water. This is how water makes things
wet. Water also clings to living things. Most plants have adapted to take
advantage of water’s adhesion that helps move water from the roots to the
leaves. This is called capillary action. This can also be seen as blood moves
through our capillaries, carrying nutrients to each cell within our body. One of
the tallest plants is the redwood tree. Water moves from its roots to its leaves,
more than 90 m above the ground. As a plant loses water through pores in the
leaves, more water moves up from roots and stems to replace the lost water.
The process of water loss by leaves is known as transpiration.
Thermal properties - Water absorbs or releases more heat than many
substances for each degree of temperature increase or decrease. Because of
this, it is widely used for cooling and for transferring heat in thermal and
chemical processes. Differences in temperature between lakes and rivers and
the surrounding air may have a variety of effects. For example, local fog or mist
is likely to occur if a lake cools in the surrounding air enough to cause
saturation—small water droplets are suspended in the air. Large bodies of
water, such as the oceans or the Great Lakes, have a profound influence on
climate. They are the world’s great heat reservoirs and heat exchangers and

54
NOTE: Practice personal hygiene protocols at all times
the source of much of the moisture that falls as rain and snow over adjacent
landmasses. When water is colder than the air, precipitation is curbed, winds
are reduced, and fog banks are formed. These properties of water are crucial
in stabilizing temperatures on earth.

5. Heat of vaporization. Large amount of heat is needed to vaporize a given


amount of water. This causes a significant drop in temperature during
evaporation. When molecules of water absorb heat energy, they move fast in
water. Eventually, the speed of movement of some molecules becomes so fast
allowing them to overcome the intermolecular attraction, detach from the multi-
molecular water, form bubbles, and leave the water surface in gas state. This
property of water helps to cool down the body of living organisms. This is called
evaporative cooling. In humans, body heat is used to vaporize sweat; in plants,
heat is likewise used in converting liquid water to water vapor which then
escapes into the atmosphere. This natural process of vaporizing plant water is
called transpiration.

6. pH. Water molecules have a tendency to ionize. They dissociate into ions
(charged particles) hydrogen ions (H+) and hydroxide ions (OH-). In pure water
a very small number of water molecules form ions in this way. The tendency of

55
NOTE: Practice personal hygiene protocols at all times
water to dissociate is balanced by the tendency of hydrogen ions and hydroxide
ions to reunite to form water. A neutral solution contains an equal number of
hydroxide ions and hydrogen ions. A solution with a greater concentration of
hydrogen ions (H+) is said to be acidic. A solution with a greater concentration
of hydroxide (OH-) ions is said to be alkaline or basic.

Learning Competency:

Explain the properties of water with its molecular structure and intermolecular forces
(STEM_GC11IMFIIIa-c-103)

56
NOTE: Practice personal hygiene protocols at all times
Activity 1: MIND POWER

Part A.
Directions: Identify what property of water is being shown on the pictures
below.

1.

→ _____________________

2.

→ _____________________

3.

→ _____________________

4.

→ _____________________

57
NOTE: Practice personal hygiene protocols at all times
5.

→ _____________________

Part B.
Directions: Answer the following questions briefly but substantially

1. What is cohesion?
______________________________________________________________
______________________________________________________________
______________________________________________________________
______________________________________

2. Why do some water insects able to walk on water? Explain.


______________________________________________________________
______________________________________________________________
______________________________________________________________
______________________________________

3. Define adhesion.
______________________________________________________________
______________________________________________________________
______________________________________________________________
______________________________________

4. Explain how adhesion and cohesion help plants move materials.


______________________________________________________________
______________________________________________________________
_________________________________________________________.

58
NOTE: Practice personal hygiene protocols at all times
5. When is water the densest?
________________________________________________________________
________________________________________________________________
________________________________________________________________
____________________________________________

6. Explain why ice floats.


________________________________________________________________
________________________________________________________________
________________________________________________________________
____________________________________________

7. How is a lake or a river that freezes over helpful to the organisms in the
water?
________________________________________________________________
________________________________________________________________
________________________________________________________________
____________________________________________

8. What property is responsible when water get to the leaves in the top of the
tallest trees against the force of gravity? Explain.
________________________________________________________________
________________________________________________________________
________________________________________________________________
____________________________________________

9. Why is solid water less dense than liquid water?


________________________________________________________________
________________________________________________________________
________________________________________________________________
____________________________________________

59
NOTE: Practice personal hygiene protocols at all times
Activity 2: EXPLORING PROPERTIES

Directions: Choose the letter/letters of the property of water related to each


phenomenon described below. More than one property may be used to explain the
given phenomenon.

a. Water molecules are cohesive, they form hydrogen bonds with each other.
b. Water molecules are adhesive, they form hydrogen bonds with polar surfaces.
c. Water is a liquid at normal physiological temperature.
d. Water has high specific heat.
e. Water has a high heat of vaporization.
f. Water shows high surface tension.

______1. During the winter, air temperature in the northern parts of the planet can
remain below zero degrees Celsius for months, however, the fish and other animals
living in the lakes survive.
______2. Many substances, for example, salt (NaCl) and sucrose, dissolve quickly in
water.
______3. When you pour water into a 25mL cylinder, a meniscus forms at the top of
the water column.
______4. Sweating and the evaporation of sweat from the body surface help reduce
a human’s body temperature.
______5. Water drops that fall on a surface tend to form rounded drops or beads.
______6. If you put the end of a paper towel to a coloured water, the water will move
up into the towel.
______7. A paper clip can float on water.
______8. When you place a straw into a glass of water, the water seems to climb up
the straw before you even place your mouth on the straw.
______9. Water is most dense at about 4°C. As a result, the water at the bottom of a
lake or the ocean usually has temperature of about 4°C.
_____10. If you drop a tiny amount of water onto a very smooth surface, the water
molecules will stick together and form a droplet, rather than spread out over the
surface.

60
NOTE: Practice personal hygiene protocols at all times
Activity 3: WORD HUNT

Part A.
Directions: Fill in the blanks with words that corresponds to the statements below.
Choose the word in the word bank.

covalent solvent deposition dissolve


cohesion adhesion polar viscosity
negatively positively surface tension

1. The hydrogen and oxygen atoms are held together by ____________ bonds.

2. The electrons are not shared equally creating a ________ molecule.

3. The polarity of water allows it to ___________ most substances.

4. Water molecules stick to other water molecules. This property is called


_______.

5. Hydrogen bonds form between adjacent molecules because the _________


charged hydrogen end of one water molecule attracts the ________ charged
oxygen end of another molecule.

Part B.
Directions: Answer the following questions briefly but substantially.

1. When you warm up oil and water. Which temperature will rise faster?
______________________________________________________________
______________________________________________________________
____________________________________________

61
NOTE: Practice personal hygiene protocols at all times
2. What items can you gently “float” on water surface? (e.g., paperclips, needles,
etc.)
______________________________________________________________
______________________________________________________________
____________________________________________
3. What happens to the bonds (hydrogen bonds) when water boils?
______________________________________________________________
______________________________________________________________
____________________________________________

62
NOTE: Practice personal hygiene protocols at all times
Reflection

1. I learned that _______________________________________________________


___________________________________________________________________
_________________________________________________________

2. I enjoyed most on ___________________________________________________


___________________________________________________________________
___________________________________________________________________.

3. I want to learn more on _______________________________________________


___________________________________________________________________
___________________________________________________________________.

63
NOTE: Practice personal hygiene protocols at all times
References:

General Chemistry 2 Textbook by Ayson, Marissa F, et.al


Chemistry and Physics of Water Module 1; w4l.prel.org
General Chemistry 2 Teaching Guide
2. Water Has Excellent Solvent Action. (n.d.). Retrieved August 03, 2020, from
https://www.cropsreview.com/solvent.html
Boundless. (n.d.). Boundless Biology. Retrieved August 03, 2020, from
https://courses.lumenlearning.com/boundless-biology/chapter/water/
Mott, V. (n.d.). Introduction to Chemistry. Retrieved August 03, 2020, from
https://courses.lumenlearning.com/introchem/chapter/the-structure-and-properties-
of-water/
(n.d.). Retrieved August 03, 2020, from http://www.sci.sdsu.edu/
Wilkin, D., & Brainard, J. (2020, February 07). Structure and Properties of
Water. Retrieved August 03, 2020, from https://www.ck12.org/biology/structure-
and-properties-of-water/lesson/Biochemical-Properties-of-Water-Advanced-
BIO-ADV/

64
NOTE: Practice personal hygiene protocols at all times
ANSWER KEY
Activity 1
Part A.
1. Capillary Action
2. Adhesion
3. Surface Tension
4. pH
5. Boiling point

Part B.
1. What is cohesion?
➢ Cohesion is the attraction between like molecules.
2. Why do some water insects able to walk on water? Explain.
➢ Insects use surface tension to walk on water, because of the strong
bond that creates a thin, invisible film on the surface of the water.
3. Define adhesion.
➢ It is the attraction between unlike molecules.
4. Explain how adhesion and cohesion help plants move materials.
➢ Helps contribute to the transport of water and dissolved nutrients
against gravity in plants.
5. When is water the densest?
➢ Water is densest at 4 degrees Celsius.
6. Explain why ice floats.
➢ Ice floats because it is about 9% less dense than liquid water. As it
cools further and becomes ice, it becomes less dense making it float
on the surface of water.
7. How is a lake or river that freezes over helpful to the organisms in the water?
➢ A lake that freeze over is helpful to the organisms inside the water
because, the solid ice serves as a barrier between the water and the
cold air above the water. The ice prevents the chilled air surrounding
the water body from getting to the organisms inside the water.

65
NOTE: Practice personal hygiene protocols at all times
8. How does water get to the leaves in the top of the tallest trees against the
force of gravity? Name property responsible for this and explain how it works.
➢ It is due to cohesion and adhesion through capillary action, which
helps plants transport materials needed for them to survive.
9. Why is solid water less dense than liquid water?
➢ Ice is less dense than water because the hydrogen bonds’ orientation
causes the molecules to push farther apart from each other, making its
density lower than of water.

Activity 2: Exploring Properties

a. Water molecules are cohesive, they form hydrogen bonds with each other.
b. Water molecules are adhesive, they form hydrogen bonds with polar surfaces.
c. Water is a liquid at normal physiological temperature.
d. Water has high specific heat.
e. Water has a high heat of vaporization.
f. Water shows high surface tension.

__D__1. During the winter, air temperature in the northern parts of the planet
can remain below 0 degrees Celsius for months, however, the fish and other animals
living in the lakes survive.

___B_2. Many substances, for example, salt (NaCl) and sucrose, dissolve
quickly in water.
A & B_3. When you pour water into a 25mL cylinder, a meniscus forms at the top
of the water column.
E & C 4. Sweating and the evaporation of sweat from the body surface help
reduce a human’s body temperature.
__A___5. Water drops that fall on a surface tend to form rounded drops or beads.
___B__6. If you put the end of a paper towel to a coloured water, the water will
move up into the towel.
__A___7. A paper clip can float on water.

66
NOTE: Practice personal hygiene protocols at all times
___F__8. When you place a straw into a glass of water, the water seems to climb up
the straw before you even place your mouth on the straw.
___C__9. Water is most dense at about 4°C. As a result, the water at the bottom of a
lake or the ocean usually has temperature of about 4°C.
__B__10. If you drop a tiny amount of water onto a very smooth surface, the water
molecules will stick together and form a droplet, rather than spread out over the
surface.

Activity 3. Word Hunt

Part A.
1. The hydrogen and oxygen atoms are held together by covalent bonds.
2. The electrons are not shared equally creating a polar molecule.
3. The polarity of water allows it to dissolve most substances.
4. Water molecules stick to other water molecules. This property is called
cohesion.
5. Hydrogen bonds form between adjacent molecules because the positively
charged hydrogen end of one water molecule attracts the negatively charged oxygen
end of another molecule.

Part B.
1. Warming up oil and water. Which temperature will rise faster?
➢ Oil. Water has some of the highest heat capacity of any known substances. Its
strong hydrogen bonds cause it to resist change in temperature. Because of
its strong hydrogen bond, changing the temperature of water is not easy.it
requires a lot of heat to make a small change in temperature.
2. What items can you gently “float” on water surface? (e.g., paperclips, needles,
etc.)
➢ Answers may vary.
3. What happens to the bonds (hydrogen bonds) when water boils?
➢ All the bonds break when water boils.

67
NOTE: Practice personal hygiene protocols at all times
Prepared by:

FLORIE MAE UNCIANO


Lallo National High School

68
NOTE: Practice personal hygiene protocols at all times
GENERAL CHEMISTRY 2

Name: ____________________________ Grade Level: _________


Date: _____________________________ Score: ______________

LEARNING ACTIVITY SHEET


THE STRUCTURE OF CRYSTLLINE AND AMORPHOUS SOLIDS
Background Information for the Learners (BIL)
A solid interface is defined as a small number of atomic layers that separate
two solids in intimate contact with one another, where the properties differ significantly
from those of the bulk material it separates.
Based on their crystal structures, solids can be classified into the following categories:
1. Crystalline solids
2. Amorphous solids
However, crystalline solids can be further classified into molecular, ionic, metallic, and
covalent solids.
An illustration detailing the classification of solids is provided below

.
Source:https://byjus.com/chemistry/classification-of-solids-based-on-crystal-structure/

69
NOTE: Practice personal hygiene protocols at all times
Crystalline are solids featuring highly ordered arrangements of their particles
(atoms, ions, and molecules) in microscopic structures.
These ordered microscopic structures make up a crystal lattice that accounts
for the structure of the solid at any given point. Examples of crystalline solids include
salt (sodium chloride), diamond, and sodium nitrate.
Ionic solids, such as sodium chloride and nickel
Figure 1. Sodium chloride is an
oxide, are composed of positive and negative ions ionic solid.
that are held together by electrostatic attractions,
which can be quite strong Figure 1. Many ionic
crystals also have high melting points. This is due to
the very strong attractions between the ions—in ionic
compounds, the attractions between full charges are
(much) larger than those between the partial charges
in polar molecular compounds. This will be looked at
in more detail in a later discussion of lattice energies.
Although they are hard, they also tend to be brittle, https://opentextbc.ca/chemistry/chapter/10-5-
the-solid-state-of-matter
and they shatter rather than bend. Ionic solids do not
conduct electricity; however, they do conduct when molten or dissolved because their
ions are free to move. Many simple compounds formed by the reaction of a metallic
element with a nonmetallic element are ionic.
Metallic solids such as crystals of copper,
aluminum, and iron are formed by metal atoms 2. Figure 2. Copper is a metallic solid.
The structure of metallic crystals is often described .
as a uniform distribution of atomic nuclei within a
“sea” of delocalized electrons. The atoms within
such a metallic solid are held together by a unique
force known as metallic bonding that gives rise to
many useful and varied bulk properties. All exhibit
high thermal and electrical conductivity, metallic
luster, and malleability. Many are very hard and quite
strong. Because of their malleability (the ability to
https://opentextbc.ca/chemistry/chapter/10-5-the-
deform under pressure or hammering), they do not solid-state-of-matter

shatter and, therefore, make useful construction materials. The melting points of the
metals vary widely. Mercury is a liquid at room temperature, and the alkali metals melt

70
NOTE: Practice personal hygiene protocols at all times
below 200 °C. Several post-transition metals also have low melting points, whereas
the transition metals melt at temperatures above 1000 °C. These differences reflect
differences in strengths of metallic bonding among the metals.
Covalent network solids include crystals of
Figure 3.
diamond, silicon, some other nonmetals, and some
covalent compounds such as silicon dioxide (sand)
and silicon carbide (carborundum, the abrasive on
sandpaper). Many minerals have networks of covalent
bonds. The atoms in these solids are held together by
a network of covalent bonds, as shown in Figure 3. To
break or to melt a covalent network solid, covalent
bonds must be broken. Because covalent bonds are
relatively strong, covalent network solids are typically
characterized by hardness, strength, and high melting
points. For example, diamond is one of the hardest https://opentextbc.ca/chemistry/chapter/10-5-the-solid-
state-of-matter
substances known and melts above 3500 °C.
A covalent crystal contains a three-dimensional network of covalent bonds, as
illustrated by the structures of diamond, silicon dioxide, silicon carbide, and graphite.
Graphite is an exceptional example, composed of planar sheets of covalent crystals
that are held together in layers by noncovalent forces. Unlike typical covalent solids,
graphite is very soft and electrically conductive.

71
NOTE: Practice personal hygiene protocols at all times
Molecular solids, such as ice, sucrose (table sugar),
Figure 4. Carbon dioxide.
and iodine, as shown in Figure 4, are composed of
neutral molecules. The strengths of the attractive
forces between the units present in different crystals
vary widely, as indicated by the melting points of the
crystals. Small symmetrical molecules (nonpolar
molecules), such as H2, N2, O2, and F2, have weak
attractive forces and form molecular solids with very
low melting points (below −200 °C). Substances
consisting of larger, nonpolar molecules have larger
attractive forces and melt at higher temperatures.
Molecular solids composed of molecules with
permanent dipole moments (polar molecules) melt at https://opentextbc.ca/chemistry/chapter/10-5-the-solid-
state-of-matter
still higher temperatures. Examples include ice (melting point, 0 °C) and table sugar
(melting point, 185 °C).
Figure 4. Carbon dioxide (CO2) consists of small, nonpolar molecules and forms a
molecular solid with a melting point of −78 °C. Iodine (I2) consists of larger, nonpolar
molecules and forms a molecular solid that melts at 114 °C.

Type of Solid Type of Types of Properties Examples


Particles Attractions
hard, brittle,
conducts
electricity as a
ionic Ions ionic bonds liquid but not NaCl, Al2O3
as a solid, high
to very high
melting points
shiny,
atoms of malleable,
Cu, Fe, Ti, Pb,
metallic electropositive metallic bonds ductile,
U
elements conducts heat
and electricity

72
NOTE: Practice personal hygiene protocols at all times
well, variable
hardness and
melting
temperature
covalent atoms of covalent very hard, not C (diamond),
network electronegative bonds conductive, SiO2, SiC
elements very high
melting points
molecular molecules (or MFs variable H2O, CO2, I2,
atoms) hardness, C12H22O11
variable
brittleness, not
conductive,
low melting
points
https://opentextbc.ca/chemistry/chapter/10-5-the-solid-state-of-matter

Amorphous are solids in which the particles are not arranged in any specific
order or the solids that lack the overall order of a crystal lattice.
The term ‘amorphous’, when broken down into its Greek roots, can be roughly
translated to “without form”. Many polymers are amorphous solids. Other examples
of such solids include glass, gels, and nanostructured materials.
An ideal crystal is defined as an atomic arrangement that has infinite
translational symmetry in all the three dimensions, whereas such a definite definition
is not possible for an ideal amorphous solid (a-solid).
Features of Crystalline and Amorphous Solids
CRYSTALLINE AMORPHOUS

NATURE True Solids. Pseudo – Solids or super-


cooled liquids.
Geometry Particles are arranged in a Particles are arranged
repeating pattern. They randomly. They do not
have a regular and have an ordered

73
NOTE: Practice personal hygiene protocols at all times
ordered arrangement arrangement resulting in
result irregular shapes.
ing in a definite shape.
Melting They have a sharp melting • They do not have
point. sharp melting
points. The solid
tends to soften
gradually over a
temperature range.
Heat of Fusion (The They have definite heat of • They do not have
change in enthalpy when fusion. definite heat of
a substance is heated to fusion.
change its state from
solid to liquid.)
Isotropism Anisotropic in nature. i.e., • Isotropic in nature.
the magnitude of physical i.e., the magnitude
properties (such as of the physical
refractive index, electrical properties is the
conductivity, thermal same along with all
conductivity etc) is directions of the
different along with solid.
different directions of the
crystal.
Cleavage When cutting with a sharp • When cutting with a
edge, the two new halves sharp edge, the two
will have smooth surfaces. resulting halves will
have irregular
surfaces.
Rigidity They are rigid solids and They are not rigid, so mild
applying mild forces will effects may change the
not distort its shape. shape.
https://byjus.com/chemistry/classification-of-solids-based-on-crystal-structure

74
NOTE: Practice personal hygiene protocols at all times
Learning Competency:
Describe the difference in structure of crystalline and amorphous solids.
(STEM_GC11IMF-IIIa-c-104)

Activity 1: CRYSTALLINE SOLID


Objective: Identify the type of crystalline solid formed by a substance.
Materials: Paper and pen
Direction: Identify the type of crystalline solid (metallic, network covalent, ionic, or
molecular) formed by each of the following substances.
__________1. CaCl2 __________6. CH3CH2CH2CH3
__________2. SiC __________7. HCl
__________3. N2 __________8. NH4NO3
__________4. Fe __________9. K3PO4
__________5. C (graphite) __________10. SiO2

Q1. Explain why ice, which is a crystalline solid, has a melting temperature of 0 °C,
whereas butter, which is an amorphous solid, softens over a range of temperatures.
___________________________________________________________________
___________________________________________________________________
___________________________________________________________________
_________________________________

75
NOTE: Practice personal hygiene protocols at all times
Activity 2: CONCEPTUAL PROBLEMS
Objective: Determine the difference in the structure of crystalline and amorphous
solids.
Materials: Paper and pen
Directions: Read and answer the questions briefly but substantially. Write your
answer on the space provided.
a. Why is the arrangement of the constituent atoms or molecules more important
in determining the properties of a solid than a liquid or a gas?
______________________________________________________________
______________________________________________________________
______________________________________________________________
______________________________________________________________
______________________________________________________________
__________________________________________________

b. A student obtained a solid product in a laboratory synthesis. To verify the


identity of the solid, she measured its melting point and found that the material
melted over a 12°C range. After it had cooled, she measured the melting point
of the same sample again and found that this time the solid had a sharp melting
point at the temperature that is characteristic of the desired product. Why were
the two melting points different? What was responsible for the change in the
melting point?
______________________________________________________________
______________________________________________________________
______________________________________________________________
______________________________________________________________
______________________________________________________________
__________________________________________________

76
NOTE: Practice personal hygiene protocols at all times
Activity 3: CRYSTAL SYSTEMS

Objectives:
• describe the main points of difference between a crystalline solid and an
amorphous solid;
• recognize and identify at least 3 of the 7 crystal systems;

Introduction/background
Traditional ceramics are clay-based. Clays have a mineral composition and
minerals have a crystalline structure. A mineral is defined as a naturally occurring
inorganic substance with a certain chemical composition and set of physical
properties. Many minerals occur in characteristic crystal shapes.

A crystalline solid is made up of an orderly repeating pattern of constituent


atoms, molecules or ions extending in all 3 spatial dimensions.

A limited number of crystal shapes have been found in nature. There are only
7 groups, or crystal systems, into which all naturally occurring crystals can be
placed. Careful observation of crystal shapes is one of the best ways to classify and
distinguish between different minerals. This activity focuses on three of these crystal
systems – cubic, triclinic and rhombohedral.

What you need


• Crystal systems diagram
• Copies of the student worksheet
• Small dropper bottles of 1 molL-1 solutions of sodium chloride (NaCl) and copper
sulfate (CuSO4)
• Clean ‘golden’ beach sand
• Simple light microscope plus microscope slides
• Electric hot plate
• Templates to construct models of cubic, triclinic and rhombohedral crystal
systems
• Paper glue

77
NOTE: Practice personal hygiene protocols at all times
What to do

1. Hand out copies of the crystal systems diagram and discuss with the class.
Explain that they will be investigating 3 of these crystal systems – cubic, triclinic
and rhombohedral.

2. Make sure each student has the necessary materials and equipment and a copy
of the student worksheet and templates.

Student worksheet – Studying crystal systems

Note: Please refer to the figure below for the reference of cubic, triclinic and
rhombohedral crystals.

1. Cubic crystals:
• Place a drop of the sodium
chloride solution supplied in
the center of a microscope
slide.
• Gently heat the slide by
placing it on a hot plate (low
setting).
• When all the water has
evaporated, view the sodium
chloride crystals that remain
under the low power of a
microscope.
• Note the shape of the
crystals and sketch what
you see.

78
NOTE: Practice personal hygiene protocols at all times
2. Triclinic crystals:
• Place a drop of the copper
sulfate solution supplied in
the center of a microscope
slide.
• Gently heat the slide by
placing it on a hot plate (low
setting).
• When all the water has
evaporated, view the copper
sulfate crystals that remain
under the low power of a
microscope.
• Note the shape of the
crystals and sketch what
you see.

3. Rhombohedral crystals:
• Place a small sample of
beach sand in the center of
a microscope slide and
spread out the grains.
• View under the low power of
a microscope.
• Note the shape of the grains
with a clear or whitish
appearance – these are
grains of the mineral quartz.
Sketch what you see.

4. Compare the sketches you have drawn to the crystal systems diagram.

79
NOTE: Practice personal hygiene protocols at all times
5. The mineral halite, a naturally occurring form of sodium chloride, has a cubic
crystal structure. Use the cubic crystal template to construct a model of a halite
crystal. Fold all edges. Glue the tabs and stick together.

6. The feldspar minerals plagioclase and orthoclase have a triclinic crystal structure.
Copper sulfate crystallizes out of solution as triclinic crystals just like the
feldspars. Use the triclinic crystal template to construct a model of a feldspar
mineral crystal. Fold all edges. Glue the tabs and stick together.

7. Quartz minerals are commonly found in beach sand. These tiny grains have a
rhombohedral shape (cubic system stretched along a body diagonal). Use the
rhombohedral crystal template to construct a model of a quartz crystal.

80
NOTE: Practice personal hygiene protocols at all times
Crystal systems

81
NOTE: Practice personal hygiene protocols at all times
Activity 4: BUILD ME UP
Objectives:
• use models to point out the angular and side length differences that
characterize the cubic, triclinic and rhombohedral crystal systems.

Materials: template of cubic, triclinic and rhombohedral crystal system


Directions: Use the given template to point out the angular and side length differences
that characterize the cubic, triclinic and rhombohedral crystal systems

Cubic crystal template

All axes are of equal length. All axes are at 90° to one another.

All axes are of variable lengths. All axes are at variable angles.

82
NOTE: Practice personal hygiene protocols at all times
Rhombohedral crystal template

All the axes are equal. All axes are at angles other than 90°.

Activity 5: CRYSTALLINE AND AMORPHOUS SOLID (Pre-lab)


Objective: Distinguish between crystalline and amorphous substances.
Materials: Worksheet Students create
patterns using
Altair
designs.
Background: The atoms in crystalline solid matter
are arranged in regular, repeating .patterns. All other
types of solid matter are amorphous or without a
regular atomic arrangement. Metals and minerals are
crystalline. Glass is amorphous. Depending upon its
composition, the crystalline pattern of a mineral may
not be visible in a hand sample. In this case minerals
are studied using X-ray diffraction, a technique that
uses the reflection of X-rays to determine crystal
Electron level picture of tin
structure and composition.

83
NOTE: Practice personal hygiene protocols at all times
Procedure:
1. Observe the following diagram below illustrating crystalline versus a non-
crystalline (amorphous) patterns.

Crystalline Amorphous
2. On the worksheet, outline or fill in spaces on the Altair designs sheet to
create patterns. Your patterns are examples of order within the overall
structure of the design. This same type of organization generates crystalline
structures in minerals. The Altair designs sheet will naturally guide your
imagination through the maze of lines. Since no two students are alike, none
of you will see the same shapes, forms or patterns hidden in these designs.
You may create some very interesting artwork.
3. After finishing your patterns pair up with your seatmate and see if there are
any similar patterns. The similarities and differences means that there are
many types of minerals, and hence many different crystal patterns.

84
NOTE: Practice personal hygiene protocols at all times
Activity 6: MULTIPLE CHOICE
Directions: Read each item carefully. Write the letter that corresponds to the correct
answer on the space provided.

_____1. In amorphous solid, the atoms or molecules are held together in a completely
random formation.
A. True B. False
_____2. Which of the following is true of solids?
A. Solids maintain a defined shape and size under all conditions.
B. All solids maintain a defined shape and size if conditions remain constant.
C. All solids have a lattice structure at atomic level.
D. All solids have a crystalline structure.
_____3. One major difference between crystalline and amorphous solids is that
A. Crystalline solids have precise melting point.
B. Amorphous solids have a lattice structure.
C. Crystalline solids break unpredictably and can produce curved fragments.
D. Amorphous solids always behave consistently and uniformly.
_____4. A friend in your chemistry class is struggling to understand why crystalline
solids are grouped into four main types: network, molecular, ionic, and metallic.
Which explanation below will best help him begin to understand why chemists
might have these groups?
A. Crystalline solids all share a lattice structure, but have different densities.
Chemists use the groups to organize the solids by density.
B. Crystalline solids all share a lattice structure and the same types of bonds,
but are composed of different elements. These elements affect the way the
solid conducts heat and electricity.
C. Crystalline solds all share a lattice structure, but behave differently under
similar conditions.
D. Crystalline solids all share a lattice structure, but the bonds that hold them
together at the atomic level differ. The elements that make up the solids also
differ. These differences affect how a solid conducts heat and electricity,
and its density.

85
NOTE: Practice personal hygiene protocols at all times
_____5. Solids have many different properties. _____ solids are known for their ability
to be flattened into a sheet, stretched into a wire, and to conduct energy well.
A. Molecular B. Metallic C. Network D. Ionic
_____6. It is possible to tell the difference between a solid with a crystalline structure
and one with an amorphous structure just by looking at it.
A. True B. False
_____7. An engineer is designing an electrical system and is looking for a material to
transmit energy. She has four solids available, each made with different
materials. To conduct energy most efficiently and effectively, she should use
material
A. Whose electrons are held with ionic bonds.
B. Whose electrons are held with covalent bonds.
C. Whose electrons are held with metallic bonds.
D. That is an electrical insulator.
_____8. Which statement is true about the properties of solids?
A. Metallic solids have a high melting point.
B. Network solids are generally not soluble in water.
C. Molecular solids do not dissolve easily in water.
D. All ionic solids are similar in density.

86
NOTE: Practice personal hygiene protocols at all times
Reflection

1. I learned that _______________________________________________________


___________________________________________________________________
_________________________________________________________

2. I enjoyed most on ___________________________________________________


___________________________________________________________________
___________________________________________________________________
__.

3. I want to learn more on _______________________________________________


___________________________________________________________________
___________________________________________________________________.

87
NOTE: Practice personal hygiene protocols at all times
References:
https://opentextbc.ca/chemistry/chapter/10-5-the-solid-state-of-matter
https://byjus.com/chemistry/classification-of-solids-based-on-crystal-structure/
https://www.sciencelearn.org.nz/resources/1784-crystal-systems

88
NOTE: Practice personal hygiene protocols at all times
ANSWER KEY
Activity 1: CRYSTALLINE SOLID

Ionic 1. CaCl2 molecular 6. CH3CH2CH2CH3


Covalent 2. SiC molecular 7. HCl
Molecular 3. N2 ionic 8. NH4NO3
Metallic 4. Fe ionic 9. K3PO4
Covalent 5. C (graphite) covalent 10. SiO2

Q1. Explain why ice, which is a crystalline solid, has a melting temperature of 0 °C,
whereas butter, which is an amorphous solid, softens over a range of temperatures.

Ice has a crystalline structure stabilized by hydrogen bonding. These


intermolecular forces are of comparable strength and thus require the same
amount of energy to overcome. As a result, ice melts at a single temperature
and not over a range of temperatures. The various, very large molecules that
compose butter experience varied van der Waals attractions of various
strengths that are overcome at various temperatures, and so the melting
process occurs over a wide temperature range

Activity 2: CONCEPTUAL PROBLEMS

a. Why is the arrangement of the constituent atoms or molecules more important


in determining the properties of a solid than a liquid or a gas?
The arrangement of the atoms or molecules is more important in
determining the properties of a solid because of the greater persistent long-
range order of solids. Gases and liquids cannot readily be described by the
spatial arrangement of their components because rapid molecular motion
and rearrangement defines many of the properties of liquids and gases.

b. A student obtained a solid product in a laboratory synthesis. To verify the


identity of the solid, she measured its melting point and found that the material
melted over a 12°C range. After it had cooled, she measured the melting point
of the same sample again and found that this time the solid had a sharp melting

89
NOTE: Practice personal hygiene protocols at all times
point at the temperature that is characteristic of the desired product. Why were
the two melting points different? What was responsible for the change in the
melting point?

The initial solid contained the desired compound in an amorphous state, as


indicated by the wide temperature range over which melting occurred. Slow
cooling of the liquid caused it to crystallize, as evidenced by the sharp second
melting point observed at the expected temperature.

Activity 3: CRYSTAL SYSTEMS

Objectives:
• describe the main points of difference between a crystalline solid and an
amorphous solid;
• recognize and identify at least 3 of the 7 crystal systems;
• use models to point out the angular and side length differences that
characterize the cubic, triclinic and rhombohedral crystal systems ;
• effectively use optical aids such as hand lenses and simple microscopes to
view crystalline solids.

Materials:
• Crystal systems diagram
• Copies of the student worksheet
• Small dropper bottles of 1 molL-1 solutions of sodium chloride (NaCl) and copper
sulfate (CuSO4)
• Clean ‘golden’ beach sand
• Simple light microscope plus microscope slides
• Electric hot plate
• Templates to construct models of cubic, triclinic and rhombohedral crystal
systems
• Paper glue

90
NOTE: Practice personal hygiene protocols at all times
Note: The answer of the students vary since the activity requires personal answer
based from their observations.

Activity 4: BUILD ME UP
Note: Shapes are represented in Crystal System

91
NOTE: Practice personal hygiene protocols at all times
Activity 5: CRYSTALLINE AND AMORPHOUS SOLID (Pre-lab)

Note: The answer of the students vary since the activity requires personal answer
based from their observations.

Activity 6: MULTIPLE CHOICE


Objective: Distinguish the difference between crystalline and amorphous solid.
Materials: Paper and pen
Direction: Read each item carefully. Write the letter that corresponds to the correct
answer on the space provided.

1. B
2. B
3. A
4. D
5. B
6. A
7. C
8. B

Prepared by:

DIVINA S. RIBIACO
Baua National High School

92
NOTE: Practice personal hygiene protocols at all times
GENERAL CHEMISTRY 2

Name: ____________________________ Grade Level: _________


Date: _____________________________ Score: ______________

LEARNING ACTIVITY SHEET


PHASE DIAGRAM OF WATER AND CARBON DIOXIDE

Background Information for the Learners

A typical phase diagram consists of discrete regions that represent the different
phases exhibited by a substance (Figure 12.4.1). Each region corresponds to the
range of combinations of temperature and pressure over which that phase is stable.
The combination of high pressure and low temperature (upper left of Figure 12.4.1)
corresponds to the solid phase, whereas the gas phase is favored at high temperature
and low pressure (lower right). The combination of high temperature and high pressure
(upper right) corresponds to a supercritical fluid.

https://chem.libretexts.org/Bookshelves/General_Chemistry/Map%3A_General_Chemistry_(Petrucci_et_al.)/12%3A_Intermolec
ular_Forces%3A_Liquids_And_Solids/12.4%3A_Phase_Diagrams

Figure 12.4.1: A Typical Phase Diagram for a Substance That Exhibits Three
Phases—Solid, Liquid, and Gas—and a Supercritical Region

The solid phase is favored at low temperature and high pressure; the gas phase
is favored at high temperature and low pressure.

93
NOTE: Practice personal hygiene protocols at all times
The lines in a phase diagram correspond to the combinations of temperature
and pressure at which two phases can coexist in equilibrium. In Figure 12.4.1, the line
that connects points A and D separates the solid and liquid phases and shows how
the melting point of a solid varies with pressure. The solid and liquid phases are in
equilibrium all along this line; crossing the line horizontally corresponds to melting or
freezing. The line that connects points A and B is the vapor pressure curve of the
liquid, which we discussed in Section 11.5. It ends at the critical point, beyond which
the substance exists as a supercritical fluid. The line that connects points A and C is
the vapor pressure curve of the solid phase. Along this line, the solid is in equilibrium
with the vapor phase through sublimation and deposition. Finally, point A, where the
solid/liquid, liquid/gas, and solid/gas lines intersect, is the triple point; it is
the only combination of temperature and pressure at which all three phases (solid,
liquid, and gas) are in equilibrium and can therefore exist simultaneously. Because no
more than three phases can ever coexist, a phase diagram can never have more than
three lines intersecting at a single point.

Remember that a phase diagram, such as the one in Figure 12.4.1, is for a
single pure substance in a closed system, not for a liquid in an open beaker in contact
with air at 1 atm pressure. In practice, however, the conclusions reached about the
behavior of a substance in a closed system can usually be extrapolated to an open
system without a great deal of error.

The Phase Diagram of Water

Figure 12.4.2 shows the phase diagram of water and illustrates that the triple
point of water occurs at 0.01°C and 0.00604 atm (4.59 mmHg). Far more reproducible
than the melting point of ice, which depends on the amount of dissolved air and the
atmospheric pressure, the triple point (273.16 K) is used to define the absolute (Kelvin)
temperature scale. The triple point also represents the lowest pressure at which a
liquid phase can exist in equilibrium with the solid or vapor. At pressures less than
0.00604 atm, therefore, ice does not melt to a liquid as the temperature increases; the
solid sublimes directly to water vapor. Sublimation of water at low temperature and
pressure can be used to “freeze-dry” foods and beverages. The food or beverage is
first cooled to subzero temperatures and placed in a container in which the pressure
is maintained below 0.00604 atm. Then, as the temperature is increased, the water

94
NOTE: Practice personal hygiene protocols at all times
sublimes, leaving the dehydrated food (such as that used by backpackers or
astronauts) or the powdered beverage (as with freeze-dried coffee).

https://chem.libretexts.org/Bookshelves/General_Chemistry/Map%3A_General_Chemistry_(Petrucci_et_al.)/12%3A_Intermolec
ular_Forces%3A_Liquids_And_Solids/12.4%3A_Phase_Diagrams

Figure 12.4.2: Two Versions of the Phase Diagram of Water. (a) In this graph
with linear temperature and pressure axes, the boundary between ice and liquid
water is almost vertical. (b) This graph with an expanded scale illustrates the
decrease in melting point with increasing pressure. (The letters refer to points
discussed in Example 12.4.1).

The phase diagram for water illustrated in Figure 12.4.2b shows the boundary
between ice and water on an expanded scale. The melting curve of ice slopes up and
slightly to the left rather than up and to the right as in Figure 12.4.1; that is, the melting
point of ice decreases with increasing pressure; at 100 MPa (987 atm), ice melts at
−9°C. Water behaves this way because it is one of the few known substances for
which the crystalline solid is less dense than the liquid (others include antimony and
bismuth). Increasing the pressure of ice that is in equilibrium with water at 0°C and 1
atm tends to push some of the molecules closer together, thus decreasing the volume
of the sample. The decrease in volume (and corresponding increase in density) is
smaller for a solid or a liquid than for a gas, but it is sufficient to melt some of the ice.

In Figure 12.4.2b point A is located at P = 1 atm and T = −1.0°C, within the solid
(ice) region of the phase diagram. As the pressure increases to 150 atm while the
temperature remains the same, the line from point A crosses the ice/water boundary
to point B, which lies in the liquid water region. Consequently, applying a pressure of
150 atm will melt ice at −1.0°C. We have already indicated that the pressure
dependence of the melting point of water is of vital importance. If the solid/liquid

95
NOTE: Practice personal hygiene protocols at all times
boundary in the phase diagram of water were to slant up and to the right rather than
to the left, ice would be denser than water, ice cubes would sink, water pipes would
not burst when they freeze, and antifreeze would be unnecessary in automobile
engines.

Referring to the phase diagram of water in Figure 12.4.2:

a. Predict the physical form of a sample of water at 400°C and 150 atm.
b. Describe a changes that occur as the sample in part (a) is slowly allowed
to cool to -50°C at a constant pressure of 150 atm
Given: phase diagram, temperature, and pressure
Asked for: physical form and physical changes
Strategy:
• Identify the region of the phase diagram corresponding to the initial conditions
and identify the phase exist in this region.
• Draw a line corresponding to the given pressure. Move along that line in the
appropriate direction (in this case cooling) and describe the phase changes.
Solution:
a. Locate the starting point on the phase diagram in part (a) in
Figure 12.4.212.4.2. The initial conditions correspond to point A, which lies in
the region of the phase diagram representing water vapor. Thus water at T =
400°C and P = 150 atm is a gas.
b. Cooling the sample at constant pressure corresponds to moving left along the
horizontal line in part (a) in Figure 12.4.212.4.2. At about 340°C (point B), we
cross the vapor pressure curve, at which point water vapor will begin to
condense and the sample will consist of a mixture of vapor and liquid. When all
of the vapor has condensed, the temperature drops further, and we enter the
region corresponding to liquid water (indicated by point C). Further cooling
brings us to the melting curve, the line that separates the liquid and solid phases
at a little below 0°C (point D), at which point the sample will consist of a mixture
of liquid and solid water (ice). When all of the water has frozen, cooling the
sample to −50°C takes us along the horizontal line to point E, which lies within
the region corresponding to solid water. At P = 150 atm and T = −50°C,
therefore, the sample is solid ice.

96
NOTE: Practice personal hygiene protocols at all times
The Phase Diagram of Carbon dioxide

In contrast to the phase diagram of water, the phase diagram of


CO2 (Figure 12.4.312.4.3) has a more typical melting curve, sloping up and to the
right. The triple point is −56.6°C and 5.11 atm, which means that liquid CO 2 cannot
exist at pressures lower than 5.11 atm. At 1 atm, therefore, solid CO2 sublimes directly
to the vapor while maintaining a temperature of −78.5°C, the normal sublimation
temperature. Solid CO2 is generally known as dry ice because it is a cold solid with no
liquid phase observed when it is warmed.

Dry ice (CO2(s)CO2(s)) sublimed in air under room


temperature and pressure.

Also notice the critical point at 30.98°C and 72.79 atm. Supercritical carbon dioxide is
emerging as a natural refrigerant, making it a low carbon (and thus a more
environmentally friendly) solution for domestic heat pumps.

https://chem.libretexts.org/Bookshelves/General_Chemistry/Map%3A_General_Chemistry_(Petrucci_et_al.)/12%3A_Intermolec
ular_Forces%3A_Liquids_And_Solids/12.4%3A_Phase_Diagrams

Figure 12.4.3: The Phase Diagram of Carbon Dioxide. Note the critical point, the
triple point, and the normal sublimation temperature in this diagram.

97
NOTE: Practice personal hygiene protocols at all times
The Critical Point

As the phase diagrams above demonstrate, a combination of high pressure and


low temperature allows gases to be liquefied. As we increase the temperature of a
gas, liquefaction becomes more and more difficult because higher and higher
pressures are required to overcome the increased kinetic energy of the molecules. In
fact, for every substance, there is some temperature above which the gas can no
longer be liquefied, regardless of pressure. This temperature is the critical temperature
(Tc), the highest temperature at which a substance can exist as a liquid. Above the
critical temperature, the molecules have too much kinetic energy for the intermolecular
attractive forces to hold them together in a separate liquid phase. Instead, the
substance forms a single phase that completely occupies the volume of the container.
Substances with strong intermolecular forces tend to form a liquid phase over a very
large temperature range and therefore have high critical temperatures. Conversely,
substances with weak intermolecular interactions have relatively low critical
temperatures. Each substance also has a critical pressure (Pc), the minimum pressure
needed to liquefy it at the critical temperature. The combination of critical temperature
and critical pressure is called the critical point. The critical temperatures and pressures
of several common substances are listed in Figure 12.4.1

Figure 12.4.1: Critical Temperatures and Pressures of Some Simple Substances


Substance Tc (°C) Pc (atm)
NH3 132.4 113.5
CO2 31.0 73.8
CH3CH2OH (ethanol) 240.9 61.4
He −267.96 2.27
Hg 1477 1587
CH4 −82.6 46.0
N2 −146.9 33.9
H2O 374.0 217.7

*High-boiling-point, nonvolatile liquids have high critical temperatures and vice versa.

98
NOTE: Practice personal hygiene protocols at all times
https://chem.libretexts.org/Bookshelves/General_Chemistry/Map%3A_General_Chemistry_(Petrucci_et_al.)/12%3A_Intermolec
ular_Forces%3A_Liquids_And_Solids/12.4%3A_Phase_Diagrams

Learning Competency:
Interpret the phase diagram of water and carbon dioxide (STEM_GC11IMFIIIa-c-
107)

Activity 1: WATER AND CARBON DIOXIDE


Objective: determine the state of water at each given temperature and pressure.
Materials: Paper and pen
Figure A. Figure B

courses.lumenlearning.com/wsu-sandbox2/chapter/phase-diagram-2/

A. Directions: Using the phase diagram (fig. a) for water, determine the state of H2O
at the following temperatures and pressures. Write your answer on the space
provided.

__________1. -10 °C and 50 kPa


__________2. 25°C and 90 kPa
__________3. 50°C and 40 kPa
__________4. 80°C and 5 kPa
__________5. -10°C and 0.3 kPa

99
NOTE: Practice personal hygiene protocols at all times
A. Directions: Using the phase diagram for carbon dioxide, determine the state of
CO2 at the following temperatures and pressures. Write your answer on the space
provided.

__________6. −30 °C and 2000 kPa

__________7. −60 °C and 1000 kPa

__________8. −60 °C and 100 kPa

__________9. 20 °C and 1500 kPa

__________10. 0 °C and 100 kPa

Activity 2: CRITICAL THINKING (H2O and CO2)


Objective: Interpret the phase diagram of water and carbon dioxide.
Materials: Paper and pen
Directions: Read and analyze the given problem, then answer the questions below.
Write your answer on the space provided.
Problem: Imagine a substance with the following points on the phase diagram: a triple
point at .5 atm and -5ºC; a normal melting point at 20ºC; normal boiling point at 150ºC;
and a critical point at 5 atm and 1000ºC. The solid liquid line is “normal” (meaning
positive sloping). For this, complete the following:

1. Describe what one would see at pressures and temperatures above 5 atm and
1000ºC.
2. Describe what will happen to the substance when it begins in a vacuum at -15 ºC
and is slowly pressurized.
3. Describe the phase changes from -80ºC to 500ºC at 2 atm.

100
NOTE: Practice personal hygiene protocols at all times
Activity 3: THE COOL CHEMISTRY OF DRY ICE
Objective: Interpret the phase diagram of water.
Materials: Paper and pen
Directions: Read and analyze the given problem, then answer the question below.
Write your answer on the space provided.
Problem: Referring to the phase diagram of water in figure 12.4.2, predict the
physical form of a sample of water at -0.0050ºC as the pressure is gradually
increased from 1.0 mmHg to 218 atm. Write your answer on the space provided.

https://chem.libretexts.org/Bookshelves/General_Chemistry/Map%3A_General_Chemistry_(Petrucci_et_al.)/12%3A_Intermolec
ular_Forces%3A_Liquids_And_Solids/12.4%3A_Phase_Diagrams

______________________________________________________________
___________________________________________________________________
___________________________________________________________________
___________________________________________________________________
____________________________________________________.

101
NOTE: Practice personal hygiene protocols at all times
Activity 4: DIHYDROGEN MONOXIDE
Objective: Interpret the phase diagram of water.
Materials: Paper and pen

A phase diagram of water is shown below

B 4
C (374 ° C, 218 atm)

1 2
3
A (0.01 ° C, 0.00603 atm)

Temperature

https://scilearn.sydney.edu.anu...
1. Identify the four phases shown as 1-4 in the phase diagram.

a. c.
b. d.

2. What names are given to the points A and C?

a. b.

3. The boundary line A-B is slightly tilted to the left. What are the physical and
biological significances of this?

102
NOTE: Practice personal hygiene protocols at all times
Activity 5: CARBON DIOXIDE
Objective: determine the state of water at each given temperature and pressure.
Materials: Paper and pen
Directions: Answer the following questions based on the P-T phase diagram of
carbon dioxide.
Write your answer on the space provided.

Phase diagram of carbon dioxide

https://www.toppr.com/ask/question/answer-the-following-questions-based-on-the-pt-phase-diagram/

1. At what temperature and pressure can the solid, liquid and vapor phases of
CO2 co-exits in equilibrium?
______________________________________________________________
______________________________________________________________
________________________________.

2. What is the effect of decrease of pressure on the fusion and boiling point of
CO2?
______________________________________________________________
____________________________________________________________.

3. What are the critical temperature and pressure for CO2?


______________________________________________________________
______________________________________________________________
_____________________________________________________________.

103
NOTE: Practice personal hygiene protocols at all times
Reflection

1. I learned that _______________________________________________________


___________________________________________________________________
_________________________________________________________

2. I enjoyed most on ___________________________________________________


___________________________________________________________________
___________________________________________________________________
__.

3. I want to learn more on _______________________________________________


___________________________________________________________________
___________________________________________________________________.

104
NOTE: Practice personal hygiene protocols at all times
References:
https://chem.libretexts.org/Bookshelves/General_Chemistry/Map%3A_General_Che
mistry_(Petrucci_et_al.)/12%3A_Intermolecular_Forces%3A_Liquids_And_Solids/12
.4%3A_Phase_Diagrams
https://msnucleus.org/membership/html/k-6/rc/minerals/3/rcm3_4a.html
https://chem.libretexts.org/Bookshelves/Physical_and_Theoretical_Chemistry_Textb
ook_Maps/Supplemental_Modules_(Physical_and_Theoretical_Chemistry)/Physical
_Properties_of_Matter/States_of_Matter/Phase_Transitions/Phase_Diagrams
https://www.toppr.com/ask/question/answer-the-following-questions-based-on-the-pt-
phase-diagram/
courses.lumenlearning.com/wsu-sandbox2/chapter/phase-diagram-2/

105
NOTE: Practice personal hygiene protocols at all times
ANSWER KEY

Activity 1: WATER AND CARBON DIOXIDE


Objective: determine the state of water at each given temperature and pressure.
Materials: Paper and pen
Figure A. Figure B

A.Directions: Using the phase diagram (fig. a) for water, determine the state of H2O
at the following temperatures and pressures. Write your answer on the space
provided.

Solid 1. -10 °C and 50 kPa


Liquid 2. 25°C and 90 kPa
Liquid 3. 50°C and 40 kPa
Gas 4. 80°C and 5 kPa
Solid 5. -10°C and 0.3 kPa

106
NOTE: Practice personal hygiene protocols at all times
B. Directions: Using the phase diagram for carbon dioxide, determine the state of
CO2 at the following temperatures and pressures. Write your answer on the space
provided.

Liquid 6. −30 °C and 2000 kPa

Solid 7. −60 °C and 1000 kPa

Gas 8. −60 °C and 100 kPa

Liquid 9. 20 °C and 1500 kPa

Gas 10. 0 °C and 100 kPa

Activity 2: CRITICAL THINKING (H2O and CO2)


Objective: Interpret the phase diagram of water and carbon dioxide.
Materials: Paper and pen

Problem: Imagine a substance with the following points on the phase diagram: a triple
point at .5 atm and -5ºC; a normal melting point at 20ºC; normal boiling point at 150ºC;
and a critical point at 5 atm and 1000ºC. The solid liquid line is “normal” (meaning
positive sloping). For this, complete the following:

4. Describe what one would see at pressures and temperatures above 5 atm and
1000ºC.
• One would see a super-critical fluid, when approaching the point, one
would see the meniscus between the liquid and gas disappear.
5. Describe what will happen to the substance when it begins in a vacuum at -15 ºC
and is slowly pressurized.
• The substance would begin as a gas and as the pressure increases, it
would compress and eventually solidify without liquefying as the
temperature is below the triple point temperature.
6. Describe the phase changes from -80ºC to 500ºC at 2 atm.

107
NOTE: Practice personal hygiene protocols at all times
• The substance would melt at somewhere around, but above 20ºC and then
boil at somewhere around, but above 150ºC. It would not form a super-
critical fluid as the neither the pressure nor temperature reach the critical
pressure or temperature.

Activity 3: THE COOL CHEMISTRY OF DRY ICE


Objective: Interpret the phase diagram of water.
Materials: Paper and pen

Problem: Referring to the phase diagram of water in figure 12.4.2, predict the
physical form of a sample of water at -0.0050ºC as the pressure is gradually
increased from 1.0 mmHg to 218 atm. Write your answer on the space provided.

The sample is initially a gas, condenses to a solid as the pressure


increases, and then melts when the pressure is increased further to give a
liquid.

108
NOTE: Practice personal hygiene protocols at all times
Activity 4: DIHYDROGEN MONOXIDE
Objective: Interpret the phase diagram of water.
Materials: Paper and pen

A phase diagram of water is shown below

B 4
C (374 ° C, 218 atm)

1 2
3
A (0.01 ° C, 0.00603 atm)

Temperature

https://scilearn.sydney.edu.a...

i. Identify the four phases shown as 1-4 in the phase diagram.

a. Solid c. Gas
b. Liquid d. Super critical Fluid

ii. What names are given to the points A and C?

a. Triple point b. Critical point

iii. The boundary line A-B is slightly tilted to the left. What are the physical and
biological significances of this?

Solid water (ice) is less dense than liquid water. Ice therefore floats and
rivers, lakes and oceans freeze from the top town.

As just the surface of a body of water freezes, the liquid environment for the
life-forms below the surface is preserved. If the water in a cell freezes, the
ice crystals take up more space than the liquid and this can cause cell walls
to break.

109
NOTE: Practice personal hygiene protocols at all times
Activity 5: CARBON DIOXIDE
Objective: determine the state of water at each given temperature and pressure.
Materials: Paper and pen
Direction: Answer the following questions based on the P-T phase diagram of carbon
dioxide.
Write your answer on the space provided.

Phase diagram of carbon dioxide

1. At what temperature and pressure can the solid, liquid and vapor phases of
CO2 co-exits in equilibrium?
The three phases can coexist at triple point. From the graph, it is at -56.6ºC
and 5.11 atm.

2. What is the effect of decrease of pressure on the fusion and boiling point of
CO2?
With the decrease in pressure, both the fusion and boiling point of carbon
dioxide will decrease.

3. What are the critical temperature and pressure for CO 2?

For carbon dioxide, the critical temperature is 31.1ºC and critical pressure
is 73.0 atm

Prepared by:

DIVINA S. RUBIACO
Baua National High School

110
NOTE: Practice personal hygiene protocols at all times
GENERAL CHEMISTRY 2
Name: Grade Level:
Date: Score:

LEARNING ACTIVITY SHEET

HEATING AND COOLING CURVES


Background Information for the Learners (BIL)

Heating Curve

Imagine that you have a block of ice that is at a temperature of -30°C, well
below its melting point. The ice is in a closed container. As heat is steadily added to
the ice block, the water molecules will begin to vibrate faster and faster as they absorb
kinetic energy. Eventually, when the ice has warmed to 0°C, the added energy will
start to break apart the hydrogen bonding that keeps the water molecules in place
when it is in the solid form. As the ice melts, its temperature does not rise. All of the
energy that is being put into the ice goes into the melting process and not into any
increase in temperature. During the melting process, the two states – solid
and liquid – are in equilibrium with one another. If the system was isolated at that
point and no energy was allowed to enter or leave, the ice-water mixture at 0°C would
remain. Temperature is always constant during a change of state.

Continued heating of the water after the ice has completely melted will now
increase the kinetic energy of the liquid molecules and the temperature will rise.
Assuming that the atmospheric pressure is standard, the temperature will rise steadily
until it reaches 100°C. At this point, the added energy from the heat will cause the
liquid to begin to vaporize. As with the previous state change, the temperature will
remain at 100°C while the water molecules are going from the liquid to the gas or
vapor state. Once all the liquid has completely boiled away, continued heating of the
steam (remember the container is closed) will increase its temperature above 100°C.

111
NOTE: Practice personal hygiene protocols at all times
The experiment described above can be summarized in a graph called a
heating curve:

D E

B C

• Between A & B, the material is a solid. The heat supplied to the material is used
to increase the kinetic energy of the molecules and the temperature rises.
• Between B & C, the solid is melting. Heat is still being supplied to the material
but the temperature does not change. Heat energy is not being changed into
kinetic energy. Instead, the heat is used to change the arrangement of the
molecules.
• At point C, all of the material has been changed to liquid.
• Between C & D, the heat supplied is again used to increase kinetic energy of
the molecules and the temperature of the liquid starts to rise.
• Between C & D, the liquid is heated until it starts to boil.
• Between D & E, the liquid is still being heated but the extra heat energy does
not change the temperature (kinetic energy) of the molecules. The heat energy
is used to change the arrangement of the molecules to form a gas.
• At point E, all of the liquid has been changed into gas.
• Between E & F, the gas is heated and the heat energy increases the kinetic
energy of molecules once more, so the temperature of the gas increases.

112
NOTE: Practice personal hygiene protocols at all times
When a system contains only one phase (solid, liquid, or gas), the temperature
will increase when it receives energy. The rate of temperature increase will be
dependent on the heat capacity of the phase in the system. When the heat capacity is
large, the temperature increases slowly, because much energy is required to increase
its temperature by one degree. Thus, the slopes of temperature increase for the solid,
liquid, and gases are different.

In the heating curve of water, the temperature is shown as heat is continually


added. Changes of state occur during plateaus because the temperature is constant.

The change of state behavior of all substances can be represented with a


heating curve of this type. The melting and boiling points of the substance can be
determined by the horizontal lines or plateaus on the curve. Other substances would
of course have melting and boiling points that are different from those of water. One
exception to this exact form for a heating would be for a substance such as carbon
dioxide which sublimes rather than melts at standard pressure. The heating curve for
carbon dioxide would have only one plateau, at the sublimation temperature of CO2.

Cooling Curves

Heating curves show how the temperature changes as a substance is heated


up. Cooling curves are the opposite. They show how the temperature changes as a
substance is cooled down. Just like heating curves, cooling curves have horizontal flat
parts where the state changes from gas to liquid, or from liquid to solid. These are
mirror images of the heating curve.

You will use lauric acid in a school lab to make your own cooling curve. Lauric
acid has a melting point of about 45°C and is easily melted in a test tube placed in a
beaker of hot water. The temperature can be followed using a thermometer or
temperature probe connected to a data logger. The liquid may be cooled by putting
the boiling tube in a beaker of cold water or just leaving it in the air.

113
NOTE: Practice personal hygiene protocols at all times
Note: The melting and freezing occur at the same temperature. During freezing,
energy is removed and during melting, energy is absorbed.

Energy Changes

Since Temperature is a measure of "Average Kinetic Energy", any change in


temperature is a change in Kinetic Energy. All of the diagonal line segments on a
heating or cooling curve show a temperature change and therefore a change in kinetic
energy.

During these regions, a single state of matter exists and the sample is either
getting hotter or cooler. During the horizontal line segments, there is no change in
temperature, so kinetic energy remains constant. However, all the energy that is
absorbed or released is related to changes in potential energy.

Remember the 3 Ps: Plateau, Phase change and Potential Energy Change.

Source:
https://www.rcboe.org/cms/lib/GA01903614/Centricity/Domain/1951/Heating%20and
%20Cooling%20Curves%20new.pdf

114
NOTE: Practice personal hygiene protocols at all times
Learning Competency

Determine and explain the heating and cooling curve of a substance


(STEM_GC11IMFIIIa-c-109)

Activity 1: THE COOLING CURVE OF WATER

Directions: Using the curve below describe what is happening between each of
the points:

i. A-B
ii. B-C
iii. C-D
iv. D-E
v. E-F

115
NOTE: Practice personal hygiene protocols at all times
Activity 2: THE HEATING CURVE OF WATER

Directions: Use the cooling curve below to answer the following questions.

Photo credit: https://sites.google.com/site/heatingandcoolingcurves/_/rsrc/1299042706797/curveexplanation/hEATING%20CURVE.png

1. In which region(s) does temperature remain constant?

2. In which region(s) does temperature increase?

3. In which region(s) of the graph does a phase change occur?

4. In which region(s) of the graph would the substance only be in one phase?

5. In which region(s) of the graph would the substance be a solid only?

6. In which region(s) of the graph would the substance be a solid and a liquid?

7. In which region(s) of the graph would the substance be a liquid and a gas?

8. In which region(s) of the graph would the substance be a gas only?

9. In which region(s) of the graph does boiling take place?

10. In which region(s) of the graph does melting take place?

116
NOTE: Practice personal hygiene protocols at all times
Reflection:

1. I learned that

2. I enjoyed most on

3. I want to learn more on

117
NOTE: Practice personal hygiene protocols at all times
References:

• https://www.quora.com/How-do-you-determine-the-freezing-point-of-a-
solution-do-you-follow-this-process-for-every-solution

• Curriculum Guide and Teaching Guide. K to 12 Basic Education Curriculum


Senior High School – Science, Technology, Engineering and Mathematics
(STEM) Specialized Subject
• http://teachtogether.chedk12.com/teaching_guides/view/499
• courses.lumenlearning.com/cheminter/chapter/heating-and-cooling-curves-
also-called-temperature-curves/
• https://www.rcboe.org/cms/lib/GA01903614/Centricity/Domain/1951/Heating%
20and%20Cooling%20Curves%20new.pdf
• https://www.tes.com/teaching-resource/graphs-and-heating-cooling-curves-
worksheet-6064146
• https://chem.libretexts.org/Bookshelves/General_Chemistry/Map%3A_Chemis
try_The_Central_Science_(Brown_et_al.)

118
NOTE: Practice personal hygiene protocols at all times
Answer Key

Activity 1:

i. A-B gas
ii. B-C condensation
iii. C-D liquid
iv. D-E freezing
v. E-F solid

Activity 2:

1. Region 2 & 4
2. Region 1, 3, 5
3. Regions 2 & 4
4. Region 1, 3, 5
5. Region 1
6. Region 2
7. Region 4
8. Region 5
9. Region 4
10. Region 2

Prepared by:

JACKIE B. UBINA
Solana National High School

119
NOTE: Practice personal hygiene protocols at all times
GENERAL CHEMISTRY 2
Name:__________________________ Grade Level:___________

Date:___________________________ Score:________________

LEARNING ACTIVITY SHEET

WAYS OF EXPRESSING CONCENTRATION OF SOLUTIONS

Background Information for the Learners (BIL)

The term solution is used in Chemistry to describe a homogeneous mixture in


which at least one substance (the solute) is dissolved in another substance (the
solvent). The solvent is the substance in greater quantity and the name of the of the
solution is taken from the name of the solute. For example, when sodium chloride is
dissolved in water, sodium chloride is the solute, and water is the solvent, and the
solution is called a sodium chloride solution.

There are different methods of expressing solution concentrations namely;


Molarity, Molality, Percent by Mass, Percent by Volume, Mole fraction and Parts Per
Million. These methods are used to express relative amounts of solute and solvent in
a solution. In other words, the concentration of a solution is the amount of solute
present in a given amount of solvent, or a given amount of solution.

Percent by Mass

The Percent by Mass (also called percent by weight or weight percent) is the ratio
of the mass of a solute to the mass of the solution, multiplied by 100 percent:

Percent by Mass = Mass of solute


X 100%
Mass of solute + Mass of solvent

120
NOTE: Practice personal hygiene protocols at all times
Or,

Mass of solute
Percent by Mass = X 100%
Total mass of Solution

Let us consider the examples below;

Example 1: In a solution prepared by dissolving 24g of Sodium Chloride


(NaCl) in 152g of water, what is the mass percent of Sodium
Chloride (NaCl)?

Solution: First, identify the given.

Given: Solute = 24g of NaCl

Solvent = 152g of Water

Second, identify the unknown or what is being asked in the


problem.

Unknown = Percent by Mass of NaCl.

Third, write the formula and calculate the unknown.

Mass of solute
Percent by Mass = X 100%
Mass of solute + Mass of solvent

24g NaCl
Percent by Mass of NaCl= X 100%
24g NaCl + 152g Water

24g
Percent by Mass of NaCl= X 100% = 14%
176g

121
NOTE: Practice personal hygiene protocols at all times
Example 2: A sample of 0.892 g of potassium chloride (KCl) is dissolved
in 54.6 g of water. What is the percent by mass of KCl in the
solution?

Solution: First, identify the given.

Given: Solute = 0.892g of KCl

Solvent = 54.6g of water

Second, identify the unknown or what is being asked in the


problem.

Unknown = Percent by Mass of KCl.

Third, write the formula and calculate the unknown.

Mass of solute
Percent by Mass = X 100%
Percent by Mass =Mass of solute + Mass of solvent

0.892g of KCl
Percent by Mass of KCl= X 100%
0.892g of Cl + 54.6g of Water

Percent by Mass of KCl= 0.892g


X 100% = 1.61%
55.492g

Percent by Volume

Percent by Volume or Volume Percent is a common expression used for


expressing concentration. It is related to the molar concentration but the difference is
that the volume percent is expressed with a denominator of 100. It is used for reporting
concentration of liquids solutes in solution. It is also called %V/V and it is always
expressed as percentage (%) and the units of the volume should be in mL. . Percent

122
NOTE: Practice personal hygiene protocols at all times
by volume is also widely use in pharmaceutical field for expressing the concentration
of different components in solution.

Volume of solute
Percent Volume = X 100%
Volume of solute +Volume of solvent

Or,

Volume of solute
Percent Volume = X 100%
Total Volume of Solution

Let us consider the examples below;

Example 1: A solution of propanol (CH3CH2CH2OH) is prepared by


dissolving 67 mL in enough water to have a final volume of 250
mL. What is the volume percent of the propanol?

Solution: First, identify the given.

Given: Solute = 67mL propanol

Solution = 250mL

Second, identify the unknown or what is being asked in the


problem.

Unknown = Percent Volume of Propanol

123
NOTE: Practice personal hygiene protocols at all times
Third, write the formula and calculate the unknown.

Volume of solute
Percent Volume = X 100%
Total Volume of Solution

Percent Volume = 67 mL
X 100% = 26.8%
250mL

Example 2: How many mL of HNO3 concentrate are needed to prepare


250 mL of solution 4%?

Solution: First, identify the given

Given: solution = 250mL

Volume = 4 percent by

Second, identify the unknown or what is being asked in the


problem.

Unknown = Volume of solute (HNO3)

Third, write the formula and calculate the unknown. But in this
case we have to derive the formula.

From the mother formula;


Volume of solute
Percent Volume = X 100%
Total Volume of Solution

124
NOTE: Practice personal hygiene protocols at all times
To derived formula to get the volume of solute.
(Percent by Volume) (Volume of Solution)
Volume of Solute =
100%

Thus,

Volume of Solute = (4%) (250mL)


= 10mL
100%

Parts per Million

When the amount of solute is very small, as with trace impurities in water,
concentration is often expressed in parts per million.

PPM is a term used in chemistry to denote a very, very low concentration of a


solution. One gram in 1000 ml is 1000 ppm and one thousandth of a gram (0.001g) in
1000 ml is one ppm. Parts Per Million (ppm) is a measurement of the concentration of
a solution.

Gram of Solute X 106


Parts per Million =
Gram of Solution

Let us consider the examples below;

Example 1: What is the concentration of a solution in parts per million, if


0.02 grams of NaCl is dissolved in 1000 grams of solution?

Solution: First, identify the given.

Given: Solute = 0.02g of NaCl

Solution = 1000g

125
NOTE: Practice personal hygiene protocols at all times
Second, identify the unknown or what is being asked in the
problem.

Unknown = Concentration in parts per million

Third, write the formula and calculate the unknown

Gram of Solute X 106


Parts per Million =
Gram of Solution

0.02 grams X 106 = 20 ppm


Parts per Million =
1000 grams

Example 2: What is the total mass of solute in 1000g of a solution having


a concentration of 5ppm?

Solution: First, identify the given

Given: Solution = 1000 grams

Concentration in ppm = 5ppm

Second, identify the unknown or what is being asked in the


problem.

Unknown = Mass of Solute

Third, write the formula and calculate the unknown. But in this
case we have to derive the formula.

126
NOTE: Practice personal hygiene protocols at all times
From the mother formula;
Gram of Solute
Parts per Million = X 106
Gram of Solution

To derived formula to get the unknown which is the mass of solute.

(ppm) (gram of solution)


Gram of Solute =
1000000

Thus,

( 5 ppm) (1000 grams)


Gram of Solute = = .005g
1000000

Mole Fraction

It is a dimensionless quantity that expresses the ratio of the number of moles


of one component to the number of moles of all components present. For a mixture of
two substances, A and B, the mole fractions of each would be written as follows:

Mole fraction of component A: mol A


XA =
mol A + mol B

mol B
Mole fraction of component B:
XB =mol A + mol B

In general, the mole fraction of component “i” in a mixture is given by;

ni
Xi =
nT

127
NOTE: Practice personal hygiene protocols at all times
where ni and nT are the number of moles of component i and the total number of
moles present, respectively. The mole fraction is always smaller than 1.

*The mole fraction is unitless or dimensionless because it is a ratio of two similar


quantities*

Let us consider the examples below;

Example 1: 0.100 mole of NaCl is dissolved into 100.0 grams of pure H 2O.
What is the mole fraction of NaCl? What is the mole fraction of
H2O?

Solution: First, identify the given

Given: 0.100 mole of NaCl

100.0 grams of pure H2O

Second, identify the unknown or what is being asked in the


problem.

Unknown: mole fraction of H2O.

*Note that the component being asked in the problem is the water (H 2O) component,
but as you may notice, the unit of water as stated in the problem is in grams. Before
you can finally input the all the given in the formula you have to make sure that units
to be used are appropriate. Since we are dealing with mole fraction, we have to
convert 100 grams of H2O into moles using the molar mass of H2O (18g/mol).*

Converting 100 grams of water into moles:

(1 mol H2O)
(100 grams of H2O) X = 5.56mol H2O
(18.0g H2O)

128
NOTE: Practice personal hygiene protocols at all times
Third, write the formula and calculate the unknown.

ni
Xi =
nT

5.56 mol
Xi = = 0.982
5.66 mol

Example 2: A solution is prepared by mixing 25.0 grams of water and 25.0


grams of ethanol (C2H5OH). Determine the mole fractions of
each substance.

Solution: First, identify the given

Given: 25 grams of water

25 grams of ethanol

*As you may notice, all the given are in grams. You may think that you could solve
right away for the mole fraction since you will arrive in a unit less answer. But that is
not how it works in mole fraction because mole fraction deals with moles, and so we
need to convert this grams into moles first before we can be able to get the mole
fraction. In converting the given grams to moles, refer to the method shown in
example 1 and the molar mass of the substance can be summed up using the mass
of the atoms in that given substance.*

Thus, 25 grams of water = 1.34 mol of water

25 grams of ethanol = 0.543 mole of ethanol

Second, identify the unknown or what is being asked in the


problem.

129
NOTE: Practice personal hygiene protocols at all times
Unknown: Mole fractions of each substance.

Therefore, for this problem we have to treat water as


component A and ethanol as component B.

Third, write the formula and calculate the unknown.

Water as component A Ethanol as component B

XA = mol A mol B
XB =
mol A + mol B mol A + mol B

XA = 1.34 mol 0.543 mol


= 0.71 XB =
= 0.29
1.34 mol+ 0.543mol 1.34 mol + 0.543mol

Molarity

Otherwise known as “molar concentration”. It is defined as the number of moles of


solute per liter of solution. The SI unit for molarity is mol/m3; however, you will almost
always encounter molarity with the units of mol/L. A solution of concentration 1 mol/L
is also denoted as “1 molar” (1 M). Mol/L can also be written in the following ways
(however, mol/L, or simply M, is most common)

It is important to keep in mind that molarity refers only to the amount of solute
originally dissolved in water and does not take into account any subsequent
processes, such as the dissociation of a salt or the ionization of an acid.

In equation form it is written as;


Moles of solute
M=
Liter of Solution

and can be expressed algebraically as;


n
M=
v

130
NOTE: Practice personal hygiene protocols at all times
Where n, denotes the number of moles of solute. And v is the volume of solution
in liters. Note that the volume in the definition of Molarity refers to the volume of
solution and not the volume of the solvent. The reason for this is because one liter of
solution usually contains either slightly more or slightly less than 1 liter of solvent, due
to the presence of the solute.

Let us consider the examples below;

Example 1: How many grams of potassium dichromate (K2Cr2O7) are required to


prepare a 250-mL solution whose concentration is 2.16 M ?

Solution: First, identify the given

Given: Molarity = 2.16M

Solution = 250mL

*Note that the solution must be converted into Liters and so


250 mL is equal to 0.250L.*

Second, identify the unknown or what is being asked in the


problem.

Unknown = Potassium dichromate in grams

Third, write the formula and calculate the unknown. But in this
case we have to derive the formula.

From the mother formula; n


M= v
M=

To derived formula to get the moles of potassium dichromate. (which later on be


converted into grams since the problem asks for the quantity of potassium dichromate
in grams). N = (M) (v)

131
NOTE: Practice personal hygiene protocols at all times
Thus,
N = (2.16M) (0.250L) = 0.54 mol of potassium dichromate.

The 0.54 mol potassium dichromate is not yet the final answer because we still must
convert it into grams. Using the molar mass of K2Cr2O7 which is 294.2 g.

Converting 0.540mol of K2Cr2O7 to grams we have;

294.2 g of K2Cr2O7
= 159g of K2Cr2O
(0.540mol of K2Cr2O7) X 1mol of K2Cr2O

Example 2: In a biochemical assay, a chemist needs to add 3.81 g of


glucose to a reaction mixture. Calculate the volume in milliliters
of a 2.53 M glucose solution she should use for the
addition.

Solution: First, identify the given

Given: 3.81 grams of glucose

2.53 M of glucose

*Glucose is given in grams and we must first convert it into moles using its molar mass
which is equal to 180.2 grams. Thus, 3.81 grams of glucose is equal to 2.114x10-2 mol
of glucose.*

Second, identify the unknown or what is being asked in the


problem.

Unknown: Volume in mL of a 2.53 M glucose solution

132
NOTE: Practice personal hygiene protocols at all times
Third, write the formula and calculate the unknown. But in this
case we have to derive the formula.

From the mother formula;


n
M= v

To derived formula;
n
V= M

Thus,
2.114x10-2mol of glucose
= 8.36x10-3 L
V= 2.53M

*Notice that our units is in Liter and the problem is asking for the uni to be in mL, that
is why we must convert 8.36x10-3 L into mL which is equivalent to 8.36 mL solution.
And 8.36 mL is the final answer for this problem.*

Molality

It is an intensive property of solutions, and it is calculated as the moles of a solute


divided by the kilograms of the solvent. Unlike molarity, which depends on the volume
of the solution, molality depends only on the mass of the solvent. Molality is the
number of moles of solute dissolved in 1 kg (1000 g) of solvent—that is,

Moles of solute
m=
Mass of solvent

The SI unit for molality is mol/kg. A solution with a molality of 3 mol/kg is often
described as “3 molal” or “3 m.” However, following the SI system of units, mol/kg or
a related SI unit is now preferred.

133
NOTE: Practice personal hygiene protocols at all times
Let us consider the examples below;

Example 1: Calculate the molality of a sulfuric acid solution containing


24.4 g of sulfuric acid in 198 g of water. The molar mass of
sulfuric acid is 98.09 g.

Solution: First,identify the given

Given: Solute = 24.4 grams of sulfuric acid

Solvent = 198 grams of water

*Always be mindful with the units, the solute given is in grams and it should be first
converted into moles (that is 0.249 molH2SO4) and also the solvent is expressed in
grams that should be in kilograms so, solvent must be 0.198 kg).*

Second, identify the unknown or what is being asked in the problem.

Unknown: Molality of sulfuric acid

Third, write the formula and calculate the unknown

Moles of solute
mm==
m = Mass of solvent

Thus,
0.249 molH2SO4
m= = 1.26m
0.198 kg

Example 2: 80.0 grams of glucose (C6H12O6 ) is dissolved in 1.00 kg of


solvent. What is its molality? Molar mass of (C6H12O6 ) is
equal to 180g/mol.

Solution: First, identify the given;

Given: Solute = 80.0 grams of glucose

Solvent = 1.00kg

134
NOTE: Practice personal hygiene protocols at all times
*Remember that, molality should be in the units of moles and kilogram. So, 80.0 grams
of glucose should be converted first into moles. Thus, glucose is equal to 0.444 mol.*

Second, identify the unknown or what is being asked in the


problem.

Unknown: Molality of Glucose solution

Third, write the formula and calculate the unknown

Moles of solute
m=
Mass of solvent

Thus,
0.444 mol glucose = 0.444m
m=
1.00kg

Learning Competency:
Use different ways of expressing concentration of solutions: Molarity, Molality, Percent
by mass, Percent by volume, mole fraction and ppm. (STEM_GC11PP-IIId-f-111)

Activity 1: Choose The “RIGHT” One

Directions: Read and analyze the following questions and choose from the given
options the best correct answer.

1. Which of the following is not the unit of concentration?


a. Mole/m3
b. Molar
c. N/m3
d. ppm

135
NOTE: Practice personal hygiene protocols at all times
2. Which of the following material present in a solution is largest in amount?
a. Salt
b. Solute
c. Solvent
d. Molecules
3. Which of the following is defined as the relative amount of solute and solvent in a
solution?
a. Polarity
b. Solubility
c. Miscibility
d. Concentration
4. Which of the following describes a solvent in a solution?
a. Always a water
b. Always a liquid
c. The substance being dissolved
d. The substance present in the greatest amount

5. Which of the following is defined as the quantity of solute per unit volume?
a. Density
b. Concentration
c. Mole
d. None of the above mentioned

136
NOTE: Practice personal hygiene protocols at all times
Activity 2: CONCEPTUAL ANALYSIS

Directions: Base on what you have learned from this lesson and from other previous
lessons. Analyze the given statement and scientifically discuss your claim.

A solution is prepared at 20oC and its concentration is expressed in two different


units; Molarity and Molality. The solution is then heated to 88oC. Which of the
concentration units will change?

Activity 3: MATCH ME!

Directions: Read and analyze the following questions and compute for what is
unknown in the given problem. Choose the correct numerical value from the response
list on the right. Responses on the right may be used more than once or need not be
used at all.

B. 58.44g
1. What is the percent by volume concentration of a
A. 1.43mL
solution in which 75.0mL of ethanol is diluted to a volume
of 250mL? D. 12.39%
2. What volume of acetic acid is present in a bottle
C. 1gram
containing 350.0mL of a solution which measures 5.00%
concentration. E.17.5mL
3. Find the percent by mass in which 41.0g of NaCl is
dissolved in 331g of water. F.2.0x1010ppm

4. How many grams of NaCl would you need to prepare G. 30%


200.0mL of a 5M solution.
J. 60ppm
5. What is the ppm concentration of 6.00 mL sample of
solution that has 3.6 x 10-4 g of sodium ions? I. 8.07%

H. 33.3%

137
NOTE: Practice personal hygiene protocols at all times
Activity 4: Calculate The Unknown

Directions: Read and analyze the following questions and compute for what is
unknown in the given problem. Show complete solution by stating the given and
unknown, and show the process from writing the formula to unit conversion (if
applicable).

1. Suppose you added 4.0 moles of sugar to 10.0 L of solution. Calculate the molar
concentration of the solution.
2. A sample of water taken from a nearby lake is found to have 0.0035 mol of salt in a
100mL solution. Determine the molar concentration of the solution in the lake.
3. You dissolve 30.0g of sodium sulfate (Na2SO4(s)) into 300mL of water. Calculate the
molar concentration of the solution.
4. What is the Molality of a solution containing 7.78g of Urea [(NH 2)2CO2] in 203g of
water?
5. Lead is a poisonous metal that especially affects children because they retain a
larger fraction of lead than adults do. Lead levels of 0.250ppm in a child cause delayed
cognitive development. How many moles of lead present in 1.00g of child’s blood
would 0.250ppm represent.
6. Acetone, C3H6, is the main ingredient of nail polish remover. A solution is made up
by adding 35.0mL of acetone (d=0.790g/mL) to 50.0mL of ethyl alcohol, C 2H6O
(d=0.789g/mL). Assuming volumes are additive, calculate (a) the mass percent of
acetone in the solution. (b) the volume percent of ethyl alcohol in the solution. (c) the
mole fraction of acetone in the solution.

Activity 5: Expressing Concentration in Different Units

Directions: Read and internalize the short story below and answer the questions that
follow. Complete solution is required.

“The coronavirus disease (COVID-19) is an infectious disease caused by a new strain


of coronavirus. This new virus and disease were unknown before the outbreak began
in Wuhan, China, in December 2019.

138
NOTE: Practice personal hygiene protocols at all times
On 30 January 2020, the Philippine Department of Health reported the first case of
COVID-19 in the country with a 38-year-old female Chinese national. On 7 March, the
first local transmission of COVID-19 was confirmed. WHO is working closely with the
Department of Health in responding to the COVID-19 outbreak.” Aki and her family
were alarmed with the news they watched and so first thing in the morning they rush
to the nearest convenient store to secure disinfectant and sanitizers but unfortunately
the store already had empty shelves of the essentials they needed. They went to other
stores searching and to their dismay they acquired nothing. They were on their way
home when she suddenly remembered her past lesson on “solutions”, and so she
immediately ran back to the store and purchase the things she needed for her simple
experiment. She bought a bleach (Zonrox), gloves and measuring spoon and cups.
Arriving at home she then put her gloves on and prepared the things she needed such
as; 5tbsp bleach (0.0739L), 1 gallon of water (3.8L), pail and stirring rods. Using the
pail with 3.8L of water, she carefully poured the 5tbsp bleach solution and then mixed
it with the stirring rod. And they now have a disinfectant.

In connection to her home made disinfectant and with our lesson, we will express her
solutions’ concentration into different units; Molarity, Molality, Percent by mass,
Percent by volume, mole fraction and ppm.

Questions:

1. What is the molar concentration of Aki’s disinfectant if she dissolved 5Tbsp. of


NaClO (sodium hypochlorite) in 3.8 liters of H2O (water)?
2. Compute for the molality of her disinfectant if she dissolved 5Tbsp. of NaClO
(sodium hypochlorite) in 3.8 liters of H2O (water).
3. Calculate the percent by mass of sodium hypochorite in her disinfectant solution. (
5Tbsp. of NaClO (sodium hypochlorite) and 3.8 liters of H2O (water). In units of grams
for both of the solute and solvent.
4. Calculate the mole fraction of sodium hypochorite and water in Aki’s solution. (
5Tbsp. of NaClO (sodium hypochlorite) and 3.8 liters of H2O (water).
5. Calculate the percent by volume of the disinfectant Aki made. Units in mL.
6. What is the concentration of her solution in parts per million? ( 5Tbsp. of NaClO
(sodium hypochlorite) and 3.8 liters of H2O (water).

139
NOTE: Practice personal hygiene protocols at all times
Reflection

1. I learned that _______________________________________________________


___________________________________________________________________
_________________________________________________________

2. I enjoyed most on ___________________________________________________


___________________________________________________________________
___________________________________________________________________.

3. I want to learn more on _______________________________________________


___________________________________________________________________
___________________________________________________________________.

References:

Masterton, William and Cecille Hurley. Chemistry Principles and Reactions.


Solutions.Fifth Edition, Thomson Books/Cole,2004.

Hein,Morris et.al. Foundations of Chemistry in the Laboratory for Sciences. Properties


of Solutions. Twelfth Edition, John Wiley and Sons Inc.,2007.

Whitten, Kenneth et.al. General Chemistry. Solutions. Seventh Edition, Thomson


Books/Cole,2004.

Chang, Raymond. Chemistry. Concentration Units. Tenth Edition, McGraw-Hill, 2010.

140
NOTE: Practice personal hygiene protocols at all times
ANSWERS:
ACTIVITY 2
ACTIVITY 1 Molarity, because it varies with
1. C temperature and because volume is
2. C temperature dependent. Molality on the
3. C other hand is temperature independent
4. D because mass units are independent of
5. B temperature.

ACTIVITY 3 ACTIVITY 4
1. G 1. 0.4 moles
2. E 2. 0.035M
3. C 3. 0.704M
4. A 4. 0.49m
5. H 5. 1.21x10-9
6. a)4.17% b)58.8% c)0.358moles

ACTIVITY 5
Conversion:
Solute = 5tbsp = 0.0739L = 73.9mL = 73.9g = 0.99moles

Solvent = 3.8L = 3.8kg = 3800g

Solution = 3.8739 L = 3873.9mL = 3.8739kg = 3873.9g

1. 0.256M
2. 0.261m
3. 1.91%
4. .00467 NaClO and 0.995 H2O
5. 1.91%
Prepared by:
6. 19,076ppm
ANGELIKA TORRES
Sta Ana Fishery National High School

141
NOTE: Practice personal hygiene protocols at all times
GENERAL CHEMISTRY 2

Name: ____________________________ Grade Level: _________


Date: _____________________________ Score: ______________

LEARNING ACTIVITY SHEET

STOICHIOMETRIC CALCULATIONS FOR REACTIONS IN SOLUTION

Background Information for the Learners (BIL)

Stoichiometry is the calculation of reactants and products in a certain chemical


reactions. It applies the law of conservation of mass wherein the total mass of the
reactants is always equal the total mass of the products, leading to the insight that the
relations among the value or amount of reactants and products typically produce a
ratio of positive numbers. This implies that if the amounts of the separate reactants
are known, then the amount of the product can be calculated and vice versa.
This image here, shows that the chemical reaction is balanced.

Source: https://en.wikipedia.org/wiki/Stoichiometry#/media/File:Combustion_reaction_of_methane.jpg

It shows that one molecule of methane, CH4 reacts with two molecules
of oxygen gas, O2 to produce one molecule of carbon dioxide, CO2 and two molecules
of water, H2O. This chemical reaction is an example of complete combustion.
Stoichiometry measures these numerical relationships and is used to calculate the
amount of products and reactants that are produced or needed in a given reaction.
Describing the mathematical relationships of the substances that contributed in
chemical reactions is what we call reaction stoichiometry. It measures the

142
NOTE: Practice personal hygiene protocols at all times
relationship between the amount of methane and oxygen that react to form carbon
dioxide and water.
Elements in the periodic table have a different atomic mass, and as collections
of single atoms or molecules have a fixed molar mass, measured with the unit mole
(6.02 × 1023 individual molecules, Avogadro's constant). Carbon-12 has a molar mass
of 12 g/mol. Thus, to compute the stoichiometry by mass, the number of molecules
needed for each reactant is expressed in moles multiplied by the molar mass of each
to give the mass of each reactant per mole of reaction. The mass ratios can be
computed by dividing each by the total number in the whole reaction.
Stoichiometry is often used to balance chemical equations. For example, the
two diatomic gases, hydrogen and oxygen, when it combine H2 and O2, it produce a
liquid, water, in an exothermic reaction, as described by the following equation:
2 H2 + O2 →2 H2O
It shows the 2:1:2 ratio of hydrogen, oxygen, and water molecules in the above
equation.
The molar ratio permits for conversion between moles of one substance and
moles of another. For example,
2 CH3OH +3 O2 →2 CO2 +4 H2O
the amount of water that formed by the combustion of 0.27 moles of CH3OH is
obtained using the molar ratio between CH3OH and H2O of 2 to 4.
Stoichiometry is also used for determining the molar proportions of elements in
stoichiometric compounds. For example, the stoichiometry of hydrogen, H2 and
oxygen, O2 in H2O is 2:1. In stoichiometric compounds, the molar proportions should
be whole numbers.

Determining the Amount of Product


The term stoichiometry can be used to find the quantity of a product produced
by a reaction. If a piece of solid copper (Cu) were added to an aqueous solution
of silver nitrate (AgNO3), the silver (Ag) would be substituted in a single displacement
reaction forming aqueous copper(II) nitrate (Cu(NO3)2) and solid silver. How many
silver, Ag is formed if 16.00 grams Cu is added to the solution of excess silver nitrate,
AgNO3?

143
NOTE: Practice personal hygiene protocols at all times
The following steps would be used:
1. Write and balance the chemical equation
2. Mass to moles conversion: Convert grams of Cu to moles of Cu
3. Mole ratio determination: Convert moles of Cu to moles of Ag produced
4. Mole to mass conversion: Convert moles of Ag to grams of Ag produced
The complete balanced equation would be:
Cu +2 AgNO3 → Cu(NO3)2 + 2 Ag
For the mass to mole conversion, the mass of copper (16.00 g) would be
converted to moles of copper by dividing the mass of copper to its molecular mass:
63.55 g/mol.

Now that the amount of Cu in moles (0.2518) is form, we can set up the mole
ratio. This is done by looking at the coefficients in the balanced equation: Cu and Ag
are in a 1:2 ratio.

Now that the moles of Ag produced is known to be 0.5036 mol, this amount can
be converted into grams of Ag produced to determine the final answer:

This set of calculations can be further shortened into a single step:

Stoichiometric Calculations
The coefficients in the balanced equation give the ratio of moles of reactants
and products.

144
NOTE: Practice personal hygiene protocols at all times
Source: https://www2.chemistry.msu.edu/courses/cem151/chap3lect_2009.pdf

From the mass of Substance A, you can use the ratio of the coefficients of A
and B to determine or calculate the mass of Substance B formed (if it’s a product) or
used (if it’s a reactant).

Example: 10 grams of glucose (C6H12O6) react in a combustion reaction. How many


grams of each product are produced?
C6H12O6(s) + 6 O2(g) 6 CO2(g) + 6 H2O(l)
10.g ? + ?
Starting with 10. g of C6H12O6, we calculate the moles of C6H12O6. Use the coefficients
to find the moles of H2O & CO2 and then turn the moles to grams
C6H12O6(s) + 6 O2(g) 6 CO2(g) + 6 H2O(l)
10.g ? + ?
MW: 180g/mol 44 g/mol 18g/mol
#mol: 10.g(1mol/180g)
0.055 mol 6(.055) 6(.055mol)
6(.055mol)44g/mol 6(.055mol)18g/mol
#grams: 15g 5.9 g

145
NOTE: Practice personal hygiene protocols at all times
Reaction Stoichiometry in Solutions
We can perform stoichiometric calculations for aqueous phase reactions just as
we can for reactions in solid, liquid, or gas phases. Much of chemistry takes place in
solution. Stoichiometry allows us to work in solution by giving us the concept of solution
concentration, or molarity. Molarity is a unit that is often abbreviated as capital M. It is
defined as the moles of a substance contained in one liter of solution. Almost always,
we will use the concentrations of the solutions as conversion factors in our
calculations. For instance, if a solution has a concentration of 1.20 M NaCl, this means
that there are 1.20 moles of NaCl per liter of solution.
Example 1: What mass of Aluminum (Al) is needed to react completely with 35.0 mL
of 2.0 M Hydrochloric acid?
Solution:
6 HCl + 2 Al 2 AlCl3 + 3 H2

35.0 mL HCl x 1L HCl x 2 mol HCl x 2 mol Al x 26.98 g Al


1000 mL 1L HCl 6 mol HCl 1 mol Al

Al = 0.63 g
Example 2. What volume (mL) of 0.75 M calcium nitrate would react completely with
148g of carbonate?
Solution:
Ca(NO3)2 + Na2CO3 CaCO3 + 2 NaNO3

148 g Na2CO3 x 1mol Na2CO3 x 1 mol Ca(NO3)2 x 1L Ca(NO3)2 x 1000 mL


105.99 g Na2CO3 1mol Na2CO3 0.75mol Ca(NO3)2 1L

CaCO3 = 1.900 mL

Learning Competency:
Perform stoichiometric calculations for reactions in solution (STEM_GC11PP-IIId-f-
112)

146
NOTE: Practice personal hygiene protocols at all times
Activity 1: FILL THE EMPTY LINE
Directions: Read the following statement below and solve the problem. In the
equation that follows each problem, write on the space provided for the mole ratio that
can be used to solve the problem. Write the correct answer on the space provided for.
The reaction of sodium peroxide and water produces sodium hydroxide
and oxygen gas. The following balanced chemical equation represents the
reaction.
2 Na2O2(s) + 2 H2O(l) → 4NaOH(s) + O2(g)
1. How many moles of NaOH are produced when 1.00 mol sodium peroxide reacts
with water?
1mol Na2O2 x _____________ = _________ mol NaOH
2. How many moles of oxygen gas are produced when 0.500 mol sodium peroxide
reacts with water?
0.5 mol Na2O2 x ___________ = __________ mol O2
3. How many moles of sodium peroxide are needed to produce 1.00 mol NaOH?
1 mol NaOH x ___________ = __________ mol NaOH
4. How many moles of water are required to produce 2.15 mol oxygen gas?
2.15 mol O2 x ___________ = __________ mol H2O
5. How many moles of water are needed for 0.100 mol of sodium peroxide to react
completely?
0.100mol Na2O2 x ___________ = __________ mol H2O

Activity 2: SIMPLE STOICHIOMETRY

Directions: Solve the following stoichiometry grams – grams problems.

The combustion of a sample of butane, C4H10 (lighter fluid), produced 2.64


grams of water.
2 C4H10 + 13O2 8CO2 + 10H2O
a. How many moles of water formed?
___________________________________________________________________
_________________________________________________________

147
NOTE: Practice personal hygiene protocols at all times
b.How many moles of butane burned?
___________________________________________________________________
_________________________________________________________
c. How many grams of butane burned?
___________________________________________________________________
_________________________________________________________
d. How much oxygen was used up in moles?
___________________________________________________________________
_________________________________________________________
e. How much oxygen was used up in grams?
___________________________________________________________________
_________________________________________________________

Activity 3: THINK ABOUT IT!

Directions: Solve the following simple stoichiometry problems.

1. 123 mL of a 1.00 M solution of NaCl is mixed with 72.5 mL of a 2.71 M solution of


AgNO3. What is the mass of AgCl(s) formed in the precipitation reaction?
AgNO3(aq) + NaCl(aq) AgCl(s) + NaNO3(aq)

___________________________________________________________________
_________________________________________________________
2. What volume (mL) of 0.70 M Sodium hydroxide (NaOH) is needed to
neutralize 270 mL of 0.40 M Sulfuric acid (H2SO4)?

2 NaOH + H2SO4 Na2SO4 + 2 H2O

___________________________________________________________________
_________________________________________________________

148
NOTE: Practice personal hygiene protocols at all times
3. Hydrogen gas can be produced through the following reaction.

Mg(s) + 2HCl(aq) → MgCl2(aq) + H2(g)

a. How many grams of HCl are consumed by the reaction of 2.50 moles of
magnesium?
______________________________________________________________
__________________________________________________
b. What is the mass in grams of H2 gas when 4.0 moles of HCl is added to the
reaction?
______________________________________________________________
__________________________________________________
4. Acetylene gas (C2H2) is produced as a result of the following reaction.

CaC2(s) + 2H2O(l) → C2H2(g) + Ca(OH)2(aq)

a. If 3.20 moles of CaC2 are consumed in this reaction, how many grams of H2O
are needed?
______________________________________________________________
__________________________________________________
b. How many grams of Ca(OH)2 would be formed with 3.20 moles of CaC2?
______________________________________________________________
__________________________________________________
5. Laughing gas (nitrous oxide, N2O) is sometimes used as an anesthetic in dentistry.
It is produced when ammonium nitrate is decomposed according to the following
reaction.

NH4NO3(s) → N2O(g) + 2H2O(l)

a. How many moles of NH4NO3 are required to produce 33.0g of N2O?


______________________________________________________________
__________________________________________________
b. How many moles of water are produced with 45.0g of N2O?
______________________________________________________________
__________________________________________________

149
NOTE: Practice personal hygiene protocols at all times
Activity 4: GIVE ME MY VALUE
Directions: Complete the equation by writing the correct value on the space
provided for.
For questions 1 – 3, refer to the equation below
4 Fe + 3 O2 → 2 Fe2O3
1. How many moles of Fe2O3 are produced when 0.275 moles of Fe is reacted?

0.275 mol Fe
mol Fe2O3

2. How many moles of Fe2O3 are produced when 31.0 moles of O2 is reacted?

3. How many moles of O2 are needed to react with 8.9 moles of Fe?

For questions 4 – 6, refer to the equation below


2 KClO3 → 2 KCl + 3 O2
4. How many moles of O2 will be formed from 1.65 moles of KClO3?

1.65 mol KClO3 ____ mol O2


____ mol O2
____ mol KClO3

5. How many moles of KClO3 are needed to make 3.50 moles of KCl?

3.50 mol KCl


_____ mol KClO3

150
NOTE: Practice personal hygiene protocols at all times
Activity 5: GIVE ME THE SOLUTION

Directions: Solve the following problems based on the chemical reaction below
4 Fe + 3 O2 → 2 Fe2O3
1. How many grams of Fe2O3 are produced when 42.7 grams of Fe is reacted?

2. How many grams of Fe2O3 are produced when 17.0 grams of O2 is reacted?

3. How many grams of O2 are needed to react with 125 grams of Fe?

151
NOTE: Practice personal hygiene protocols at all times
Reflection:

1. I learned that _______________________________________________________


___________________________________________________________________
_________________________________________________________

2. I enjoyed most on ___________________________________________________


___________________________________________________________________
_________________________________________________________

3. I want to learn more on _______________________________________________


___________________________________________________________________
_________________________________________________________

152
NOTE: Practice personal hygiene protocols at all times
References:
Hill, Petrucci. General Chemistry: An integrated approach, second edition. New
Jersey: Prentice Hall, 1999.

https://courses.lumenlearning.com/introchem/chapter/solution-stoichiometry/

https://chem.libretexts.org/Bookshelves/Inorganic_Chemistry/Modules_and_Website
s_(Inorganic_Chemistry)/Chemical_Reactions/Reactions_in_Solution

https://www2.chemistry.msu.edu/courses/cem151/chap3lect_2009.pdf

http://www.calhoun.k12.al.us/teacherpages/teacherfiles/Stoichiometric

https://iasmisserica.weebly.com/uploads/4/2/6/4/42642303/escanear0094.pdf

http://www2.ucdsb.on.ca/tiss/stretton/CHEM1/stoicwk2.html

https://www.murrieta.k12.ca.us/cms/lib5/CA01000508/Centricity/ModuleInstance/868
1/Stoichiometry_-_mole_to_mass.doc

153
NOTE: Practice personal hygiene protocols at all times
ANSWER KEY
Activity 1: FILL THE EMPTY LINE
The reaction of sodium peroxide and water produces sodium hydroxide and oxygen
gas. The following balanced chemical equation represents the reaction.
2 Na2O2(s) + 2 H2O(l) → 4NaOH(s) + O2(g)
1. How many moles of NaOH are produced when 1.00 mol sodium peroxide reacts
with water?
Answer:
1mol Na2O2 x 4 mol NaOH = 2.0 mol NaOH
2 mol Na2O2
2. How many moles of oxygen gas are produced when 0.500 mol sodium peroxide
reacts with water?
Answer:
0.5 mol Na2O2 x 1 mol O2 = 0.25 mol O2
2 mol Na2O2
3. How many moles of sodium peroxide are needed to produce 1.00 mol NaOH?
Answer:
1 mol NaOH x 2 mol Na2O2 = 0.5 mol NaOH
4 mol NaOH
4. How many moles of water are required to produce 2.15 mol oxygen gas?
Answer:
2.15 mol O2 x 2 mol H2O = 4.3 mol H2O
1 mol O2
5. How many moles of water are needed for 0.100 mol of sodium peroxide to react
completely?
Answer:
0.100mol Na2O2 x 2 mol H2O = 0.1 mol H2O
2 mol Na2O2

154
NOTE: Practice personal hygiene protocols at all times
Activity 2: SIMPLE STOICHIOMETRY

1. The combustion of a sample of butane, C4H10 (lighter fluid), produced 2.64 grams
of water.
2 C4H10 + 13O2 8CO2 + 10H2O
a. How many moles of water formed?
2.64g of H2O x 1 mol H2O = 0.147 mol H2O
18g H2O
b. How many moles of butane burned?
0.147 mol H2O x 2mol C4H10 = 0.0294 mol C4H10
10 mol H2O
c. How many grams of butane burned?
0.0294 mol C4H10 x 58 g C4H10 = 1.705 g C4H10
1 mol
d. How much oxygen was used up in moles?
0.147 mol H2O x 13 mol O2 = 0.191 mol O2
10 mol H2O
e. How much oxygen was used up in grams?
0.191 mol O2 x 32 g O2 = 6.112g O2
1 mol O2

Activity 3: THINK ABOUT IT!


1. 123 mL of a 1.00 M solution of NaCl is mixed with 72.5 mL of a 2.71 M solution of
AgNO3. What is the mass of AgCl(s) formed in the precipitation reaction?
AgNO3(aq) + NaCl(aq) AgCl(s) + NaNO3(aq)
AgCl(s) = 17.6 g

2. What volume (mL) of 0.70 M Sodium hydroxide (NaOH) is needed to


neutralize 270 mL of 0.40 M Sulfuric acid (H2SO4)?

2 NaOH + H2SO4 Na2SO4 + 2 H2O

NaOH = 3.09 ml

155
NOTE: Practice personal hygiene protocols at all times
3. Hydrogen gas can be produced through the following reaction.

Mg(s) + 2HCl(aq) → MgCl2(aq) + H2(g)

a. How many grams of HCl are consumed by the reaction of 2.50 moles of
magnesium?
HCl = 182g
b. What is the mass in grams of H2 gas when 4.0 moles of HCl is added to the
reaction?
H2 = 4.0g
4. Acetylene gas (C2H2) is produced as a result of the following reaction.

CaC2(s) + 2H2O(l) → C2H2(g) + Ca(OH)2(aq)

a. If 3.20 moles of CaC2 are consumed in this reaction, how many grams of H2O
are needed?
H20 = 115g
b. How many grams of Ca(OH)2 would be formed with 3.20 moles of CaC2?
Ca(OH)2 = 237g

5. Laughing gas (nitrous oxide, N2O) is sometimes used as an anesthetic in dentistry.


It is produced when ammonium nitrate is decomposed according to the following
reaction.

NH4NO3(s) → N2O(g) + 2H2O(l)

c. How many moles of NH4NO3 are required to produce 33.0g of N2O?


NH4NO3 = 0.749 mol
d. How many moles of water are produced with 45.0g of N2O?
H2O = 2.04 mol

156
NOTE: Practice personal hygiene protocols at all times
Activity 4: GIVE ME MY VALUE
For questions 1 – 3, refer to the equation below
4 Fe + 3 O2 → 2 Fe2O3
1. How many moles of Fe2O3 are produced when 0.275 moles of Fe is reacted?

0.275 mol Fe 2 mol Fe2O3


0.138mol Fe2O3
4 mol Fe

2. How many moles of Fe2O3 are produced when 31.0 moles of O2 is reacted?

31.0 mol O2 2 mol Fe2O3


20.7 mol Fe2O3
3 mol O2

3. How many moles of O2 are needed to react with 8.9 moles of Fe?

8.9 mol Fe 3 mol O2


6.7 mol O2
4 mol Fe

For questions 4 – 5, refer to the equation below

2 KClO3 → 2 KCl + 3 O2
4. How many moles of O2 will be formed from 1.65 moles of KClO3?

1.65 mol KClO3 _3_ mol O2


2.48 mol O2
__2__ mol KClO3

5. How many moles of KClO3 are needed to make 3.50 moles of KCl?

3.50 mol KCl 2 mol KClO3


3.50 mol KClO3
2 mol KCl

157
NOTE: Practice personal hygiene protocols at all times
Activity 5: GIVE ME THE SOLUTION
Directions: Solve the following problems based on the chemical reaction below
4 Fe + 3 O2 → 2 Fe2O3
1. How many grams of Fe2O3 are produced when 42.7 grams of Fe is reacted?

159.70 g
42.7 g Fe 1 mol Fe 2 mol Fe2O3 Fe2O3
61.0 g
55.85 g Fe 2 mol Fe 1 mol Fe2O3
Fe2O3

2. How many grams of Fe2O3 are produced when 17.0 grams of O2 is reacted?

159.70 g
17 g O2 1 mol O2 2 mol Fe2O3 Fe2O3
56.6 g
32 g O2 3 mol O2 1 mol Fe2O3
Fe2O3

3. How many grams of O2 are needed to react with 125 grams of Fe?

125 g Fe 1 mol Fe 3 mol O2 32 g O2

1 mol O2 53.7 g O2
55.85 g Fe 4 mol Fe

Prepared by:

SHAROLYN T. GALURA
Licerio Antiporda Sr National High School- Dalaya Annex

158
NOTE: Practice personal hygiene protocols at all times
GENERAL CHEMISTRY 2

Name: ____________________________ Grade Level: _________


Date: _____________________________ Score: ______________

LEARNING ACTIVITY SHEET


Effects of Concentration on the Colligative Properties of Solutions
Background Information for the Learners (BIL)
Colligative properties of solutions are properties that depend upon the
concentration of solute molecules or ions, but not upon the identity of the solute.
Colligative properties include vapor pressure lowering, boiling point elevation, freezing
point depression, and osmotic pressure.
Lowering the Vapor Pressure
Vapor pressure is the pressure of a vapor in thermodynamic equilibrium with ts
condensed phase in a closed container. When non-volatile solute is dissolved in
solvent, the vapor pressure of solvent is lowered. The presence of solute decreases
the rate of escape of solvent molecules resulting to lower vapor pressure.
Boiling Point Elevation
The boiling point of a liquid is defined as the temperature at which the vapor
pressure of that liquid equals the atmospheric pressure (760mm Hg). The addition of
the solute increases the boiling point of the solution. The atmospheric pressure
remains the same while the vapor pressure of the solution is lowered resulting in the
increase of the difference in atmospheric pressure and vapor pressure of the solution.
Therefore, a higher temperature is required to boil the solution.
Freezing Point Depression
Normal freezing or melting point is the temperature at which solid and liquid
are in equilibrium under 1 atm. Addition of solute will decrease the vapor pressure
and so will decrease the freezing point. In order for a liquid to freeze it must achieve
a very ordered state that results in the formation of a crystal.
If there are impurities in the liquid, i.e. solutes, the liquid is inherently less
ordered. The presence of impurities in a liquid or in a substance makes variation in the

159
NOTE: Practice personal hygiene protocols at all times
freezing point by making them low or high. Therefore, a solution is more difficult to
freeze than the pure solvent so a lower temperature is required to freeze the liquid.
Osmotic Pressure
This is the external pressure that must be applied to the solution in order to
prevent it being diluted by the entry of solvent via osmosis. Diffusion in liquids,
substance tend to move or diffuse from regions of higher concentration to region of
lower concentration. The overall effect is to equalize concentration throughout the
medium.
Osmosis, on the other hand is the movement of solvent particles across a
semipermeable membrane from a dilute solution into concentrated solution. The
solvent moves to dilute the concentrated solution and equalize the concentration on
both sides of the membrane.
Osmotic pressure is directly proportional to the concentration of the solution.
Therefore, doubling the concentration will also double the osmotic pressure. The
osmotic pressure of two solutions having the same molal concentration are identical.

Learning Competency
Describe the effect of concentration on the colligative properties of solutions (STEM-
GC11-PPIIId-f-115)

Activity 1: FACT OR BLUFF


Directions: Read each statement carefully. Identify whether the statement is a FACT
or BLUFF. Draw ☺ on the space if it is a FACT and  if it is a BLUFF.
1. Colligative properties arise from the fact that solute affects the concentration of
solvent. _________
2. Vapor pressure is a colligative property. __________
3. Lowering of vapor pressure is not dependent on the number of species present
in the solution. __________
4. Colligative properties of solution depend on the nature of the solute and the
solvent. _______
5. Colligative molality is the molality times the number of solute particles per
formula unit. ________

160
NOTE: Practice personal hygiene protocols at all times
6. Osmotic pressure is directly proportional to the concentration of the solution.
_____
7. Relative lowering of vapor pressure is a colligative property. ________
8. The boiling point of a solution decreases in direct proportion to the molality of
the solute._________
9. When non-volatile solute is dissolved in solvent, the vapor pressure of solvent
is lowered. __________
10. The depression of the freezing point is directly proportional to the molality of the
solvent. _________

Activity 2: 1 PIC, 4 SENTENCES


Directions: The pictures below illustrate the different effects of colligative properties
to solutions. Using four (4) sentences, answer the question that each picture depicts.
Write your answer on the spaces provided.

1. Why adding salt to water increases the boiling point?

__________________________________________
__________________________________________
__________________________________________
__________________________________________
__________________________________________
__________________________________________
__________________________________________
__________________________________________
__________________________________________

Source:
https://www.thoughtco.com/thmb/Wlx0HpISUQfVc401y0XNpBdcms
=/768x0/filters:no_upscale():max_bytes(150000):strip_icc():format(
webp)/GettyImages-1166175911-
fafaea7fa0f54e418c93d8aff001460b.jpg

161
NOTE: Practice personal hygiene protocols at all times
2. Why does seawater have lower freezing point than pure water?

__________________________________________
__________________________________________
__________________________________________
__________________________________________
__________________________________________
__________________________________________
__________________________________________
__________________________________________
__________________________________________

https://www.google.com.ph/url?sa=i&url=https%3A%2F%2Ftwitter.com%2
Fmpi_meteo%2Fstatus%2F1074242734129434624&psig=AOvVaw0lK2WkzZ
5BHXZeNI34w3fI&ust=1596870848098000&source=images&cd=vfe&ved=0
CAIQjRxqFwoTCIDOpu7EiOsCFQAAAAAdAAAAABAD

3.
__________________________________________
__________________________________________
__________________________________________
__________________________________________
__________________________________________
__________________________________________
__________________________________________
__________________________________________
__________________________________________

https://www.google.com/search?sxsrf=ALeKk03ZiIKudLOeyQ1ocit_OtM
P6JIKQw:1596790694111Eq

4. Which sample has the lowest vapor pressure? Why?

________________________________________________________________________________
________________________________________________________________________________
________________________________________________________________________________
________________________________________________________________________________
__________________________________________________________

162
NOTE: Practice personal hygiene protocols at all times
5. How can you regain the crispyness of a carrot and celery that have become limp?
What colligative property is involved in the process?

__________________________________________
__________________________________________
__________________________________________
__________________________________________
__________________________________________
__________________________________________
__________________________________________
__________________________________________
__________________________________________

https://opentextbc.ca/chemistry/chapter/11-4colligative-properties

163
NOTE: Practice personal hygiene protocols at all times
Activity 3: WHO WANTS TO BE A CHEMIST?
Directions: To become a chemist and receive a score, you need to answer the
question in each level. Your score increases as you go to a higher level.

Compare the properties of 1.0 M aqueous


LEVEL 5 sugar solution to a 0.5 M aqueous solution
of NaCl.

Why is vapor pressure lowering a


LEVEL 4
colligative property?

LEVEL 3 What is needed for a liquid to freeze?

What do colligative properties of solutions


LEVEL 2 depend on?

What are the different colligative


properties of solution?
LEVEL 1

164
NOTE: Practice personal hygiene protocols at all times
Reflection:

1. I learned that _______________________________________________________


___________________________________________________________________
_________________________________________________________

2. I enjoyed most on ___________________________________________________


___________________________________________________________________
___________________________________________________________________.

3. I want to learn more on _______________________________________________


___________________________________________________________________
___________________________________________________________________.

165
NOTE: Practice personal hygiene protocols at all times
References
Books
Santos, Gil Nonato S., Danac, Alfonso C., O-Chemistry III, 2009
Mortimer Charles E., Chemistry 6th Edition
Hagad, Hilda R., Phoenix Next Century Chemistry, 2003
Websites
https://chem.libretexts.org
https://opentextbc.ca
https://www.sparknotes.com
https://www.bhsu.edu

166
NOTE: Practice personal hygiene protocols at all times
Answer Key
ACTIVITY 1
1. FACT 6. FACT
2. BLUFF 7. FACT
3. BLUFF 8. BLUFF
4. BLUFF 9. FACT
5. FACT 10. BLUFF
ACTIVITY 2
1. Boiling point of water increases when salt is added because salt dissociates into Na+
and Cl- ions. The addition of these particles raises the boiling point because part of the
pressure the solution exerts on the atmosphere now comes from solute particles, not
just solvent (water) molecules. The phenomenon depends on the number of particles
formed in the solution.
2. Seawater has lower freezing point than pure water because of the presence of salt in
the seawater. Salt lowers the freezing point of water dissociating its components into
ions to work. This phenomenon is called freezing point depression.
3. Sample C has the lowest vapor pressure because it has higher solute concentration.
Lowering the vapor pressure is a colligative property of solution so it depends on the
concentration of solute molecules not on their nature.
4. If a limp carrot and celery are placed in a fresh water, the water moves into the carrot
and celery. This causes the carrot and celery to stiffen and return to its crispyness.
The movement of water from the surroundings to the carrots involved osmosis.
5. Rock salt are often placed in an icy road in order to prevent the increase of ice. As salt
lowers the freezing point of water, it also causes ice to melt. Thus, places with icy road
often do the process of de-icing using rock salt. This is a colligative property called
freezing point depression.
ACTIVITY 3
LEVEL 1
1. Lowering of vapor pressure
2. Boiling point elevation
3. Freezing point depression
4. Osmotic pressure

167
NOTE: Practice personal hygiene protocols at all times
LEVEL 2
Colligative properties of solution depend on the number of particles rather
than the nature of the solvent.

LEVEL 3
In order for a liquid to freeze it must achieve a very ordered state that
results in the formation of a crystal.
LEVEL 4
Lowering of vapor pressure is a colligative property because it only
depends on number of dissolved solute particles.

LEVEL 5
Compare the properties of 1.0 M aqeuous sugar solution to a 0.5 M aqeuous solution
of NaCl.
Two solutions – one containing 1 mol of
Despite the conc. of NaCl = ½ the conc. of sucrose both solutions have precisely the
same number of dissolved particles because each NaCl unit creates two particles
upon dissolution a Na+ and a Cl-. Both solutions have the same freezing point, boiling
point, vapor pressure, and osmotic pressure because those colligative properties of
a solution only depend on the number of dissolved particles.

Prepared by:

LOVEJOICE L. AMBABAG
Tuao Vocational and Technical School

168
NOTE: Practice personal hygiene protocols at all times
GENERAL CHEMISTRY 2

Name ________________________ Grade Level: __________________


Date: _________________________ Score: _______________________

LEARNING ACTIVITY SHEET

DIFFERENCE BETWEEN THE COLLIGATIVE PROPERTIES OF ELECTROLYTE


AND NONELECTROLYTE SOLUTIONS

Background Information for the Learners (BIL)

Colligative properties are physical properties of a solution that depends on the


amount of a solute but not on the nature of solute. This means similar amounts of
completely different solutes can alter these physical properties in similar quantities.
Hence, the colligative properties depend on the ratio of the solute amount and solvent
amount. Solutions have four colligative properties namely: vapor pressure lowering,
boiling point elevation, freezing point depression, and osmotic pressure. For a given
solute-solvent mass ratio, all colligative properties are inversely proportional to solute
molar mass. Electrolytes are substances that can form solutions that are able to
conduct electricity through this solution. Such solutions are known as electrolytic
solutions. Nonelectrolytes are substances that are not capable of forming electrolytic
solutions. Both these types (electrolytes and nonelectrolytes) have colligative
properties.

Colligative Properties of Electrolytes vs Nonelectrolytes


Colligative properties of electrolytes are Colligative properties of nonelectrolytes
the physical properties of electrolytic are the physical properties of non-
solutions that depend on the amount of electrolytic solutions that depend on the
solutes regardless the nature of solutes. amount of solutes regardless the nature
of solutes.
Solute
Electrolytes provide more solutes to the Nonelectrolytes provide low solute to
solution via dissociation; hence, the the solution since there is no

169
NOTE: Practice personal hygiene protocols at all times
colligative properties are considerably dissociation; hence, the colligative
changed. properties are not considerably changed
Effect on Colligative Properties
The effect of electrolytes on colligative The effect of nonelectrolytes on
properties is very high compared to colligative properties is very low
nonelectrolytes compared to electrolytes.

When an electrolyte is dissolved in a solvent such as water, the electrolyte


separates into ions (or any other conductive species). Therefore, dissolving one mole
of electrolyte always yields two or more moles of conductive species. Hence, the
colligative properties of the electrolytes are considerably changing when an electrolyte
is dissolved in a solvent. A simple example of an electrolyte solution is sodium chloride
in water. In the presence of water, solid sodium chloride dissociates as it is dissolved,
forming an electrolyte solution:

NaCl(s)→Na+(aq)+Cl−(aq)

When a nonvolatile (does not turn to vapor easily) solute is added to a liquid
to form a solution, the vapor pressure above that solution decreases. To understand
why that might occur, do a simple experiment in Activity 1.

Learning Competency:
Differentiate the colligative properties of nonelectrolyte solutions and of electrolyte
solutions (STEM_GC11PPIIId-f-116)

170
NOTE: Practice personal hygiene protocols at all times
Activity 1. Vapor-Pressure Lowering

Materials: Water
Figure 1.1 Experiment set-up
Aqueous sugar solution
for vapor-pressure lowering
Two glasses

One sealed enclosed container

Procedure:

1. Get two glasses placed side by side in a sealed enclosed container.


One glass contains pure water, the other an equal volume of an
aqueous solution of sugar. Take note of the volume of water and
sugar solution. (See figure 1.a)
2. Leave the set-up until the next day. Gradually measure the volume
of the sugar solution and that of the pure water. Is there any change
in their original volume? (See figure 1.b)

https://www.google.com/search?q=solvent+solution+imag
e+on+water+vapor+lowering&tbm=isch&ved=2ahUKEwi4
v8qau5PrAhUG6ZQKHZYrD-0Q2-
cCegQIABAA&oq=solvent+solution+image+on+water+va
por+lowering&gs_lcp=CgNpbWcQA1DNogRYsc4EYNHS
BGgAcAB4AIAB2ASIAYIjkgEMMC4xOC4yLjEuMS4xmAE
AoAEBqgELZ3dzLXdpei1pbWfAAQE&sclient=img&ei=u7
gyX_j1PIbS0wSW17zoDg&bih=705&biw=1525#imgrc=AX
-ZYiNHxp18xM

Analysis:
1. What can you infer about the change in volume of the sugar solution and that of
pure water?
________________________________________________________________
________________________________________________________________
________________________________________________________________
________________________________________________________________
2. What has caused this change in volume of the sugar solution and that of pure
water?
________________________________________________________________
________________________________________________________________
________________________________________________________________
________________________________________________________________

171
NOTE: Practice personal hygiene protocols at all times
After finishing the activity, you should now see why the vapor pressure
of the solvent decreases upon addition of a non-volatile, non-
electrolyte solute (sugar).

Figure 1.2 A nonvolatile solute reduces the rate of


vaporization of the solvent.

https://www.google.com/search?q=solvent+solution+imag
e+on+water+vapor+lowering&tbm=isch&ved=2ahUKEwi4
v8qau5PrAhUG6ZQKHZYrD-0Q2-
cCegQIABAA&oq=solvent+solution+image+on+water+va
por+lowering&gs_lcp=CgNpbWcQA1DNogRYsc4EYNHS
BGgAcAB4AIAB2ASIAYIjkgEMMC4xOC4yLjEuMS4xmAE
AoAEBqgELZ3dzLXdpei1pbWfAAQE&sclient=img&ei=u7
gyX_j1PIbS0wSW17zoDg&bih=705&biw=1525#imgrc=AX
-ZYiNHxp18xM

Non-electrolytes are substances that do not create conductive


solutions when dissolved in a solvent. In the activity, sugar is a nonelectrolyte because
when sugar is dissolved in water, it exists in molecular form (does not dissociate into
ions). These sugar molecules are incapable of conducting electric currents through
the solution.

The number of solutes present in a non-electrolytic solution is less compared


to an electrolytic solution. Therefore, the effect of nonelectrolytes on colligative
properties is also very low. For example, the degree of vapor pressure lowering by
adding NaCl is higher compared to the addition of sugar to a similar solution.

The vapor pressure of an electrolytic solution is dependent on the ratio of solute


to solvent molecules in a solution while the vapor pressure of nonelectrolyte solution
is directly influenced by the number of solute molecules present in a given amount of
solvent.

The extent to which a nonvolatile solute lowers the vapor pressure is


proportional to its concentration. This was discovered by French chemist Francois
Raoult (1830-1907). Raoult’s law states that for nonelectrolyte solutions, the partial
vapor pressure of a solvent over a solution (P1) is equal to the vapor pressure of the
pure solvent (Po1) multiplied by the mole fraction of the solvent (X1). This law is
mathematically expressed as:

P1 = X1 Po1

For solutions of electrolytes, the vapor pressure lowering equation can be


expressed as:

172
NOTE: Practice personal hygiene protocols at all times
P1= iPo1 X1

where i is the van’t Hoff factor, named after Jacobus Henricus van’ Hoff (1852-1911),
who won the very first Nobel Prize in chemistry in 1901 for his work on colligative
properties of solution. The i factor gives the number of particles per formula unit of
the solute. For example, NaCl solution dissociates to give one Na+(aq) and one Cl-(aq),
the iNaCl = 2, because there is one Na+ and one Cl- ion in solution per formula unit of
solute.

On the other hand, if both components of a solution are volatile (readily


evaporates), the vapor pressure of the solution is the sum of the individual partial
pressures. The total pressure is given by Dalton’s law of partial pressure:

PT = PA + PB
or
PT = XA PoA + XB PoB

Sample Problem:
What is the vapor pressure of the solution containing 20 g of sugar (C12H22O11)
in 1.5 kg water at 25oC. Note: Powater at 25oC is 24 torr

Explanation:

1 mole of sugar (C12H22O11) is 342 g and 1 mole of water (H2O) is 18 g. The number

of moles of each component is computed as:

For sugar:

1mol C12H22 O11


20 g C12H22 O11  = 0.06 mol C12H22 O11
342 g C12H22 O11

173
NOTE: Practice personal hygiene protocols at all times
For water:

1mol C12H22O11
1500 g H2O  = 83.33 mol H2O
18 g C12H22O11

And mole fraction of water (solvent) is computed as:

83.33 C12H22O11
XH2 O = = 1.00
83.33 + 0.06 C12H22O11

PH2 O = XH2 O  PoH2 O


Therefore, the vapor pressure is: = 1.00  24 torr
PH2 O = 24.0 torr

Solve the given problem:


Calculate the vapor pressure of salt solution containing 20 g salt (NaCl) in
1.5 kg of water at 25oC? Note: Powater at 25oC is 24 torr and molecular
weight of NaCl is 58 g/mol.
Note: Remember that NaCl is a strong electrolyte and dissociates to Na+
and Cl- ion respectively.

Osmotic Pressure

Osmosis is of prime importance to living organisms because it influences the


distribution of nutrients and the release of metabolic waste products. Living cells of
both plants and animals are enclosed by a semipermeable membrane called the cell
membrane, which regulates the flow of liquids and of dissolved solids and gases into
and out of the cell.

174
NOTE: Practice personal hygiene protocols at all times
Did you know?

Oceans cover over 72% of the earth and a reservoir for 97.2% of the earth’s water. However, out of
the 97.2% of earth’s water, only 2.5% is available as freshwater and less than 1% is available as
groundwater or surface water for human use. Scientists have found a new, cheaper and more economical
way of making seawater possible for drinking. The process is known as reverse osmosis. Through this
process, seawater is made to pass through a semi-permeable membrane and by applying a pressure
greater than 30 atm (Note: 30 atm is the pressure that must be applied to saline solution of seawater in
order to facilitate osmosis), the osmotic flow would be reversed, and fresh water made to run through the
membrane. Now, that’s chemistry in action!

The osmotic pressure ( ) of a solution is the pressure required to stop


osmosis. The osmotic pressure of the solution is given by:

 = MRT

where M is the molarity of solution, R the gas constant (0.0821 L . atm / K . mol), and
T the absolute temperature (in Kelvins).
The osmotic pressure is expressed in atm. And since osmotic pressure
measurements are carried out under constant temperature, molarity is preferred over
molality as concentration. Again, for solutions of electrolytes, the osmotic pressure
equation can be expressed as:

 = iMRT
where i is the van’t Hoff factor, the number of particles per formula unit of the solute.

Sample Problem:
What is the osmotic pressure of the solution containing 0.2 M sugar (C12H22O11)
solution at 25oC?

Explanation:

175
NOTE: Practice personal hygiene protocols at all times
 = MRT
0.2 mol C12H22O11 L  atm
 =  0.0821  298 K = 4.89
1L solution K  mol
 = 4.89 atm

Solve the given problem:


Calculate the osmotic pressure of 0.2 M salt solution (NaCl) at 25 oC?

Boiling Point Elevation

Remember that boiling point is the temperature at which the vapor of a liquid is
equal to the atmospheric pressure. In the presence of a non-volatile, non-electrolytic
solute such as sugar, the pressure that will be exerted by the vapor of the solvent is
lesser in comparison with that of the pure solvent at equal volumes. And as a
consequence, the solution will boil at a higher temperature than that of the pure
solvent.
To clearly understand what boiling point elevation is all about, you re task to do
Activity 2.

Activity 2. Boiling Point Elevation


Materials: three eggs

1 tbsp of salt

1 tbsp of sugar

176
NOTE: Practice personal hygiene protocols at all times
boiling water

Procedure:
1. Put the first egg in two cups of water and take note of the time until the water
boils.
2. Again, using two cups of water, put the second egg in the water and add 1
tbsp of salt. Record the time it will take the water to boil.
3. Repeat step 2, but add 1 tbsp of sugar instead of salt. Again, take note of
the time it will take the water to boil.
4. Compare the time it will take for water to boil and cook the hard-boiled egg
in step 1 to step 3. Record your observations.

Analysis:
1. Which set-up took less time to cook hard-boiled eggs? Why?
___________________________________________________________
___________________________________________________________
2. Did the water take more time to boil when electrolyte (NaCl) solute was
added? Why? ______________________________________________
___________________________________________________________
3. Did the water take more time to boil upon addition of non-electrolyte (sugar)
solute? Justify your answer.
___________________________________________________________

___________________________________________________________

___________________________________________________________

___________________________________________________________

Since we have already established that the boiling point of the solution is
greater than that of the pure solvent, the boiling point elevation (Tb) is mathematically
expressed as follows:

177
NOTE: Practice personal hygiene protocols at all times
Tb = Kbm

where (Kb) is boiling point elevation constant, equivalent to 0.52 oC/m for aqueous
solutions. This means that, for example, 1 mole of sugar (nonelectrolyte) in 1 kilogram
of water will increase the boiling point from 100oC to 100.52oC. And (m) is the molal
concentration of solute. It is also important to note that Tb is a positive quantity and
should be added to the boiling point of pure solvent (water), which is 100oC.
Remember that molality is used here over molarity because we are dealing with
a solution whose temperature is not constant and concentration cannot be expressed
in molarity because it changes with temperature.

Again, for solutions of electrolytes, the boiling point elevation equation can be
expressed as:

Tb = iKbm

where i is the van’t Hoff factor, the number of particles per formula unit of the solute.
Sample Problem:

At what temperature will the sugar solution boil if 20 g sucrose (C 12H22O11) is


added to 1.5 kg of water?

Explanation:

Remember that sugar is a nonelectrolyte so there will be no need for the van’t
Hoff factor. A mole of sugar (C12H22O11) is 342 g. Thus, molality of sugar can be
computed as:

20 g C12H22O11 1mol C12H22O11 0.04 mol C12H22O11


 = = 0.04 m
1.5 kg H2O 342 g C12H22O11 kg H2O

178
NOTE: Practice personal hygiene protocols at all times
The boiling point elevation is calculated as:

Tb = Kbm
= 0.52 oC x 0.04 m
m

Tb = 0.02oC

Thus, the boiling point of the solution is:

100oC + 0.02oC = 100.02oC

Solve the given problem:


At what temperature will salt solution boil if 20 g salt (NaCl) is added
to 1.5 kg of water? (Molecular weight of NaCl is 58 g/mol)
Note: Remember that NaCl is a strong electrolyte and dissociates to
Na+ and Cl- ion, respectively.

Freezing Point Depression?

Have you ever tried sprinkling salt over ice? What have you noticed? Salts such
as NaCl and CaCl2 cause ice to melt. This method of thawing depresses the freezing
point of water. To learn more about freezing point depression, it will help to do Activity
3.

179
NOTE: Practice personal hygiene protocols at all times
Activity 3.Freezing Point Depression

Materials: one tsp. salt stirrers


one glass of water spoon
one glass of crushed Ice thermometer

Procedure:
1. Using one glass of water and one glass of crushed ice, stir the mixture; then
using a thermometer, observe and record this temperature.
2. Add one tsp. of salt to the water/ice mixture, then observe and record the
temperature. You should repeat this procedure until the temperature
reaches 10oC. More ice should be added if necessary.
Analysis:
3. What happens to water and ice when salt is added to this mixture?
___________________________________________________________
___________________________________________________________
___________________________________________________________
4. What happens to the temperature when salt is added to the mixture?
___________________________________________________________
___________________________________________________________
___________________________________________________________
5. What variables would cause these differences?
___________________________________________________________
___________________________________________________________
___________________________________________________________

Water freezes at 0oC and boils at 100oC. Salt water will not freeze until the
temperature is below 0oC. The more salt, the lower the freezing point of the solution.
In the above experiment, energy is lost from the water in the form of heat. This heat
is used to melt the ice. Since heat is lost from the water the temperature of the water
goes down. Since there is now salt dissolved in the water it cannot freeze again, hence
we observe a lower temperature.

180
NOTE: Practice personal hygiene protocols at all times
The freezing point depression (Tf) is mathematically expressed as:

Tf = Kf m

where Kf is the freezing point depression constant. equivalent to -1.86 oC/m for
aqueous solutions. Again, for example, 1 mole of sugar (nonelectrolyte) in 1 kilogram
of water will decrease the freezing point from 0oC to -1.86oC. (m) is the molal
concentration of solute. And since we have already established that the freezing point
of the solution is less than that of the pure solvent, then Tf is a negative quantity and
should be subtracted from the freezing point of pure solvent (water), which is 0 oC.
Again, for solutions of electrolytes, the freezing point lowering equation can be
expressed as:

Tf = iKfm

where i is the van’t Hoff factor, the number of particles per formula unit of the solute.
Sample Problem:
At what temperature will the sugar solution freeze if 20 g sucrose (C 12H22O11)
is added to 1.5 kg of water?
Explanation:
Since sugar is a nonelectrolyte, there will be no need for the van’t Hoff factor. A
mole of sugar (C12H22O11) is 342 g. Molality (m) of sugar can be computed as:

20 g C12H22O11 1mol C12H22O11 0.04 mol C12H22O11


 = = 0.04 m
1.5 kg H2O 342 g C12H22O11 kg H2O

The freezing point depression is calculated as:


Tf = Kf m
= -1.86 oC x 0.04 m
m

Tf = -0.07oC
Thus, the freezing point of the solution is:

0oC - 0.07oC = -0.07oC

181
NOTE: Practice personal hygiene protocols at all times
Practice Exercise:
At what temperature will salt solution freeze if 20 g salt (NaCl) is added
to 1.5 kg of water? (Molecular weight of NaCl is 58 g/mol)
Note: Remember that NaCl is a strong electrolyte and dissociates to
Na+ and Cl- ion, respectively.

Did you know?

NaCl (salt) is added to ice to make a freezing mixture that


results in a tasty, homemade ice cream. Lowering the freezing
temperature of the ice-water-salt mixture causes the ice cream
ingredients to freeze more quickly.

182
NOTE: Practice personal hygiene protocols at all times
Let’s Summarize

A. Summary of Key Equations

For nonelectrolytes:

Raoult’s Law: P1 = Po1 X1


Dalton’s Law of Partial Presssure: PT = PA + PB

Osmotic Pressure:  = MRT


Boiling Point Elevation: Tb = Kbm
Freezing Point Depression:  T f = Kf m

For electrolytes:

Raoult’s Law: P1 = iPo1 X1


Dalton’s Law of Partial Presssure: PT = iPA + iPB

Osmotic Pressure:  = iMRT


Boiling Point Elevation: Tb = iKbm
Freezing Point Depression: Tf = iKf m

183
NOTE: Practice personal hygiene protocols at all times
References
Printed Materials

1.“5.9: Colligative Properties of Electrolyte Solutions.” Chemistry LibreTexts,


Libretexts, 21 July 2016.
2.“Colligative Properties.” Wikipedia, Wikimedia Foundation, 10 Mar.
2018.3.Britannica, The Editors of Encyclopaedia. “Electrolyte.” Encyclopædia
Britannica, Encyclopædia Britannica, Inc., 7 June 2017.

4. EASE module

Websites
https://www.google.com/search?source=univ&tbm=isch&q=electrolyte+and+nonelect
rolyte+solution+image&sa=X&ved=2ahUKEwj1ma6ez_zqAhWXdd4KHbCvDQYQsA
R6BAgKEAE&biw=1506&bih=687#imgrc=bDq8vMbx-zbsQM
http://csef.usc.edu/History/2004/Projects/S0531.pdf
http://www2.ucdsb.on.ca/tiss/stretton/CHEM4/APProperties_Solutions.html
https://www.thoughtco.com/boiling-point-elevation-problem-609464
https://www.thoughtco.com/understanding-freezing-point-depression-609182
https://www.differencebetween.com/difference-between-colligative-properties-of-
electrolytes-and-vs-nonelectrolytes/

Prepared by:

IVON A. ADDATU
Tuao Vocational Technical School- Patta Annex

184
NOTE: Practice personal hygiene protocols at all times
GENERAL CHEMISTRY 2

Name: ____________________________ Grade Level: _________


Date: _____________________________ Score: ______________

LEARNING ACTIVITY SHEET


Boiling Point Elevation and Freezing Point Depression

Background Information for the Learners (BIL)

Boiling Point Elevation


The boiling point of a liquid is defined as the temperature at which the vapor
pressure of the liquid is equal to the prevailing atmospheric pressure. Boiling points
measured under 1 atm pressure are called normal boiling points. Since a non-volatile
solute decreases the vapor pressure of a liquid, a solution will not boil at normal boiling
point of the solvent at 1 atm pressure. It is necessary to increase the temperature
above this point in order to attain a vapor pressure over the solution of 1 atm. The
boiling point of a solution containing a non-volatile molecular solute, therefore, is
higher than that of the pure solvent. The elevation is proportional to the concentration
of solute in the solution. Thus, boiling point elevation is the amount the boiling point
temperature increases compared to the original solvent.
The elevation of the boiling point; Δtb, can be calculated by multiplying the molal
boiling point elevation constant of the solvent, kb, by the molality of the solution,m.
That is, Δtb = mkbi
Wherein, Δtb = boiling point elevation
m = molality of the solution
kb = molal boiling point elevation constant of solvent
i = the number of particles formed when that compound dissolves (for
covalent compounds, this is always equal to 1)

185
NOTE: Practice personal hygiene protocols at all times
Freezing Point Depression
The freezing point is the temperature at which the liquid changes to solid. At
a given temperature, if a substance is added to a solvent (such as water), the solute-
solvent interactions prevent the solvent from going into the solid phase. The solute-
solvent interactions require the temperature to decrease further in order to solidify the
solution. A common example is found when salt is put on icy roadways. The salt is put
on the roads so that the water on the roads will not freeze at the normal 0°C but at a
lower temperature as low as -9°C.
The freezing point depression is the difference in the freezing points of the
solution from the pure solvent. This is true to any solute added to a solvent; the
freezing point of the solution will be lower than the freezing point of the pure solvent.
The freezing point depression, Δtf, of a solution can be calculated from the
molality of the solution and the constant for the solevnt, kf.
That is, Δtf = mkfi
Wherein, Δtf = freezing point depression
m = molality of the solution
kf = molal freezing point depression constant of solvent
i = the number of particles formed when that compound dissolves
(for covalent compounds, this is always equal to 1)

Learning Competency
Calculate boiling point elevation and freezing point depression from the concentration
of a solute in a solution (STEM_GC11PPIIId-f-117)

186
NOTE: Practice personal hygiene protocols at all times
ACTIVITY 1: COLOR THERMOMETER

Directions: Search for the boiling point and freezing of each sample. Mark the
thermometers with the boiling and freezing point of the given samples. Using your
coloring material, color the thermometer where the substance BOIL and FREEZE.
Color it RED where it freezes and BLUE where it boils.


0° 0°

WATER ACETIC ACID BENZENE ETHANOL CHLOROFORM

Activity 2: BOILING OR FREEZING?


Directions: The table below shows the temperature at which the solution begins to
boil or freeze. Assume that Concentration B is greater than Concentration A. Examine
the data carefully and answer the questions that follow.
Table1. Temperatures at which each solution is expected to begin boiling or
freezing.
Solution Temperature at which boiling Temperature at which freezing
begins (°C) begins (°C)
Concentration A Concentration B Concentration A Concentration B
Sucros 100.10 100.73 -0.35 -2.68
Glycerol 100.10 100.77 -0.35 -2.68
Sodium 100.20 101.46 -0.70 -5.36
chloride
Calcium 100.30 102.19 -1.05 -8.04
chloride
*Santos,G.N., Danac, A.C., O-Chemistry III, p.106

187
NOTE: Practice personal hygiene protocols at all times
Q1. What do you notice with the boiling point and freezing point of the different
solutions?
Q2. What do you notice when you compare the initial boiling temperatures for
the two concentrations?
Q3. What about the two groups of freezing temperatures?
Q4. How will you compare the boiling temperature of the solutions with that of
a pure water?
Q5. Does sodium chloride seem to deviate more from the boiling temperature
and freezing temperature of pure water than the sucrose and glycerol solutions? Why?

Activity 3: LET’S BOIL IT OUT!


Directions: Read and analyse each problem carefully. Identify the known variables
and solve for the unknown following the steps in solving a problem.
1. At what temperature will a solution that is composed of 0.73 moles of glucose
(C6H12O6) in 650mL of water (H2O) begin to boil?

2. At what temperature will a sucrose solution boil if it contains 1.55 moles of sucrose
in 600mL of water? kb of water is 0.51°C/m.

3. What is the boiling point elevation of a solution that contains 40.5g gylcerol,
C3H5(OH)3,in 100g of water? kb of water is 0.51°C/m.

188
NOTE: Practice personal hygiene protocols at all times
4. What is the boiling point of a solution that contains 12.5g biphenyl, C 12H10, in 100g
of bromobenzene? The normal boiling point of bromobenzene is 156°C, and kb for
bromobenzene is +6.26°C/m.

5. A solution of 10g of sodium chloride (NaCl) is added to 100g water (H 2O) in an


attempt to elevate the boiling point. What is the boiling point of the solution? kb of water
is 0.51°C/m.

Activity 4: FROZEN
Directions: Apply your knowledge of molality to solve the problems below involving
freezing point depression of a solution. Label each variable with the correct unit of
measure.
1. What is the lowest freezing temperature for a saltwater solution? The solubility of
sodium chloride (NaCl) is 280g per 1000g of water at 0°C. kf of H2O = 1.86°C/m.

189
NOTE: Practice personal hygiene protocols at all times
2. At what temperature will a solution freeze if 200g of calcium chloride, CaCl 2, is
dissolved in 850mL of water? kf H2O = 1.86°C/m

3. What is the freezing point of a solution that contains 64.3g of sucrose, C6H12O6, in
200g of water? kf H2O = 1.86°C/m

4. What is the freezing point of a solution that contains 610g of benzoic acid,
HC7H5O2, in125g of camphor? kf camphor = 40°C/m

190
NOTE: Practice personal hygiene protocols at all times
5. Determine the freezing point depression of 24g urea, CH4N2O, dissolved in 200g of
water. kf H2O = 1.86°C/m

Reflection

1. I learned that _______________________________________________________


___________________________________________________________________
_________________________________________________________

2. I enjoyed most on ___________________________________________________


___________________________________________________________________
___________________________________________________________________.

3. I want to learn more on _______________________________________________


___________________________________________________________________
___________________________________________________________________.

191
NOTE: Practice personal hygiene protocols at all times
References:

Santos, Gil Nonato S., Danac, Alfonso C., O-Chemistry III, 2009,pp. 105-106

Mortimer Charles E., Chemistry 6th Edition, pp.317-320

Hagad, Hilda R., Phoenix Next Century Chemistry, 2003, pp. 240-241

192
NOTE: Practice personal hygiene protocols at all times
ANSWER KEY

ACTIVITY 1

181°
118.1°
100°
80.2°
78.4°
43°

17°
0° 5.5°
0° 0°

-114.7°

ACTIVITY 2

Q1. The boiling point and freezing point of the different solutions increased. Q2.
The boiling point temperatures are greater in Concentration B than in
Concentration A.

Q3. The freezing temperatures are lower in Concentration B than in Concentration


A.

Q4. The boiling temperature of the solutions are higher than that of a pure water
since the addition of a solute affects the boiling point and freezing point
temperatures of the solution.

Q5. Yes. Sodium chloride deviate more from the boiling and freezing temperatures
rather than sucrose and glycerol since sodium chloride produces two ions when
dissolved in water thus, increasing its mole concentration.

193
NOTE: Practice personal hygiene protocols at all times
ACTIVITY 3 ACTIVITY 4

1. 0.56°C 1. -18°C
2. 1.32°C 2. -7.96°C
3. 2.24°C 3. -3.35°C
4. 5.07°C 4. -1600°C
5. 17.34°C 5. -4.65°C

Prepared by:

LOVEJOICE L. AMBABAG
Tuao Vocational and Technical School-Main

194
NOTE: Practice personal hygiene protocols at all times
GENERAL CHEMISTRY 2

Name: ____________________________ Grade Level: _________


Date: _____________________________ Score: ______________

LEARNING ACTIVITY SHEET


Molar Mass from Colligative Property

Background Information for the Learners (BIL)

Calculating Molar Mass


Freezing point or boiling point data can be used to determine the molar mass
of an unknown solute. Since we know the relationship between a decrease in freezing
point and the concentration of solute, if we dissolve a known mass of our unknown
solute into a known amount of solvent, we can calculate the molar mass of the solute.
The kf or kb of the solvent must be known. We also need to know if the solute is an
electrolyte or a nonelectrolyte. If the solvent is an electrolyte, you would need to know
the number of ions that is produced when it dissociates.
For instance, the molar masses of Vitamin C and nicotine can be determined
from the temperatures at which water solutions of these substances begin to freeze.
This technique is specially helpful when identifying an unknown compound or
determining the formula of a newly isolated compound.
The molar masses of many organic substances that are not soluble in water,
like fat soluble vitamins and other natural products, can be found by dissolving known
amounts in a nonpolar solvent such as camphor. The temperature at which the
solution begins to freeze is noted. Then the molar mass of the solute is determined
from the rearrangement of the freezing point equation that we have studied previously.

195
NOTE: Practice personal hygiene protocols at all times
Steps in finding for the molar mass from colligative property data.
1. List the known quantities from the given problem.
2. Find the molality of the solution using the freezing point depression (Δtf) or
boiling point elevation (Δtb).
m = Δtf /kf
3. Use the molality equation to calculate the moles of solute.
mol solute = m x kg H2O
4. Divide the mass solute by the moles solute to determine the molar mass.
Molar mass = mass solute/moles solute

Sample Problem 1.
The Δtf of Camphor is 179.80°C and its kf is 39.7°C/m. When 0.2000g of a
compound X is added to 100g of camphor, its freezing piont drops to 179.29°C. what
is the molar mass of compound X?
a) Find the molality, m = Δtf /kf
m = Δtf /kf
m = (179.8°C – 179.29°C) / 39.7°C/m
m = 0.013mol/kg
b) Calculate the number of moles X
m = mol (X) / kg (camphor)
m = 0.013mol/kg x 0.1000kg
m = 0.0013mol
c) Find the molar mass
Molar mass = grams (X) / mol (X)
Molar mass = 0.2000g / 0.0013mol
Molar mass = 153.85g/mol

196
NOTE: Practice personal hygiene protocols at all times
Sample Problem 2.
A solution prepared by dissolving 0.300g of an unknown volatile solute in 30g
of carbon tetrachloride (CCl4) has a boiling point that is 0.392°C higher than that of
pure CCl4. What is the molecular weight of the solute? kb = 5.03°C/m

a) Find the molality, m = Δtb /kb


m = Δtb /kb
m = 0.392°C / 5.03°C/m
m = 0.078 mol/kg

b) Calculate the number of moles X


m = mol (X) / kg (CCl4)
m = 0.078mol/kg x 0.03kg
m = 0.0023mol

c) Find the molar mass


Molar mass = grams (X) / mol (X)
Molar mass = 0.300g / 0.0023mol
Molar mass = 130.43g/mol

Learning Competency
Calculate molar mass from colligative property data (STEM_GC11PPIIId-f-118)

197
NOTE: Practice personal hygiene protocols at all times
Activity 1: IT’S HUNT TIME!
Directions: Find and circle all of the listed words below that are hidden in the grid.
The words may be hidden horizontally, vertically, diagonally and backward.

COLLIGATIVE FREEZING POINT SOLUTE


MOLAR MASS BOILING POINT MOLE
MOLE SOLUTE ELEVATION
MASS SOLUTE DEPRESSION

198
NOTE: Practice personal hygiene protocols at all times
Activity 2: PROBLEM SOLVING
Directions: Read and analyze each problem carefully. Identify the known variables
and solve for the unknown variable following the steps in solving a problem. Label
each known variable with the correct unit of measure.

1. A solution is prepared by dissolving 38.7g of nonelectrolyte into 218g of water.


The freezing point of the solution is measured to be -5.53°C. Calculate the
molar mass of the solute. kf of water = 1.86°C/m

2. When 36g of a nonvolatile, molecular substance is dissolved in 100g of water,


the solution begins to freeze at -37.2°C. What is the molar mass of the solute?
kf of water = 1.86°C/m

199
NOTE: Practice personal hygiene protocols at all times
3. When 27.3g of nonvolatile molecular substance is dissolved in 300g of water,
the solution begins to freeze at -0.49°C. Find the molar mass of the solute. kf of
water = 1.86°C/m

4. When 2.55g of a substance is dissolved in 150g of water, the solution freezes


at -0.42°C. What would be the molar mass of the substance? kf of water =
1.86°C/m

5. Find the molar mass of an unknown nonvolatile molecular solute if the solution
begins to freeze at -0.186°C. The solution was prepared by dissolving 1.80g of
unknown solute in 1000g of water. kf of water = 1.86°C/m

200
NOTE: Practice personal hygiene protocols at all times
Activity 3: CALCULATING MOLAR MASS FROM BOILING POINT ELEVATION
Directions: Read and analyze each problem carefully. Identify the known variables
and solve for the unknown variable following the steps in solving a problem. Label
each known variable with the correct unit of measure.

1. 97.30g of a mystery compound is added to 500g of water, raising its boiling


point to 100.78°C. What is the molar mass of the mystery compound? kb of
water = 0.51°C/m

2. 5g of an organic solid is dissolved in 100g of benzene. The boiling temperature


of this solution is 82.42°C. The boiling temperature of pure benzene is 80.10°C;
kb = 2.53°C/m. What is the molecular weight of the unknown compound?

3. A 166.5mg sample of the compound eugenol was dissolved in 1.0g of


chloroform (kb =3.63°C/m), increasing the boiling point of chloroform by 3.68°C.
What is the molar mass of eugenol?

201
NOTE: Practice personal hygiene protocols at all times
4. 0.64g of adrenaline in 36g of CCl4 produces a boiling point elevation of 0.49°C.
What is the adrenaline’s molecular weight? kb=5.03°C/m

5. A 5g sample of a large biomolecule was dissolved in 16g of carbon


tetrachloride. The boiling point of this solution was determined to be 77.85°C.
Calculate the molar mass of the biomolecule. The boiling point constant for
carbon tetrachloride is 5.03°C/m and the boiling point of pure carbon
tetrachloride is 76.50°C.

202
NOTE: Practice personal hygiene protocols at all times
Reflection:

1.I learned that


___________________________________________________________________
___________________________________________________________________
___________________________________________________________________
_______________________________________________
______________________________________________________________

2.The thing I enjoyed the most in this topic


___________________________________________________________________
___________________________________________________________________
___________________________________________________________________
_______________________________________________
______________________________________________________________

3.I want to learn more about


___________________________________________________________________
___________________________________________________________________
___________________________________________________________________
_______________________________________________
______________________________________________________________

203
NOTE: Practice personal hygiene protocols at all times
References:

Santos, Gil Nonato S., Danac, Alfonso C., O-Chemistry III, 2009

Mortimer Charles E., Chemistry 6th Edition

Hagad, Hilda R., Phoenix Next Century Chemistry, 2003

flexbooks.ck12.org/cbook/ck-12-chemistry-flexbook

Chem Libre Text

204
NOTE: Practice personal hygiene protocols at all times
ANSWER KEY
Activity 1

Activity 2 Activity 3
1. 59.54g/mol 1. 127.19g/mol
2. 18g/mol 2. 54.5g/mol
1. 350g/mol 3. 164.2g/mol
2. 72.86g/mol 4. 180g/mol
3. 18g/mol 5. 1160g/mol

Prepared by:

LOVEJOICE L. AMBABAG
Tuao Vocational and Technical School-Main

205
NOTE: Practice personal hygiene protocols at all times
GENERAL CHEMISTRY 2

Name: ____________________________ Grade Level: _________


Date: _____________________________ Score: ______________

LEARNING ACTIVITY SHEET


Laboratory Solution Preparation

Background Information for the Learners (BIL)


Many of the reagents used in science are in the form of solutions which need
to be purchased or prepared. For many purposes, the exact value of concentration is
not critical; in other cases, the concentration of the solution and its method of
preparation must be as accurate as possible.

This learning material is designed for both the beginner and experienced
solution maker. It provides valuable information on the basic concepts of preparing
solutions and instructions for preparing most solutions required in the senior high
school science laboratory. Professional quality solutions are possible when high
quality and fresh chemicals and solvents are used, and meticulous procedures are
followed

Basic Concepts of Preparing Solutions

Molarity
The most common unit of solution concentration is molarity (M). The molarity of a
solution is defined as the number of moles of solute per one liter of solution. Note that
the unit of volume for molarity is liters, not milliliters or some other unit. Also note that
one liter of solution contains both the solute and the solvent. Molarity, therefore, is a
ratio between moles of solute and liters of solution. To prepare laboratory solutions,
usually a given volume and molarity are required. To determine molarity, the formula
weight or molar mass of the solute is needed. The following examples
illustrate the calculations for preparing solutions.

206
NOTE: Practice personal hygiene protocols at all times
If starting with a solid, use the following procedure:

1. Determine the mass in grams of one mole of solute, the molar


mass, MMs.
2. Decide volume of solution required, in liters, V.
3. Decide molarity of solution required, M.
4. Calculate grams of solute (gs) required using equation 1.eq. 1. gs = MMs x M
xV

Example:
Prepare 800 mL of 2 M sodium chloride.
(MMNaCl = 58.45 g/mol)
gNaCl = 58.45 g/mol x 2 mol/L x 0.8 L
gNaCl = 93.52 g NaCl

Dissolve 93.52 g of NaCl in about 400 mL of distilled water, then add more
water until final volume is 800 mL.

If starting with a solution or liquid reagent:

1. When diluting more concentrated solutions, decide what volume (V2) and
molarity (M2) the final solution should be. Volume can be expressed in liters or
milliliters.

2. Determine molarity (M1) of starting, more concentrated solution.

3. Calculate volume of starting solution (V1) required using equation


2. Note: V1 must be in the same units as V2.
M1V1= M2V2

207
NOTE: Practice personal hygiene protocols at all times
Example:
Prepare 100 mL of 1.0 M hydrochloric acid from concentrated (12.1 M)
hydrochloric acid.
M1V1= M2V2
(12.1 M)(V1) = (1.0 M)(100 mL)
V1 = 8.26 mL conc. HCl

Add 8.26 mL of concentrated HCl to about 50 mL of distilled water, stir,


then add water up to 100 mL.

Percent Solutions

Mass percent solutions are defined based on the grams of solute per 100 grams of
solution.

Example: 20 g of sodium chloride in 100 g of solution is a 20% by mass solution.

Volume percent solutions are defined as milliliters of solute per 100 mL of solution.

Example: 10 mL of ethyl alcohol plus 90 mL of H2O (making approx. 100 mL of


solution) is a 10% by volume solution.

Mass-volume percent solutions are also very common. These solutions are
indicated by w/v% and are defined as the grams of solute per 100 milliliters of
solution.

Example: 1 g of phenolphthalein in 100 mL of 95% ethylalcohol is a 1 w/v% solution.


Conversion Between Percent Solutions

You may wish to convert mass percent to volume percent or vice versa. If so, follow
this procedure:

208
NOTE: Practice personal hygiene protocols at all times
A 10% by mass solution of ethyl alcohol in water contains 10 g of ethyl alcohol and
90 g of water.

1. The formula for determining the volume of the component (ethyl alcohol in our
example) is:

mass of ethyl alcohol


Volume = ——————————
density of ethyl alcohol

2. Determine the volume of the total solution by dividing the mass of the solution by
the density of the solution.

3. Determine the percent by volume by dividing the volume of the component by the
volume of the solution.

Let’s solve 1, 2, and 3 above as follows:

1. Mass of ethyl alcohol = 10 g (given)


Density of ethyl alcohol = 0.794 g/mL

mass
Volume = ____________________
density
10 g
Volume of ethyl alcohol = ————— = 12.6 mL
0.794 g/mL

209
NOTE: Practice personal hygiene protocols at all times
2. Mass of solution = 100 g (given)
Density of solution (10% ethyl alcohol) = 0.983 g/mL
100 g
Volume of solution = ————— = 101.8 mL*
0.983 g/mL

3. Volume percent of solution


volume of ethyl alcohol 12.6
Percent = —————————— = ——— = 12.4%
total volume of solution 101.8

Reverse the procedure to convert volume percent to mass percent.

Calculating Molarity from Percent Solutions


To determine the molarity of a mass percent solution, the density of the solution is
required. Use the following procedure:

1. Determine the mass of solution by multiplying the volume of the solution by the
density of the solution.
mass = volume x density

2. Determine concentration in percent by mass of the solute in solution. Change to the


decimal equivalent.

3. Calculate the molar mass of the compound, MM.

4. Multiply mass (step 1) by mass % (step 2) and divide by molecular mass (step 3) to
find the number of moles present in the whole solution.

5. Divide the number of moles (step 4) by the volume in liters of the solution to find the
molarity of the solution.

210
NOTE: Practice personal hygiene protocols at all times
Example: Determine molarity of 37.2% hydrochloric acid (density 1.19 g/mL).

1. Mass of solution = 1,000 mL x 1.19 g/mL = 1,190 g


2. Mass % = 37.2 % = 0.372
3. Molar mass of hydrochloric acid = 36.4 g/mol

4. mass x mass % 1,190 g x 0.372


———————— = ———————— = 12.1 moles
MMHCl 36.4 g/mol

5. Molarity = moles/liters = 12.1 moles/1 liter = 12.1 M

Terms to Remember:
Buffer: A solution which tends to maintain a constant pH when excess acid or base is
added.
Concentrated: For some commonly used acids and bases, the maximum solubility
(at room temperature) in an aqueous solution or as a pure liquid.

Concentration: The relative amount of solute and solvent in a solution.

Hydrates: Compounds containing water chemically combined in a definite ratio.


Computations using formula weight must take the water molecules into account.

Miscible: The ability of two liquids to be completely soluble in one another.

Normality: A concentration unit (N); defined as the number of equivalents of solute


per liter of solution. (e.g., 1 M H2SO4 = 2 N H2SO4)

211
NOTE: Practice personal hygiene protocols at all times
Saturated Solution: A solution that contains the maximum amount of a particular
solute that will dissolve at that temperature.

Solute: The substance which is dissolved, or has gone into solution (typically a solid).

Solution: A uniform homogeneous mixture of two or more substances. The individual


substances may be present in varying amounts.

Solvent: The substance which does the dissolving (typically a liquid, such as water or
alcohol). Must be greater than 50% of the solution.

Standard Solution: A very precise solution, usually to 3–4 significant figures, used in
quantitative analysis or an analytical procedure.

Supersaturated Solution: A solution that contains more solute than equilibrium


conditions allow; it is unstable and the solute may precipitate upon slight agitation or
addition of a single crystal.

Molality: A concentration unit (m); defined as the number of moles of solute divided
by the number of kilograms of solvent.

Molar Mass: The mass of a mole of any element or compound.

Molarity: A concentration unit (M); defined as the number of moles of solute divided
by liters of solution.

212
NOTE: Practice personal hygiene protocols at all times
Learning Competency:
Describe laboratory procedures in determining concentration of solutions
(STEM_GC11PPIIId-f-119)

Activity 1: PROBLEM SOLVING ON SOLUTION PREPARATION


Directions: Perform the needed calculations in the following.

1.)23.5g of NaCl is dissolved in enough water to make .683 L of solution.

a) What Is the molarity (M) Of the solution?

b) How Many moles of NaCl Are contained in 0.0100 L Of the above NaCl solution?

c) What Volume (L) Of this NaCl Solution would contain 0.200 Moles of NaCl?

2) 12.5g Of glucose (C6H12O6) is dissolved in enough water to make 750.0 mL of


solution.

a) What Is the molarity (M) Of the solution?

b) How Many moles of glucose are contained in 237 mL of the above glucose
solution?

c) What Volume (L) Of this glucose solution would contain 0.079 Moles of glucose?

213
NOTE: Practice personal hygiene protocols at all times
Activity 2: PREPARING DISINFECTANT SOLUTIONS AT HOME

Before proceeding with the activity, please read the following information.

Chlorine is one of the best disinfectant for use against virus contamination because it
is cheap and easily available. Also, it has the ability to fragment the Nucleic Acid of
Polymerase Chain Reaction (www.lentech.com)

The best compound for the preparation of chlorine solutions for disinfection is
household bleach (also known by other names such as Chlorox®, Eau-de-Javel).
Household bleach is a solution of sodium hypochlorite which generally contains 5%
(50 g/liter or 50 000 ppm) available chlorine.

Note that:

1. different products may contain different concentrations of available chlorine


and the concentration should be checked before use;

2. household bleach preparations can lose some of their chlorine over time.
Use newly manufactured bleach if possible. If the bleach does not smell
strongly of chlorine it may not be satisfactory for the purpose and should not
be used;

3. thick bleach solutions should never be used for disinfection purposes (other
than in toilet bowls) as they contain potentially poisonous additives.

When preparing chlorine solutions for use note that:

1. chlorine solutions gradually lose strength, and freshly diluted solutions must
therefore be prepared daily;

214
NOTE: Practice personal hygiene protocols at all times
2. clear water should be used because organic matter destroys chlorine;

3. 1:10 bleach solution is caustic. Avoid direct contact with skin and eyes;

4. bleach solutions give off chlorine. Prepare them in a well ventilated area;

5. use plastic containers for mixing and storing bleach solutions as metal
containers arecorroded rapidly and also affect the bleach.

Two different dilutions of bleach are used for disinfection.

• 1:10 bleach solution, a strong disinfectant that is used to disinfect:


- Excreta
- Bodies
- Spills of blood/body fluids
- Vehicles and tires
- It is also used to prepare 1:100 bleach solution

• 1:100 bleach solution, which is used to disinfect:


- Surfaces
- Medical equipment
- Bedding
- Reusable protective clothing before it is laundered

* Also recommended for:


- Rinsing gloves between contact with different patients (if new gloves are not
available
- Rinsing gloves, aprons, boots before leaving a patient's room
- Disinfecting contaminated waste before disposal

215
NOTE: Practice personal hygiene protocols at all times
Supposed groceries ran out of stock for disinfectant products and you found
out that there is still 1L household bleach left in your home. Apply your learnings
from this material to perform the needed calculations in order for you to produce
your own home made disinfectant.

1. From 1 L of household bleach available,


a. How much volume of 1:10 house hold bleach solution will you be able to
prepare ?
b. How much volume of water is needed?
c. It has been mentioned earlier that household bleach is a solution of
sodium hypochlorite which generally contains 5% (50 g/liter or 50 000
ppm) available chlorine, how many percent of chlorine is present in this
solution?

2. Prepare 100 mL of 1:100 bleach solution from a 1:10 bleach solution


a. How much volume of bleach solution will you need?
b. How much volume of water is needed?
c. Determine the percentage of chlorine concentration from the solution

216
NOTE: Practice personal hygiene protocols at all times
Safety Reminder
Become a Label Fanatic!
• Do not use chemicals from unlabeled containers.
• Do not place labels on top of one another.
• Label chemicals clearly and permanently.

An unlabeled container will become tomorrow’s “Mystery


Substance.” A grease pencil or label can help eliminate a future problem
and a lot of expense.

You Make It—You Label It!


Minimum label requirements:
1. Identity of contents
2. Concentration
3. Your name
4. Date of preparation (if applicable)
5. Hazard alert (if applicable)

217
NOTE: Practice personal hygiene protocols at all times
Reflection:

1. I learned that
___________________________________________________________________
___________________________________________________________________
____________________________________________________________.

2. I enjoyed most on ___________________________________________________


___________________________________________________________________
_______________________________________________________________

3. I want to learn more on _______________________________________________


___________________________________________________________________
_______________________________________________________________.

218
NOTE: Practice personal hygiene protocols at all times
References:

CDC Centers for Disease Control and Prevention


https://www.cdc.gov/coronavirus/2019-nc0v/disinfecting-your-home

Chemistry LibreTexts
https://chemlibretexts.org/Courses/Prince_Georges_Community_College/Che
mistry_2000:_Chemistry_for_Engineeres_

219
NOTE: Practice personal hygiene protocols at all times
ANSWER KEY:

Activity 1:

1. a.) 0 .402 moles NaCl


b.) 0.00589 moles NaCl
c.) 0 .340 L of solution

2. a.) 0.0925 mole glucose/L=0.0925 M glucose


b.) 0.0219 mole glucose
c.) 0.85 L of solution

Activity 2:

1. a.) 10 L of 1:10 household bleach solution can be prepared from a liter of


household bleach
b.) 9L of water
c.) 0.5% chlorine

2. a.) 10mL of 1:10 bleach solution


b.) 90 mL of water
c.) 0.05% chlorine

Prepared by:

JENNIFER LOU B. ABUZO


Andarayan National High School

220
NOTE: Practice personal hygiene protocols at all times
GENERAL CHEMISTRY 2

Name: ____________________________ Grade Level: _________


Date: _____________________________ Score: ______________

LEARNING ACTIVITY SHEET


The First Law of Thermodynamics

Background Information for the Learners (BIL)

To understand and perform any sort of thermodynamic calculation, we must


first understand the fundamental laws and concepts of thermodynamics. For example,
work and heat are interrelated concepts. Heat is the transfer of thermal energy
between two bodies that are at different temperatures and is not equal to thermal
energy. Work is the force used to transfer energy between a system and its
surroundings and is needed to create heat and the transfer of thermal energy. Both
work and heat together allow systems to exchange energy. The relationship between
the two concepts can be analyzed through the topic of Thermodynamics, which is
the scientific study of the interaction of heat and other types of energy.

Definition: 1st Law of Thermodynamics

The First Law of Thermodynamics states that “energy cannot be created nor
destroyed. It can be transformed into another form but the total amount of energy
remains the same.

How energy is conserved is shown when you eat your meal. The chemical
energy in the food will be converted into mechanical energy that enables you to
perform your daily task. But not all the chemical energy from the food you take will be
transformed into mechanical energy. Some of it will be released from your body as
heat when you sweat or feel warm. In this law we talk of energy, work and heat.

221
NOTE: Practice personal hygiene protocols at all times
Figure on the left shows two
examples of energy being
transferred from one system to
another and from one form to
another. Humans can convert the
chemical energy in food like the
ice cream cone into kinetic energy
(the energy of movement to ride a
bicycle. Plants can convert light
energy from the sun into chemical
energy.

©courses.lumenlearning.com

How are these quantities related when we speak of thermodynamic systems?


In this Law, the energy added to a system is equal to the sum of increase in internal
energy and the external work done by the system.
In simpler term:
Heat Input= increase in internal energy + work output
Mathematically the relationship among the three is expressed as:
ΔU=q+w
Where:
ΔU is the total change in internal energy of a system,
q is the heat exchanged between a system and its surroundings, and
w is the work done by or on the system.
Example
A gas in a system has constant pressure. The surroundings around the system
lose 62 J of heat and does 474 J of work onto the system. What is the internal energy
of the system?

SOLUTION
To find internal energy, ΔU, we must consider the relationship between the
system and the surroundings. Since the First Law of Thermodynamics states that
energy is not created nor destroyed we know that anything lost by the surroundings is

222
NOTE: Practice personal hygiene protocols at all times
gained by the system. The surrounding area loses heat and does work onto the
system. Therefore, q and w are positive in the equation ΔU=q+w because the system
gains heat and gets work done on itself.
ΔU=(62J)+(474J) =536J

©physicsabout.com

The diagram above shows that the energy of a potato in an oven is equal to the heat
transferred to it. Also in the absence of any heat interaction, the energy change of a system is
equal to the net heat transfer.

The internal energy of a system would decrease if the system gives off heat or
does work. Therefore, internal energy of a system increases when the heat increases
(this would be done by adding heat into a system). The internal energy would also
increase if work were done onto a system. Any work or heat that goes into or out of a
system changes the internal energy. However, since energy is never created nor
destroyed (thus, the first law of thermodynamics), the change in internal energy always
equals zero. If energy is lost by the system, then it is absorbed by the surroundings. If
energy is absorbed into a system, then that energy was released by the surroundings:
ΔUsystem=−ΔUsurroundings(3)
Where:
ΔUsystem is the total internal energy in a system, and
ΔUsurroundings is the total energy of the surroundings.

Table 1: Visual example of the First Law of Thermodynamics


Sign of Sign of
Process
heat (q) Work (w)
Work done by the system N/A -
Work done onto the system N/A +

223
NOTE: Practice personal hygiene protocols at all times
Table 1: Visual example of the First Law of Thermodynamics
Sign of Sign of
Process
heat (q) Work (w)
Heat released from the system- exothermic
- N/A
(absorbed by surroundings)

The table symbolizes how energy lost by a system is gained by the


surroundings. The effects of different surroundings and changes on a system help
determine the increase or decrease of internal energy, heat and work.

Example:
A system has constant volume (ΔV=0) and the heat around the system increases by
45 J.
a. What is the sign for heat (q) for the system?
b. What is ΔU equal to?
c. What is the value of internal energy of the system in Joules?

SOLUTION
Since the system has constant volume (ΔV=0) the term -PΔV=0 and work is equal
to zero. Thus, in the equation ΔU=q+w w=0 and ΔU=q. The internal energy is equal to
the heat of the system. The surrounding heat increases, so the heat of the system
decreases because heat is not created nor destroyed. Therefore, heat is taken away
from the system making it exothermic and negative. The value of Internal Energy will
be the negative value of the heat absorbed by the surroundings.
a. negative (q<0)
b. ΔU=q + (-PΔV) = q+ 0 = q
c. ΔU = -45J

Learning Competency:
Explain the first law of thermodynamics (STEM_GC11TCIIIg-i-124)

224
NOTE: Practice personal hygiene protocols at all times
Activity 1: True or False
____1. The heat added to a system is used to increase the system’s internal energy

____2. The first Law of thermodynamics is a restatement of the conservation principle


of the Law of Conservation of Mass.

____3. Steam engines operate by boiling water to produce steam. The work done by
the steam is due to the expansion of water as it turns into the steam.

____4. Heat is the transfer of thermal energy between two bodies that are at equal
temperatures and are not equal to thermal energy.

____5. Work is needed to create heat and the transfer of thermal energy.

Activity 2: Analyze and solve the problem (5 points)

1. If 5J of energy is added to a system that does no external work, by how much


will the internal energy of that system be raised?

225
NOTE: Practice personal hygiene protocols at all times
2. A system has constant volume (ΔV=0) and the heat around the system
increases by 50 J.
a. What is the sign for heat (q) for the system?

b. What is ΔU equal to?

c. What is the value of internal energy of the system in Joules?

3. 300J of heat is added to a system and 250 J of work is added by the system.
What is the change in internal energy by the system?

4. 300 J of heat is added to a system and 250 J of work is done on the system.
What is the change in internal energy of the system?

5. 300 J of heat leaves the system and 250 J of work is done on the system. What
is the change in internal energy of the system?

226
NOTE: Practice personal hygiene protocols at all times
Reflection:
1. I learned that ______________________________________________________
___________________________________________________________________
__________________________________________________________________

2. I enjoyed most on___________________________________________________


___________________________________________________________________
__________________________________________________________________

3. I want to learn more on ______________________________________________


___________________________________________________________________
________________________________________________________________.

227
NOTE: Practice personal hygiene protocols at all times
References:
Chemical thermodynamics: the First Law.”
http://www.chem1.com/acad/webtext/energetics/CE-2.html#SEC1
Steve Lower’s Website
CC BY-SA.

“Laws of thermodynamics.”
http://en.wikipedia.org/wiki/Laws_of_thermodynamics
Wikipedia
CC BY-SA 3.0. Hi

228
NOTE: Practice personal hygiene protocols at all times
ANSWER KEY
Activity 1: True or False
1. T
2. F
3. T
4. F
5. T

Activity 2: Problem solving

1. 5 Joules

2. a.) negative (q<0)


b.) ΔU=q + (-PΔV) = q+ 0 = q
c. ΔU = -50 J
3. Internal energy increases by 50 J
4. Internal energy increases by 550 J
5.. Internal energy increases by 550 J

Prepared by:

JENIFER LOU ABUZO


Andarayan National High School

229
NOTE: Practice personal hygiene protocols at all times
GENERAL CHEMISTRY 2

Name: ____________________________ Grade Level: _________


Date: _____________________________ Score: ______________

LEARNING ACTIVITY SHEET


The Enthalpy of Reaction

Basic Information for the Learner (BIL)

We have stated that the change in energy (ΔU) is equal to the sum of the heat
produced and the work performed. Work done by an expanding gas is called pressure-
volume work, (or just PV work). Consider, for example, a reaction that produces a gas,
such as dissolving a piece of copper in concentrated nitric acid. The chemical equation
for this reaction is as follows:
Cu(s)+4HNO3(aq)→Cu(NO3)2(aq)+2H2O(l)+2NO2

If the reaction is carried out in a closed system that is maintained at constant


pressure by a movable piston, the piston will rise as nitrogen dioxide gas is formed
(Figure 5.4.1). The system is performing work by lifting the piston against the
downward force exerted by the atmosphere (i.e., atmospheric pressure). We find the
amount of PV work done by multiplying the external pressure P by the change in
volume caused by movement of the piston (ΔV). At a constant external pressure (here,
atmospheric pressure),
w=−PΔV

The negative sign associated with PV work done indicates that the system loses
energy when the volume increases. If the volume increases at constant pressure
(ΔV>0), the work done by the system is negative, indicating that a system has lost
energy by performing work on its surroundings. Conversely, if the volume decreases
(ΔV<0), the work done by the system is positive, which means that the surroundings
have performed work on the system, thereby increasing its energy.

230
NOTE: Practice personal hygiene protocols at all times
Work Performed by a Reaction Carried out at Constant Pressure

The figure above is an Example of Work Performed by a Reaction Carried Out at Constant
Pressure. (a) Initially, the system (a copper penny and concentrated nitric acid) is at
atmospheric pressure. (b) When the penny is added to the nitric acid, the volume of NO 2 gas
that is formed causes the piston to move upward to maintain the system at atmospheric
pressure. In doing so, the system is performing work on its surroundings .

The internal energy U of a system is the sum of the kinetic energy and potential
energy of all its components. It is the change in internal energy that produces heat
plus work. To measure the energy changes that occur in chemical reactions, chemists
usually use a related thermodynamic quantity called enthalpy (H) (from the Greek
enthalpein, meaning “to warm”). The enthalpy of a system is defined as the sum of its
internal energy U plus the product of its pressure P and volume V:
H=U+PV
Because internal energy, pressure, and volume are all state functions, enthalpy is also
a state function. So we can define a change in enthalpy (ΔH) accordingly

ΔH=Hfinal−Hinitial

If a chemical change occurs at constant pressure (i.e., for a given P, ΔP=0), the
change in enthalpy (ΔH) is
ΔH=Δ(U+PV)
=ΔU+ΔPV
=ΔU+PΔV

231
NOTE: Practice personal hygiene protocols at all times
Substituting q+w for ΔU (First Law of Thermodynamics) and −w for PΔV , we obtain
ΔH=ΔU+PΔV
=qp+w−w
=qp

The subscript p is used here to emphasize that this equation is true only for a process
that occurs at constant pressure. From Equation above we see that at constant
pressure the change in enthalpy, ΔH of the system, is equal to the heat gained or lost.

ΔH=Hfinal−Hinitia=qp

Just as with ΔU, because enthalpy is a state function, the magnitude of ΔH depends
on only the initial and final states of the system, not on the path taken. Most important,
the enthalpy change is the same even if the process does not occur at constant
pressure.
To find ΔH for a reaction, measure qp.
When we study energy changes in chemical reactions, the most important quantity is
usually the enthalpy of reaction (ΔHrxn), the change in enthalpy that occurs during a
reaction (such as the dissolution of a piece of copper in nitric acid). If heat flows from
a system to its surroundings, the enthalpy of the system decreases, so ΔHrxn is
negative. Conversely, if heat flows from the surroundings to a system, the enthalpy of
the system increases, so ΔHrxn
is positive. Thus:
• ΔHrxn <0 for an exothermic reaction, and
ΔHrxn >0 for an endothermic reaction.

232
NOTE: Practice personal hygiene protocols at all times
©ThoughtCo
In chemical reactions, bond breaking requires an input of energy and is therefore an
endothermic process, whereas bond making releases energy, which is an exothermic
process. The sign conventions for heat flow and enthalpy changes are summarized in
the following table:
Reaction
Q ΔHrxn
Type
<0 (heat flows from a system
Exothermic <0
to its surroundings)
> 0 (heat flows from the
>0
Endothermic surroundings to a system)

If ΔHrxn is negative, then the enthalpy of the products is less than the enthalpy of the reactants;
that is, an exothermic reaction is energetically downhill. Conversely, if ΔHrxn is positive, then
the enthalpy of the products is greater than the enthalpy of the reactants; thus, an endothermic
reaction is energetically uphill . Two important characteristics of enthalpy and changes in
enthalpy are summarized in the following discussion.

Bond breaking ALWAYS requires an input of energy; bond making ALWAYS releases
energy.

233
NOTE: Practice personal hygiene protocols at all times
The Enthalpy of Reaction

Figure: The Enthalpy of Reaction. Energy changes in chemical reactions are usually measured
as changes in enthalpy. (a) If heat flows from a system to its surroundings, the enthalpy of the
system decreases, ΔHrxn is negative, and the reaction is exothermic; it is energetically
downhill. (b) Conversely, if heat flows from the surroundings to a system, the enthalpy of the
system increases, ΔHrxn is positive, and the reaction is endothermic; it is energetically uphill.

Reversing a reaction or a process changes the sign of ΔH. Ice absorbs heat when
it melts (electrostatic interactions are broken), so liquid water must release heat when
it freezes (electrostatic interactions are formed):
heat+H2O(s)→H2O(l)ΔH>0
H2O(l)→H2O(s)+heatΔH<0
In both cases, the magnitude of the enthalpy change is the same; only the sign is
different.
For a clearer understanding of reaction, consider the following application:

Problem no.1 How many joules are required to heat 15.2 g of water at 35.0°C to
70.0°C?
In order to be able to determine how much heat is required to increase the temperature

of your sample of water from 35.0∘C to 70.0∘C , you need to know the value of

water's specific heat.


As you know, a substance's specific heat tells you how much heat is required to

increase the temperature of 1.0 g of that substance by 1 ∘C.

Water has a specific heat of about 4.18 J /g ∘C .

234
NOTE: Practice personal hygiene protocols at all times
This tells you that in order to increase the temperature of 1 g of water by 1 ∘C , you

need to provide it with 4.18 J of heat.

Therefore, in order to increase the temperature of 4.18 g of water by 1 ∘C , you would

need 4.18 times more heat than water's specific heat value.

Also, , in order to increase the temperature of 4.18 g of water by 4.18∘C , you will

need (4.18×4.18) times more heat than water's specific heat value.

The problem above wants you to increase 15.2 grams of water by 35.0∘C. This means

that you will be needing (15.2 x 35) times more heat than water’s specific heat value
Mathematically, this is expressed as:
q = (m)( c) ( ΔT)
where:
q is the heat absorbed
m is the mass of the sample
c is the specific heat of the substance
ΔT is the change in temperature (defined as final temperature minus initial
temperature
Substitute the equation with the given values:

q = (15.2 g) (4.18 J/ g ∘C) (70.0-35.0) ∘C

= 2,223.76 J

235
NOTE: Practice personal hygiene protocols at all times
Learning Competency:
Explain enthalpy of reaction (STEM_GC11TCIIIg-i-125)

Activity 1: IDENTIFY THE TYPE OF REACTION IN THE FOLLOWING SITUATIONS


1. Melting ice
2. Sublimation of naphthalene balls
3. Thermal decomposition
4. Dissolving ammonium chloride in water
5. Photosynthesis
6. Cellular Respiration
7. Water freezes into ice cube
8. Rusting Iron
9. Burning of candles
10. Setting of cements and concrete

ACTIVITY 2: Solve the following. 5 pts. Each

1. How many joules are needed to warm 25.5 grams of water from14O C to
22.5OC? (Note that the specific heat capacity of water is 4.18J/gO C)

2. Calculate the number of Joules released when 75 grams of water are


cooled from 100 OC to 27.5 OC?

3. If 1.13 x 10 4J of heat is added to a water sample and temperature rises


from 88.0 OC to its boiling point, what mass of water is in the sample?

236
NOTE: Practice personal hygiene protocols at all times
4. How much heat would be needed to to warm 250 grams of gold from 25 OC
to 100 OC if the specific heat capacity of gold is 0.128 J/g OC?

5. What is the temperature change if 1.386 x 10 3J is absorbed by 60.0 grams


of copper with specific heat capacity of 0.385 J/g OC?

237
NOTE: Practice personal hygiene protocols at all times
Reflection:

1. I learned that _______________________________________________________


___________________________________________________________________
_________________________________________________________

2. I enjoyed most on___________________________________________________


___________________________________________________________________
_____________________________________________________________.

3. I want to learn more on ______________________________________________


___________________________________________________________________
_____________________________________________________________.

238
NOTE: Practice personal hygiene protocols at all times
References:

Khan Academy
https://www.jhanacademy.org/science/chemistry/thermodynamics-
chemistry/enthalpy-chemistry-sal/a/bond-enthalpy-and-enthalpy-of-reaction

239
NOTE: Practice personal hygiene protocols at all times
Answer Key
Activity 1:
1. Endothermic
2. Endothermic
3. Endothermic
4. Endothermic
5. Endothermic
6. Exothermic
7. Exothermic
8. Exothermic
9. Exothermic
10. Exothermic

Activity 2:

1. 9.1 x 10 2 J
2. 2.27 x 10 4 J
3. 225 grams
4. 2.4 x 10 3 J
5. 60.0 OC

Prepared by:

JENIFER LOU ABUZO


Andarayan National High School

240
NOTE: Practice personal hygiene protocols at all times
GENERAL CHEMISTRY 2

Name: ____________________________ Grade Level: _________


Date: _____________________________ Score: ______________

LEARNING ACTIVITY SHEET


ENTHALPY

Background Information for the Learners (BIL)


Enthalpy, or enthalpy change, is
how much energy (in the form of
heat) has been transferred out or
taken in during a chemical reaction.

Many compounds cannot be directly


synthesized from their elements. In
some cases, the reaction proceeds
too slowly, or side reactions
produce substances other than the
desired compound. In these cases,
https://www.google.com/search?sa=G&hl=en&tbs=simg:CAQSxgIJWbgojuFMP4saugILELCMpwgaYgpgCAMS
KK0XvgvAF7gXygrNCpIMyQrjDM8egynfNJ8nnDa6KekhxDWrJ8oq2ycaMGtxFjinol4Ic_1zRZ2D5_1LRzZDD6 ∆H °f (enthalpy change) can be
1ySYvVvxrs88vIVXXNc6JRPXw2-l8cFxoxa_1lSAEDAsQjq7-CBoKCggIARIEY-
5fLwwLEJ3twQkasgEKJAoSYWxjb2hvbGljIGJldmVyYWdl2qWI9gMKCggvbS8wMTJtagoqChZzdGlsbCBsaW
ZlIHBob3RvZ3JhcGh52qWI9gMMCgovbS8wMjdfbnkzCiAKDWJvdHRsZWQgd2F0ZXLapYj2AwsKCS9tLzAy determined by an indirect approach,
NDI0NwofCgxnbGFzcyBib3R0bGXapYj2AwsKCS9tLzA4OW14cQobCghkZWNhbnRlctqliPYDCwoJL20vMDZr
N19mDA&q=Laboratory&tbm=isch&ved=2ahUKEwiqoZbk6PjqAhXDIqYKHdoZCfEQwg4oAHoECBAQJg&bi
w=1366&bih=646#imgrc=2K1buJOhOoQrMM which is based on Hess’s law of heat
summation, or simply Hess’s law,
named after the Swiss chemist
Germain Henri Hess.

Hess’s law is also known as "Hess's Law of Constant Heat Summation," can be
stated as: “states that the total enthalpy of a chemical reaction is the sum of the
enthalpy changes for the steps of the reaction”.

241
NOTE: Practice personal hygiene protocols at all times
Therefore, you can find enthalpy change by breaking a reaction into component
steps that have known enthalpy values. This example problem demonstrates
strategies for how to use Hess's Law to find the enthalpy change of a reaction using
enthalpy data from similar reactions.

Example 1:

What is the value of ΔH for the following reaction?


2CO(g) + O2(g) → 2 CO2(g) Eq.1

Given:
2C(s) + O2(g) →2CO(g); ΔHf = -221.0 kJ/mol Eq. 2
C(s) + O2(g) → CO2(g); ΔHf = -393.5 kJ/mol Eq. 3

Solution
Hess's Law says the total enthalpy change does not rely on the path taken
from beginning to end. Enthalpy can be calculated in one grand step or multiple
smaller steps.

Step 1: Manipulate given equations to most closely resemble equation of


interest.

To solve this type of problem, organize the given chemical reactions where the total
effect yields the reaction needed. There are a few rules that you must follow when
manipulating a reaction.
1. The reaction can be reversed. This will change the sign of ΔHf.
2. The reaction can be multiplied by a constant. The value of ΔHf must be
multiplied by the same constant.
We see resemblance of value from the equation of interest (Eq. 1) to equation 2

2CO(g) + O2(g) → 2 CO2(g) Eq 1

2C(s) + O2(g) →2CO(g) ; ΔHf = -221.0 kJ/mol Eq 2

242
NOTE: Practice personal hygiene protocols at all times
Flip equation 2 so that 2CO(g) will be on the reactant side as reflected by the equation
of interest. Applying rule #1, this will become:

2CO(g) → 2C(s) + O2(g) ; ΔHf = +221.0 kJ/mol Eq 4

Another resemblance from the equation of interest (Eq. 1) to equation 3.

2CO(g) + O2(g) → 2 CO2(g) Eq 1

C(s) + O2(g) → CO2(g); ΔHf = -393.5 kJ/mol Eq. 3

Both resemblance are on the product side but their coefficients are not the same,
therefore multiple the whole equation 3 to have a coefficient of 2. Applying rule #2,
this will become:

2C(s) + 2O2(g) → 2CO2(g); ΔHf = -787 kJ/mol Eq. 5

Step 2: Add new reactions together.

Add equation 4 and equation 5 together with their ΔHf This will become

. 2CO(g) → 2C(s) + O2(g) ; ΔHf = +221.0 kJ/mol Eq 4


2C(s) + 2O2(g) → 2CO2(g) ; ΔHf = -787 kJ/mol Eq. 5
______________________________________________________________________________

2CO(g) + 2C(s)+ 2O2(g) → 2CO2(g) + 2C(s) + O2(g) ΔHf = -566 kJ/mol

243
NOTE: Practice personal hygiene protocols at all times
Step 3: Cancel out any compounds that rethe same on both sides of the reaction
arrow.
Anything that are the same on both of the reaction arrows can be cancelled out.

2CO(g) → 2C(s) + O2(g) ; ΔHf = +221.0 kJ/mol Eq 4


2C(s) + 2O2(g) → 2CO2(g) ; ΔHf = -787 kJ/mol Eq. 5
______________________________________________________________________________

2CO(g) + 2C(s)+ 2O2(g) → 2CO2(g) + 2C(s) + O2(g) ΔHf = -566 kJ/mol

Step 4: Write the new equation and double check to make sure it matches the
equation of interest.

2CO(g) + O2(g) → 2CO2(g) : ΔHf = -566 kJ/mol

Therefore the value of ΔH for the following reaction 2CO(g) + O2(g) → 2 CO2(g)
is -566 kJ/mol

Learning Competency:
Calculate the change in enthalpy of a given reaction using Hess Law
(STEM_GC11TCIIIg-i-127)

244
NOTE: Practice personal hygiene protocols at all times
Activity 1: JUSTIFY ME
Directions: Explain your answer comprehensively. Limit your justification in
3-5 sentences. Refer to the scoring rubric below the questions.

1. All Hess's Law says is that if a reaction is the sum of two or more other
reactions, then the change in enthalpy of this reaction is going to be ?
Explain
_________________________________________________________
_________________________________________________________
_________________________________________________________
_________________________________________________________
_________________________________________________________
_________________________________________________________
__________________________________________
2. What does it mean to have a lower enthalpy? Does the energy be
released or absorbed? Explain.
_________________________________________________________
_________________________________________________________
_________________________________________________________
_________________________________________________________
_________________________________________________________
_________________________________________________________
__________________________________________

Criteria Point
Content (topic or idea) 4
Clarity and Organization 3
of thoughts
Grammar 3

245
NOTE: Practice personal hygiene protocols at all times
Activity 2: FACT OR BLUFF
Directions: Write FACT if the statement is correct and BLUFF if the statement is
wrong.

____________1. The enthalpy change accompanying a chemical change is


independent of the route by which the chemical change occurs.
____________2. Hess's Law is saying that if you convert reactants A into products
B, the overall enthalpy change will be exactly different whether you do it in
one step or two steps or however many steps.
____________3. The enthalpy change is the heat evolved or absorbed during a
reaction happening at constant pressure.
____________4. All steps in enthalpy change have to proceed at the same
temperature and the equations for the individual steps must balance out
____________5. Enthalpy is a state function that the magnitude of ΔH does not
depend on the path

Activity 3: LET’S CALCULATE!


Directions: Solve the following problems completely.
1. Calculate the heat of hydrogenation of ethane (C2H6) given the following
thermochemical equations:
a. 2C (graphite) + 3 H2 (g) —> C2H6 (g) ΔHfo = - 84.5 kJ/mol
b. 2C (graphite) + 2 H2 (g) —> C2H4 (g) ΔHfo = 52.3 kJ/mol

2. Find the ΔH for the reaction below, given the following reactions and subsequent
ΔH values: 2C2(g) + H2O(g) → C2H2(g) + ⁵/₂ O2(g)
C2H2(g) + 2H2(g) → C2H6(g) ΔH = –94.5 kJ
H2O(g) → H2(g) + ½ O2 (g) ΔH =71.2 kJ
C2H6(g) + ⁷/₂ O2(g) → 2 CO2(g) + 3 H2O(g) ΔH = –283 Kj

246
NOTE: Practice personal hygiene protocols at all times
Activity 4: THINK OUTSIDE THE BOX!
Directions: Study and analyze the given situstions, think of a possible outcome or
consequences of the situations basing from the concept of enthalpy and Hess law.
SITUATION CONSEQUENCE/OUTCOME
A reaction is allowed to take place in an
insulated container containing 100 mL
of water. If the reaction is exothermic,
what happens to the temperature of
water?
The thermochemical equation showing
the formation of ammonia, NH3 from its
elements is:
N2 (g) + 3 H2 (g) —> 2 NH3 (g) ΔH = -
92 kJ . This equation shows that 92 kJ
of heat is __.
If heat is released by a chemical
system, an equal amount of heat will
be?
At constant pressure, enthalpy is?

If ΔH is positive then it is?

247
NOTE: Practice personal hygiene protocols at all times
Activity 5: ANALOGY
Directions: Cite an example or analogy that best interpret Enthalpy or Hess Law.

EXAMPLE: Suppose you go from the first floor to the sixth floor of
a building by elevator. The gain in your gravitational potential energy
(which corresponds to the enthalpy change for the overall process) is
the same whether you go directly there or stop at each floor on your
way up (breaking the trip into a series of steps).
__________________________________________________________________
__________________________________________________________________
__________________________________________________________________
__________________________________________________________________
__________________________________________________________________
__________________________________________________________________
__________________________________________________________________
__________________________________________________________________
__________________________________________________________________
_______________________________________________________

Reflection:

1. I learned that _______________________________________________________


___________________________________________________________________
_________________________________________________________

2. I enjoyed most on ___________________________________________________


___________________________________________________________________
______________________________________________________________.

3. I want to learn more on _______________________________________________


___________________________________________________________________
___________________________________________________________________.

248
NOTE: Practice personal hygiene protocols at all times
References:

1. Kto12 Curriculum guide in Earth and Life Science


2. Chemistry 10TH Edition by Raymond Chang. Chemistry 10th edition. New York
City, NY 10020. McGraw-Hill Companies, Inc., pp. 239-245
4. https://www.khanacademy.org/science/chemistry/thermodynamics-
chemistry/enthalpy-chemistry-sal/v/hess-s-law-example
5.https://chem.libretexts.org/Bookshelves/Physical_and_Theoretical_Chemistry_Text
book_Maps/Supplemental_Modules_(Physical_and_Theoretical_Chemistry)/Thermo
dynamics/Thermodynamic_Cycles/Hess's_Law

249
NOTE: Practice personal hygiene protocols at all times
ANSWER KEY:
ACTIVITY 1

1. The sum of the change in enthalpies of those reactions


2. Because there's now less energy in the system which means Energy was
released.

ACTIVITY 2

1.FACT 2. BLUFF 3. FACT 4. FACT 5. FACT


ACTIVTY 3
1.
The desired equation is:
C2H4 (g) + H2 (g) —> C2H6 (g) ΔH =
Applying Hess’s Law,
Reverse equation (b) and change the sign of ΔH:
C2H4 (g) —> 2 C (graphite) + 2 H2 (g) ΔH = - 52.3 kJ/mol
Add equation (a):
2 C(graphite) + 3 H2 (g) —> C2H6 (g) ΔH = - 84.5 kJ/mol
Overall equation:
C2H4 (g) + H2 (g) —> C2H6 (g) ΔH = - 136.8 kJ/mol
2. 235. KJ
ACTIVITY 4
SITUATION CONSEQUENCE/OUTCOME
A reaction is allowed to take place in an The temperature of the water goes
insulated container containing 100 mL down.
of water. If the reaction is exothermic,
what happens to the temperature of
water?
The thermochemical equation showing Lost to the surroundings when 2 moles
the formation of ammonia, NH3 from its of ammonia is formed.
elements is:

250
NOTE: Practice personal hygiene protocols at all times
N2 (g) + 3 H2 (g) —> 2 NH3 (g) ΔH = -
92 kJ . This equation shows that 92 kJ
of heat is __.
If heat is released by a chemical Absorbed by the surroundings
system, an equal amount of heat will
be?
At constant pressure, enthalpy is? The heat of reaction is equal to the
enthalpy change of the system.
If ΔH is positive then it is? The reaction is endothermic.

Prepared by:

CATHERINE PASCUAL
Baggao National High School

251
NOTE: Practice personal hygiene protocols at all times
GENERAL CHEMISTRY 2

Name: ____________________________ Grade Level: _________


Date: _____________________________ Score: ______________

LEARNING ACTIVITY SHEET


FACTORS AFFECTING RATE OF REACTION

Background Information for the Learners


While rust may take years to develop
on a car, logs burning in a fire can be
turned to ash in a matter of hours. That
only means different factors can affect
the rate of a chemical reaction and able
to predict how to speed up or slow
down a process.
Chemical kinetics is the area
of chemistry concerned with the
speeds, or rates, at which a chemical
reaction occurs. The word “kinetic”
https://www.google.com/search?sa=G&hl=en&tbs=simg:CAQStAIJerAAvekIGY8aqAILELCMpwgaYgpgC
AMSKMIQhRv7Du0bnAWND8UQwxCpBcwb6SHfNNsnxDWcNqsnnyeDKdQhpTYaMH1Puci251g6sQp
znTw1DMGOqtLgADw1Xf8dbVYJ-YEuDHU1THE7MHGxGEMgKRYplyAEDAsQjq7-
suggests movement or change; kinetic
CBoKCggIARIENZQQCAwLEJ3twQkaoAEKFgoEbWlsa9qliPYDCgoIL20vMDR6cHYKKgoWc3RpbGw
gbGlmZSBwaG90b2dyYXBoedqliPYDDAoKL20vMDI3X255MwoXCgVzbW9rZdqliPYDCgoIL20vMDZ
xNDAKHwoMZ2xhc3MgYm90dGxl2qWI9gMLCgkvbS8wODlteHEKIAoOZ3JhcGhpYyBkZXNpZ27apYj
energy as the energy available
2AwoKCC9tLzAzYzMxDA&q=chemical+reactions&tbm=isch&ved=2ahUKEwiu8diMsPnqAhVyJaYKHc8
5BFsQwg4oAHoECBEQJg&biw=1366&bih=695
because of the motion of an object.

THE CHEMICAL NATURE OF THE REACTING SUBSTANCES


The rate of a reaction depends on the nature of the participating substances.
Reactions that appear similar may have different rates under the same conditions,
depending on the identity of the reactants. For example, when small pieces of the
metals iron and sodium are exposed to air, the sodium reacts completely with air
overnight, whereas the iron is barely affected. The active metals calcium and sodium
both react with water to form hydrogen gas reacts at a moderate rate, whereas sodium
reacts so rapidly that the reaction is and a base. Yet calcium almost explosive.

252
NOTE: Practice personal hygiene protocols at all times
THE STATE OF SUBDIVISION OF THE REACTANTS
Except for substances in the gaseous state or in solution, reactions occur at the
boundary, or interface, between two phases. Hence, the rate of a reaction between
two phases depends to a great extent on the surface contact between them. A finely
divided solid has more surface area available for reaction than does one large piece
of the same substance. Thus a liquid will react more rapidly with a finely divided solid
than with a large piece of the same solid. For example, large pieces of iron react slowly
with acids; finely divided iron reacts much more rapidly. Large pieces of wood smolder,
smaller pieces burn rapidly, and saw dust burns explosively.

TEMPERATURE OF THE REACTANTS


Chemical reactions typically occur faster at higher temperatures. Food can spoil
quickly when left on the kitchen counter. However, the lower temperature inside of a
refrigerator slows that process so that the same food remains fresh for days. We use
a burner or a hot plate in the laboratory to increase the speed of reactions that proceed
slowly at ordinary temperatures

CONCENTRATIONS OF THE REACTANTS


The rates of many reactions depend on the concentrations of the reactants.
Rates usually increase when the concentration of one or more of the reactants
increases. For example, calcium carbonate (CaCO3) deteriorates as a result of its
reaction with the pollutant sulfur dioxide. The rate of this reaction depends on the
amount of sulfur dioxide in the air. An acidic oxide, sulfur dioxide combines with water
vapor in the air to produce sulfurous acid in the following reaction:
SO2(g)+H2O(g)⟶H2SO3(aq)SO2(g)+H2O(g)⟶H2SO3(aq)
Calcium carbonate reacts with sulfurous acid as follows:
CaCO3(s)+H2SO3(aq)⟶CaSO3(aq)+CO2(g)+H2O(l)CaCO3(s)+H2SO3(aq)⟶CaSO3(a
q)+CO2(g)+H2O(l)
In a polluted atmosphere where the concentration of sulfur dioxide is high, calcium
carbonate deteriorates more rapidly than in less polluted air. Similarly, phosphorus
burns much more rapidly in an atmosphere of pure oxygen than in air, which is only
about 20% oxygen.

253
NOTE: Practice personal hygiene protocols at all times
THE PRESENCE OF A CATALYST
Hydrogen peroxide solutions foam when poured onto an open wound because
substances in the exposed tissues act as catalysts, increasing the rate of hydrogen
peroxide’s decomposition. However, in the absence of these catalysts (for example,
in the bottle in the medicine cabinet) complete decomposition can take months.
A catalyst is a substance that increases the rate of a chemical reaction by lowering
the activation energy without itself being consumed by the reaction. Activation
energy is the minimum amount of energy required for a chemical reaction to proceed
in the forward direction. A catalyst increases the reaction rate by providing an
alternative pathway or mechanism for the reaction to follow

PHYSICAL STATE OF THE REACTANTS AND SURFACE AREA


If reactant molecules exist in different phases, as in a heterogeneous mixture,
the rate of reaction will be limited by the surface area of the phases that are in contact.
For example, if a solid metal reactant and gas reactant are mixed, only the molecules
present on the surface of the metal are able to collide with the gas molecules.
Therefore, increasing the surface area of the metal by pounding it flat or cutting it into
many pieces will increase its reaction rate.

Learning Competency and Code: Describe how various factors influence the rate
of a reaction (STEM_GC11CKIIIi-j-130)

254
NOTE: Practice personal hygiene protocols at all times
Activity 1: MATCH ME
Directions: Match the terms in column A with the best descriptors in colum B. Each
descriptor may be used only once.
A B
1. catalyst a. a measure of how much area of an object is exposed
2. temperature b. the amount of substance dissolved in a given volume of
solution
3. surface area c. a measure of the average kinetic energy of all the
particles in a sample of matter
4. concentration d. a substance that speeds up the rate of a chemical
reaction without being used up itself or changed
5. rate of reaction e. a measure of how quickly products form, or given
amounts of reactants react, in a chemical reaction

Activity 2: YOU COMPLETE ME!


Directions: Complete the following table by indicating whether each of the following
scenarios would either increase or decrease the rate of reaction.
SCENARIO INCREASE OR DECREASE
Adding heat
Removing heat
Adding a catalyst
Diluting a solution
Removing an enzyme (catalyst)
Lowering the temperature
Increasing the temperature
Decreasing the surface area
Increasing the concentration of a
solution
Breaking a reactant down into smaller
pieces

255
NOTE: Practice personal hygiene protocols at all times
ACTIVITY 3: Choose the Right One
Directions: Fill in the blanks with words that correspond to the statements about the
rates of reaction. Choose the words from the box below.
1. A freshly exposed surface of metallic sodium tarnishes almost instantly if exposed
to air and moisture, while iron will slowly turn to rust under the same conditions. In
these two situations, the __________________ refers to how quickly or slowly
reactants turn into products.
2. Adding _______________will increase the rate of reaction because this causes the
particles of the reactants to move more quickly, resulting in more collisions and more
___________.
3. Removing heat will lower the ____________________, causing the particles of the
reactants to slow down, resulting in less frequent collisions.
4. ___________________ refers to how much solute is dissolved in a solution. If there
is a greater concentration of reactant particles present, there is a greater chance that
__________________ among them will occur. More collisions mean a higher rate of
reaction.
5. A concentrated acid solution will react more quickly than a _______________ acid
solution because there are more molecules present, increasing the chance of
collisions.
6. Grains of sugar have a greater ______________________ than a solid cube of
sugar of the same mass, and therefore will dissolve quicker in water.
7. A ______________________, for example an enzyme, is used to speed up a
chemical reaction but is not used up in the reaction itself.
8. __________is the area of chemistry concerned with the speeds, or rates, at which
a chemical reaction occurs.
9. Smaller particle size allows for a _________ surface area to be exposed for the
reaction.
10. ________ is the minimum amount of energy needed to start a reaction.

CHEMICAL KINETICS SURFACE AREA TEMPERATURE

DILUTE COLLISION HEAT

ACTIVATION ENERGY CONCENTRATION ENERGY


ACTIVITY
SMALLER 4: Choose the Greatest!
CATALYST RATE OF REACTION

256
NOTE: Practice personal hygiene protocols at all times
Directions: Complete the following table by indicating which factor would have the
greatest impact on the rate of reaction. Choose from concentration, temperature,
surface area or catalyst. The first one has been done for you.

Scenario Factor that has the greatest impact on


the rate of reaction.

Blowing air on a campfire to help get it Concentration


going.

Raw carrots are cut into thin slices for


cooking.

Protein is broken down in the stomach by


the enzyme pepsin.

Woolly Mammoth is found, perfectly


preserved, near the Arctic circle.

More bubbles appear when a


concentrated solution of hydrochloric acid
is added to a magnesium strip than when
a dilute solution of the acid is added.

Exhaust from a car engine passes


through a catalytic converter changing
most of the poisonous nitrogen oxides
into nitrogen gas and oxygen gas.

A dust explosion occurs in a saw mill.

Reactions that appear similar may have


different rates under the same conditions,
depending on the identity of the reactants

Liquid will react more rapidly with a finely


divided solid than with a large piece of
the same solid.

257
NOTE: Practice personal hygiene protocols at all times
ACTIVITY 5: Choose X without Asking Y!
Directions: Identify which situation (X or Y) would have a higher reaction rate. Then
state the factor that affected the rate of reaction in each situation (concentration,
surface area, catalyst, temperature)
Situation X Situation Y Situation with Factor
a higher affecting
reaction rate the rate of
(X or Y) reaction
1 g of sugar (cubes) 1 g of sugar (grains)

https://www.google.com/ https://www.google.com/
search?q=cube+sugar&t search?q=sugar+grain&t
bm=isch&chips=q:cube+ bm=isch&ved=2ahUKEw
sugar,g_1:transparent:O iFva270PnqAhVMBaYK
nAchH4Y77M%3D&hl=e HTLtD8kQ2-
n&sa=X&ved=2ahUKEwj cCegQIABAA&oq=sugar
KueG20PnqAhVtyIsBHflr +grain&gs_lcp=CgNpbW
B5gQ4lYoAXoECAEQF cQA1Ds5XlYj4J6YISGe
w&biw=1349&bih=667 mgAcAB4AIABAIgBAJIB
AJgBAKABAaoBC2d3cy
13aXotaW1nwAEB&sclie
nt=img&ei=XC0lX4WfKc
yKmAWy2r_IDA&bih=66
7&biw=1349&hl=en#imgr
c=ZKybO9T7BaAvzM

258
NOTE: Practice personal hygiene protocols at all times
5O ºC O ºC

https://www.google.com/
https://www.google.com/
search?q=thermometer+i
search?q=thermometer+i
n+beaker&tbm=isch&ved
n+beaker&tbm=isch&ved
=2ahUKEwjVuZjY2vnqA
=2ahUKEwjVuZjY2vnqA
hVNdJQKHfIZACUQ2-
hVNdJQKHfIZACUQ2-
cCegQIABAA&oq=therm
cCegQIABAA&oq=therm
ometer+&gs_lcp=CgNpb
ometer+&gs_lcp=CgNpb
WcQARgAMgQIIxAnMg
WcQARgAMgQIIxAnMg
UIABCxAzICCAAyAggA
UIABCxAzICCAAyAggA
MgIIADICCAAyAggAMgI
MgIIADICCAAyAggAMgI
IADICCAAyAggAOgcIIx
IADICCAAyAggAOgcIIx
DqAhAnOgcIABCxAxBD
DqAhAnOgcIABCxAxBD
OgQIABBDUMCJCljWvA
OgQIABBDUMCJCljWvA
pgws8KaAFwAHgAgAH
pgws8KaAFwAHgAgAH
ZBYgB8RmSAQsyLTQu
ZBYgB8RmSAQsyLTQu
MC4xLjIuMZgBAKABAa
MC4xLjIuMZgBAKABAa
oBC2d3cy13aXotaW1ns
oBC2d3cy13aXotaW1ns
AEKwAEB&sclient=img&
AEKwAEB&sclient=img&
ei=FTglX5XbHs3o0QTys
ei=FTglX5XbHs3o0QTys
4CoAg&bih=667&biw=13
4CoAg&bih=667&biw=13
49&hl=en#imgrc=0AeLq
49&hl=en#imgrc=92ZGzj
9r71K9U-M
G1Uu_T8M

259
NOTE: Practice personal hygiene protocols at all times
Low number of High number of

particles = few particles = more


collisions collisions
https://www.google.com/ https://www.google.com/
search?q=collision+theor search?q=collision+theor
y&tbm=isch&ved=2ahUK y&tbm=isch&ved=2ahUK
EwjWjf2l3PnqAhWKSJQ EwjWjf2l3PnqAhWKSJQ
KHQXDCyQQ2- KHQXDCyQQ2-
cCegQIABAA&oq=colli& cCegQIABAA&oq=colli&
gs_lcp=CgNpbWcQARg gs_lcp=CgNpbWcQARg
AMgQIABBDMgQIABBD AMgQIABBDMgQIABBD
MgUIABCxAzIFCAAQsQ MgUIABCxAzIFCAAQsQ
MyAggAMgIIADICCAAy MyAggAMgIIADICCAAy
AggAMgIIADICCAA6BA AggAMgIIADICCAA6BA
gjECc6BwgjEOoCECc6B gjECc6BwgjEOoCECc6B
wgAELEDEENQi58CWK wgAELEDEENQi58CWK
_HAmCM2wJoA3AAeAS _HAmCM2wJoA3AAeAS
AAc8DiAHzEJIBCTAuO AAc8DiAHzEJIBCTAuO
C4wLjEuMZgBAKABAao C4wLjEuMZgBAKABAao
BC2d3cy13aXotaW1nsA BC2d3cy13aXotaW1nsA
EKwAEB&sclient=img&ei EKwAEB&sclient=img&ei
=xTklX5aXBIqR0QSFhq- =xTklX5aXBIqR0QSFhq-
gAg&bih=667&biw=1349 gAg&bih=667&biw=1349
&hl=en&hl=en#imgrc=XX &hl=en&hl=en#imgrc=XX
DU4FH5VqKPyM DU4FH5VqKPyM

Enzyme added No enzyme added

260
NOTE: Practice personal hygiene protocols at all times
https://www.google.com/
search?q=ENZYME+TE
ST+TUBE&tbm=isch&ve
d=2ahUKEwiI8teF3vnqA
hXpxosBHR71Ap8Q2-
cCegQIABAA&oq=ENZY
ME+TEST+TUBE&gs_lc
https://www.google.com/
p=CgNpbWcQA1DBowF
search?q=ENZYME+TE
Yw8gBYMbPAWgAcAB4
ST+TUBE&tbm=isch&ve
AIABAIgBAJIBAJgBAKA
d=2ahUKEwiI8teF3vnqA
BAaoBC2d3cy13aXotaW
hXpxosBHR71Ap8Q2-
1nwAEB&sclient=img&ei
cCegQIABAA&oq=ENZY
=mjslX4i9DemNr7wPnuq
ME+TEST+TUBE&gs_lc
L-
p=CgNpbWcQA1DBowF
Ak&bih=667&biw=1366#i
Yw8gBYMbPAWgAcAB4
mgrc=FCzmLmT-
AIABAIgBAJIBAJgBAKA
uR4u4M&imgdii=dM990u
BAaoBC2d3cy13aXotaW
d50148OM
1nwAEB&sclient=img&ei
=mjslX4i9DemNr7wPnuq
L-
Ak&bih=667&biw=1366#i
mgrc=FCzmLmT-
uR4u4M&imgdii=dM990u
d50148OM
Twigs Logs

261
NOTE: Practice personal hygiene protocols at all times
https://www.google.com/ https://www.googl
search?q=TWIGS&sxsrf e.com/search?q=LOGS&
=ALeKk00ssvMW- sxsrf=ALeKk01CsYUiIzq
fizVqcwfuM7MxV3yEww AOxUEmnAi3wbsoqBov
bg:1596275744097&sour g:1596275864443&sourc
ce=lnms&tbm=isch&sa= e=lnms&tbm=isch&sa=X
X&ved=2ahUKEwjIxMLF &ved=2ahUKEwiF8vP-
3vnqAhUuGqYKHZoWDt 3vnqAhU2y4sBHRH7C8
QQ_AUoAXoECA4QAw EQ_AUoAXoECBIQAw&
&biw=1366&bih=667#im biw=1366&bih=667#imgr
grc=dl26dw8WZ82l8M&i c=XhZaKfJS-KgPxM
mgdii=QSmfuAcBcempe
M

Reflection:
1. I learned that _________________________________________________
___________________________________________________________________
_________________________________________________________

2. I enjoyed most on ______________________________________________


___________________________________________________________________
___________________________________________________________________.

3. I want to learn more on __________________________________________


___________________________________________________________________
___________________________________________________________________.

262
NOTE: Practice personal hygiene protocols at all times
References:
1. Curriculum Guide in General Chemistry 2
2. Chemistry 10TH Edition by Raymond Chang. Chemistry 10th edition. New York
City, NY 10020. McGraw-Hill Companies, Inc., pp. 558-559

3.https://opentextbc.ca/chemistry/chapter/12-2-factors-affecting-reaction-
rates/https://opentextbc.ca/introductorychemistry/chapter/factors-that-affect-the-rate-
of-reactions-2/

263
NOTE: Practice personal hygiene protocols at all times
ANSWER KEY
ACTIVITY 1
1. d 2. c 3. a 4. b 5. e

ACTIVITY 2
1. Increase 6. Decrease
2. Decrease 7. Increase
3. Increase 8. Decrease
4. Decrease 9. Increase
5. Decrease 10. Increase
ACTIVITY 3
1. Rate of Reaction 6. Surface Area
2. Heat, Energy 7. Catalyst
3.Temperature 8. Chemical Kinetics
4.Concentration, Collision 9. Smaller
5. Dilute 10. Activation Energy
ACTIVITY 4
1. Concentration 6. Catalyst
2. Surface Area 7. Concentration
3. Catalyst 8. Nature of Reactants
4. Temperature 9. State of Subdivision of the reactants
5. Concentration
ACTIVITY 5
a. Y - Concentration b. X - Temperature c. Y -
Concentration
d. X – Catalyst e. X - Surface Area

Prepared by:

CATHERINE PASCUAL
Baggao National High School

264
NOTE: Practice personal hygiene protocols at all times
GENERAL CHEMISTRY 2
Name: ____________________________ Grade Level: _________
Date: _____________________________ Score: ______________

LEARNING ACTIVITY SHEET

CHEMICAL KINEMATICS: REACTION RATE AND THE RATE LAW

Background Information for the Learners (BIL)

Every action has a reaction. So, everything we do there will always be a


reaction.
Rusting of iron is an example of a slow reaction over the years. The iron reacts
with the oxygen resulting to the corrosion or rusting of iron. The oxygen is a very good
oxidizing agent whereas the iron is the reducing agent. The rusting of iron can be
quickly done when salt and acid are present in the surrounding. The salt accelerates
the rusting process by lowering the electrical resistance of water. The easier the
electrons flow from iron to oxygen, the quicker the iron rust. The speed or rate at which
the chemical reactions occur is called Chemical Kinetics. Let’s dig deeper:

Reaction Rate
Reaction rate in chemistry is the speed at which a chemical reaction proceeds.
It is often expressed in terms of either the concentration (amount per unit volume) of
a product that is formed in a unit or the concentration of a reactant that is consumed
in a unit of time. It can also be defined as the amount of reactants consumed or
products formed in a unit of time.
The rate of a chemical reaction tells how fast a given amount of a reactant or
product changes with time. It can be expressed either as the disappearance of a
reactant or the appearance of the product.
A→B (1)
∆ [𝐴]
Rate = - ∆A = change in concentration of A over time period ∆t
∆𝑡
∆ [𝐵]
Rate = ∆B = change in concentration of B over time period ∆t
∆𝑡

Because [A] decreases with time, ∆[A] is negative

265
NOTE: Practice personal hygiene protocols at all times
To generalized, for the reaction
aA + bB → cC + dD

Reactants (decrease) Products (increase)


https://users.cs.duke.edu/~reif/courses/molcomplectures/Kinetics/KineticsOve
rview/KineticsOverview.ppt
The formula above describes that the reactants decreases as the products
increases.

Types of Rates
1. Initial Rate is the rate measured at the beginning of the reaction, which is
dependent on the initial concentrations of reactants.
Example of the initial rate is the figure 1 below. As the figure indicated, it is
found on the left side in which it is the beginning of the reaction and the initial
concentrations of reactants.

Figure 1 Initial Rate

https://www.google.com/search?q=initial+rate+of+reaction&sxsrf=ALeKk00eU
Piw0F4g9Fn0d0ih1Tt5Udt0Xw:1598569396012&source=lnms&tbm=isch&sa=
X&ved=2ahUKEwjO0f2Hv7zrAhWryIsBHWrHCowQ_AUoAXoECAwQAw&biw
=1366&bih=657#imgrc=ywQ1JdChakRFxM

266
NOTE: Practice personal hygiene protocols at all times
2. Instantaneous Rates are the rates measured at any point during the reaction.
Example:
Experimental Data for the Reaction Between Phenolphthalein and Base
Concentration of Phenolphthalein (M) Time (s)
0.0050 0.00
0.0045 10.5
0.0040 22.3
0.0035 35.7
0.0030 51.1
0.0025 69.3
0.0020 91.6
0.0015 120.4
0.0010 160.9
0.00050 230.3
0.0025 299.6
0.00015 350.7
0.00010 391.2
Table 1
http://www.berkeleycitycollege.edu/wp/somar/files/2013/01/Chapter_12_Che
mical_Kinetics.ppt

Plotting the data above in a graph, results in a curved line. Curved line indicates
change in reaction rate with time. The rate at any instant in time (instantaneous
rate) is the slope of the tangent to the curve. Instantaneous rate is different from
average rate, so when we refer to the rate of a reaction, we will be assuming
it’s the instantaneous rate unless otherwise told. The slope of a tangent at any
point on the curve yields the instantaneous rate at that point.
∆𝑦
Slope of tangent =
∆𝑥
Where:
∆y is the concentration of the product (y-axis in the cartesian plane)
∆x is the time (x-axis in the cartesian plane).

267
NOTE: Practice personal hygiene protocols at all times
∆ y, p

∆ x, t

Figure 2
http://www.berkeleycitycollege.edu/wp/somar/files/2013/01/Chapter_12_Che
mical_Kinetics.ppt
So, if we determine the instantaneous rate of reaction for Phenolphthalein at
t=0
∆ [𝑃ℎ𝑒𝑛𝑜𝑙𝑝ℎ𝑡ℎ𝑎𝑙𝑒𝑖𝑛]
Rate = -
∆ 𝑡𝑖𝑚𝑒
0.00050−0.0010
Rate = -
120−0
Rate = 4.17 x 10-6

3. Average Rate is an overall rate measured over a period of time interval.


Example:
Concentrations of Reactant and Products as a Function of Time for the
Reaction
2NO2(g) → 2NO(g) + O2(g) (at 300ºC)

268
NOTE: Practice personal hygiene protocols at all times
Table 2
http://www.berkeleycitycollege.edu/wp/somar/files/2013/01/Chapter_12_Che
mical_Kinetics.ppt

Using the data above to plot will result like the figure below.
12_291
0.0100

NO2

0.0075
Concentrations (mol/L)

0.0026 [NO2 ]
0.0006

t 70s
0.005
110 s

NO

0.0003
0.0025 70s

O2

50 100 150 200 250 300 350 400


Time (s)
Figure 3
http://www.cabrillo.edu/~dscoggin/chem1b/lecturenotes/Ch16Kinetics(a).ppt

269
NOTE: Practice personal hygiene protocols at all times
Then, if we will calculate the average rates of this reaction during the first 150
seconds and during the second 150 seconds. The initial concentration of NO 2
is 0.0100 mol/L and its concentration after 150 seconds is 0.0055 mol/L.

Solution:

Average Rate During the 1st 150 seconds


∆ [𝑁𝑂2 ]
Average Rate = -
∆𝑡
𝑚𝑜𝑙 𝑚𝑜𝑙
∆ [0.0055 −0.0100 ]
𝐿 𝐿
=- 150 𝑠𝑒𝑐
𝑚𝑜𝑙
0.0045 𝐿
=150 𝑠𝑒𝑐
Average Rate = 3.0 x 10-5 mol/Ls

Average Rate During the 2nd 150 seconds


∆ [𝑁𝑂2 ]
Average Rate = -
∆𝑡
𝑚𝑜𝑙 𝑚𝑜𝑙
∆ [0.0038 𝐿 −0.0055 𝐿 ]
=- 150 𝑠𝑒𝑐
𝑚𝑜𝑙
0.0017
= 150 𝑠𝑒𝑐𝐿
Average Rate = 1.1 x 10-5 mol/Ls

Then we can conclude that the average rate decreases as reaction


progresses because the reactant concentration has decreased.

Rate Laws
The rate law expresses the reaction rate as a function of reactant
concentrations, product concentrations, and temperature. Rate laws displays how the
rate depends on the concentrations of reactants.
The rate law for a chemical reaction is an equation that relates the reaction rate
with the concentrations or partial pressures of the reactants. For the general reaction
aA+bB→C with no intermediate steps in its reaction mechanism, meaning that it is an
elementary reaction, the rate law is given by:
Rate = k[A]m[B]n
Where:

270
NOTE: Practice personal hygiene protocols at all times
[A] and [B] express the concentrations of A and B, respectively, in units
of moles per liter.
The exponents m and n vary for each reaction, and they must be
determined experimentally; they are not related to the stoichiometric coefficients of the
chemical equation.
k is known as the rate constant of the reaction.

The value of this coefficient k will vary with conditions that affect reaction rate,
such as temperature, pressure, surface area, etc. A smaller rate constant indicates a
slower reaction, while a larger rate constant indicates a faster reaction.
The rate law or rate equation or rate expression is a mathematical explanation
of the variation of the rate of a chemical reaction at a function of time. The reaction
rate is dependent on concentration of reactants at a fixed temperature. The rate of
reaction is directly proportional to the reactant concentrations, each concentration
raised to some power.
The rate law expresses the relationship of the rate of a reaction to the rate
constant and the concentration of the reactants raised to some power.
aA + bB → cC + dD
Rate = k [A]m [B]n
Where:
k = constant
m = order with respect to A
n = order with respect to B
x and y are called the order of each reactant
the rate of constant, k, is a proportionality constant in the relationship between
rate and concentration

For example, the rate of the gas-phase decomposition of dinitrogen pentoxide


2N2O5 → 4NO2 + O2
Has been found to be directly proportional to the concentration of N2O5:
Rate = k [N2O5]
For the decomposition of nitrogen dioxide:
2NO2(g) → 2NO(g) + O2(g)
Rate = k[NO2]n

271
NOTE: Practice personal hygiene protocols at all times
Reaction Rate Law
CH3CN → CH3NC Rate = k[CH3CN]
CH3CHO → CH4 + CO Rate = k[CH3CHO]3/2
2 N2O5 → 4 NO2 + O2 Rate = k[N2O5]
H2I2 → 2 HI Rate = k[H2][I2]
Tl3 +Hg2+2 → Tl+1 + 2 Hg+2 Rate = k[Tl+3][Hg2+2][Hg+2]-1
Table 3
https://www.slideshare.net/sathiakumaran/81-rate-law

Types of Rate Laws


1. Differential Rate Law (rate law) – shows how the rate of a reaction depends
on concentrations.
2. Integrated Rate Law – shows how the concentrations of species in the
reaction depend on time.

Reaction Order
To reiterate, the exponents m and n are not derived from the balanced
chemical equation, and the rate law of a reaction must be determined experimentally.
These exponents may be either integers or fractions, and the sum of these exponents
is known as the overall reaction order. A reaction can also be described in terms of
the order of each reactant.
For example, the rate law Rate=k[NO]2[O2] describes a reaction which is
second-order in nitric oxide, first-order in oxygen, and third-order overall. This is
because the value of x is 2, and the value of y is 1, and 2+1=3.
Order of reaction with respect to a reactant is the exponent of its concentration
term in the rate expression. Order must be determined through experiment
Rate = k [A]m [B]n
Where:
m = order with respect to A
n = order with respect to B
m and n can be 0, 1, 2 or fractions

The total reaction order is the sum of all exponents on all concentration of terms;

272
NOTE: Practice personal hygiene protocols at all times
Rate = k [A]m [B]n [C]p
Total order = m + n+ p
The reaction rate for a given reaction is an important tool that enables us to
calculate the specific order of reaction. The order of reaction is important in that it
enables us to classify specific chemical reactions easily and efficiently, within the
reaction including the rate law, units of constant rate, half-life, and much more.
Reaction order can be calculated from the rate law by adding the exponent values of
the reactants in the rate law.
The exponents m, n, and p are frequently, but not always, integers. They must
be determined experimentally and cannot be obtained by simply looking at the
balanced equation.

Overall Reaction Order


Sum of the order of each component in the rate law
Example:
1. Rate = k[H2SeO3][H+]2[1-]3
The overall reaction order is 1 + 2 + 3 = 6

2. A certain rate law is given as Rate=k[H2][Br2]1/2 . What is the reaction order?


m=1, n=1/2
reaction order = m + n=1+1/2=3/2
3. The reaction is first-order in hydrogen, one-half-order in bromine, and 3/2
order overall.
The reaction between nitric oxide and ozone, NO(g)+O3(g)→NO2(g)+O2(g), is
first order in both nitric oxide and ozone. The rate law equation for this reaction
is: Rate=k[NO]1[O3]1. The overall order of the reaction is 1 + 1 = 2.

Types of Reactions with Respect to their order

1. Zero-Order Reaction
➢ The rate of the zero-order reaction does not vary with increasing nor
decreasing reactants concentrations. This means that the rate of the reaction
is equal to the rate of constant, k, of that reaction.
➢ Rate = k

273
NOTE: Practice personal hygiene protocols at all times
➢ It is independent of the concentration of the reacting substance or reacting
substance or reaction rate depends on the zero power of the reactant.
➢ Examples of this type of reaction is the enzyme-catalyzed oxidation of
CH3CH2OH (ethanol) to CH3CHO (acetaldehyde)

There are two general conditions that can give rise to zero-order rates:

- Only a simple reactant molecule is in a location or state in which they are able
to react, and this fraction is continually replenished from the larger pool.
- When two or more reactants are involved, the concentrations of some are much
greater than those of others.
2. First-Order Reaction
➢ It depends on the first power of concentration of a single reactant
➢ When the reaction rate depends on the first power of concentration of a single
reactant
➢ The rate of reaction is directly proportional to the concentration of reacting
substance
➢ It often has the general form A ⟶ products
➢ Rate = k[A]1
➢ Example of a first-order reaction is 2H2O2 ⟶ 2H2O + O2
➢ A first-order reaction depends on the concentration of one reactant, and the
rate law is: r=−dA/dt=k[A].

➢ As usual, k is the rate constant, and must have units of concentration/time; in


this case it has units of 1/s.

A. Using the Method of Initial Rates to Determine Reaction Order


Experimentally
2N2O5(g)→4NO2(g)+O2(g)
➢ The balanced chemical equation for the decomposition of dinitrogen pentoxide
is given above. Since there is only one reactant, the rate law for this reaction
has the general form:
o Rate=k[N2O5]m
➢ In order to determine the overall order of the reaction, it is needed to determine
the value of the exponent m. To do this, measure an initial concentration of
N2O5 in a flask, and record the rate at which the N2O5 decomposes. Then run

274
NOTE: Practice personal hygiene protocols at all times
the reaction a second time, but with a different initial concentration of N 2O5.
Then measure the new rate at which the N2O5 decomposes. By comparing
these rates, it is possible for us to find the order of the decomposition reaction.

3. Second-Order Reaction
➢ It depends on the concentration of two reactant species
➢ Rate = k[A]2 or rate = k[A][B]
➢ Example is NO2 + CO → NO + CO2
➢ A reaction is said to be second-order when the overall order is two. For a
reaction with the general form aA+bB→C, the reaction can be second order in
two possible ways. It can be second-order in either A or B, or first-order in both
A and B. If the reaction were second-order in either reactant, it would lead to
the following rate laws:
o Rate=k[A]2 or
o Rate=k[B]2
➢ The second scenario, in which the reaction is first-order in both A and B,
would yield the following rate law:
o Rate=k[A][B]
Determining Rate Laws from Initial Rates
Example:
O2(g) + 2NO(g) → 2NO2
Rate = k [O2]m[NO]n
1. Find m and n:

Initial rates in a series of experiments for the reaction between O 2 and NO


Experiment Initial Concentration (mol/L) Initial rate (mol/Ls)
O2 NO
1 1.10 x 10-2 1.30 x 10-2 3.21 x 10-3
2 2.20 x 10-2 1.30 x 10-2 6.40 x 10-3
3 1.10 x 10-2 2.60 x 10-2 12.8 x 10-3
4 3.30 x 10-2 1.30 x 10-2 9.60 x 10-3
5 1.10 x 10-2 3.90 x 10-2 28.8 x 10-3
Table 4
https://www3.nd.edu/~aseriann/CHAP16.html/chapt16_lecture.ppt

275
NOTE: Practice personal hygiene protocols at all times
Solution:
To find m and n, a series of experiment is conducted starting with different sets of
reactant concentration. The initial reaction rate is measured in each experiment.

𝑅𝑎𝑡𝑒 2 𝑘 [𝑂2 ]𝑚 𝑛
2 [𝑁𝑂]2
= ([NO] is held constant)
𝑅𝑎𝑡𝑒 1 𝑘 [𝑂2 ]𝑚 𝑛
1 [𝑁𝑂]1

𝑅𝑎𝑡𝑒 2 [𝑂2 ]𝑚
2
=
𝑅𝑎𝑡𝑒 1 [𝑂2 ]𝑚
1

𝑅𝑎𝑡𝑒 2 [𝑂 ]
= ( [𝑂2 ]2 )𝑚
𝑅𝑎𝑡𝑒 1 1 1

6.40 𝑥 10−3 𝑚𝑜𝑙/𝐿∙𝑠 2.20 𝑥 10−2 𝑚


= (1.10 𝑥 10−2)
3.21 𝑥 10−3 𝑚𝑜𝑙/𝐿∙𝑠

1.99 = (2.00)𝑚
m=1
Thus, the reaction is 1st order with respect to O2. When [O2] doubles, the rate
doubles.
𝑅𝑎𝑡𝑒 3 𝑘 [𝑂2 ]𝑚 𝑛
3 [𝑁𝑂]3
= ([O2] is held constant)
𝑅𝑎𝑡𝑒 1 𝑘 [𝑂2 ]𝑚 𝑛
1 [𝑁𝑂]1

𝑅𝑎𝑡𝑒 3 [𝑁𝑂]𝑛
3
=
𝑅𝑎𝑡𝑒 1 [𝑁𝑂]𝑛
1

𝑅𝑎𝑡𝑒 3 [𝑁𝑂]
3 𝑛
= ( )
𝑅𝑎𝑡𝑒 1 [𝑁𝑂] 1

12.8 𝑥 10−3 𝑚𝑜𝑙/𝐿∙𝑠 2.60 𝑥 10−2


= (1.30 𝑥 10−2)𝑛
3.21 𝑥 10−3 𝑚𝑜𝑙/𝐿∙𝑠

3.99 = (2.00)𝑛
n=2
Thus, the reaction is second order with respect to NO. When [NO] doubles, the rate
quadruples.
The rate law is: Rate = k [O2][NO]2
The reaction is third overall

276
NOTE: Practice personal hygiene protocols at all times
2. Many gaseous reactions occur in a car engine and exhaust system. One
such reaction is as follows. Using the initial rates method and the
experimental data, determine the value of the rate constant for this reaction:
NO2(g) + CO(g) → NO(g) + CO2(g)
Rate = k [NO2]m[CO]n
Use the following data to determine the individual and overall reaction orders
Experiment Initial rate (mol/Ls) Initial rate [NO2] (mol/L) Initial rate [CO] (mol/L)
1 0.0050 0.10 0.10
2 0.080 0.40 0.10
3 0.0050 0.10 0.20
Table 5
https://www3.nd.edu/~aseriann/CHAP16.html/chapt16_lecture.ppt

Solution:
Solve for each reactant using the general rate law by applying the method describe
previously.

First, choose 2 experiments in which [CO] remain constant and [NO2] varies

𝑅𝑎𝑡𝑒 2 𝑘 [𝑁𝑂2]𝑚 𝑛
2 [𝐶𝑂]2
= ([CO] is held constant)
𝑅𝑎𝑡𝑒 1 𝑘 [𝑁𝑂2 ]𝑚 𝑛
1 [𝐶𝑂]1

𝑅𝑎𝑡𝑒 2 [𝑁𝑂2 ]𝑚
2
=
𝑅𝑎𝑡𝑒 1 [𝑁𝑂2 ]𝑚
1

𝑅𝑎𝑡𝑒 2 [𝑁𝑂 ]
= ( [𝑁𝑂2 ]2 )𝑚
𝑅𝑎𝑡𝑒 1 1 1

0.080 𝑚𝑜𝑙/𝐿∙𝑠 0.40 𝑚


=( )
0.0050 𝑚𝑜𝑙/𝐿∙𝑠 0.10

16 = (4)𝑚
m=2
Thus, the reaction is 2nd order with respect to NO 2.
𝑅𝑎𝑡𝑒 3 𝑘 [𝑁𝑂2]𝑚 𝑛
3 [𝐶𝑂]3
𝑅𝑎𝑡𝑒 1
= 𝑘 [𝑁𝑂2 ]𝑚 𝑛 ([NO2] is held constant)
1 [𝐶𝑂]1

277
NOTE: Practice personal hygiene protocols at all times
𝑅𝑎𝑡𝑒 3 [𝐶𝑂]𝑛
3
=
𝑅𝑎𝑡𝑒 1 [𝐶𝑂]𝑛
1

𝑅𝑎𝑡𝑒 3 [𝐶𝑂]
3 𝑛
= ( )
𝑅𝑎𝑡𝑒 1 [𝐶𝑂] 1

0.0050 𝑚𝑜𝑙/𝐿∙𝑠 𝑛 0.20


= ( )
0.0050 𝑚𝑜𝑙/𝐿∙𝑠 0.10

1 = (2.00)𝑛
n=0
Thus, the reaction is zero order with respect to CO.
The rate law is: Rate = k [NO2]2[CO]0
The reaction is second order overall
3. This reaction has been studied in the laboratory and the following rate data
were determined at 25ºC. Determine the rate law and the rate constant for the
reaction at 25 °C.
NO(g) + O3(g) → NO2(g) + O2(g)
Trial [NO] (mol/L) [O3] (mol/L) Initial rate
1 1.00 x 10-6 3.00 x 10-6 6.60 x 10-5
2 1.00 x 10-6 6.00 x 10-6 1.32 x 10-6
3 1.00 x 10-6 9.00 x 10-6 1.98 x 10-6
4 2.00 x 10-6 9.00 x 10-6 3.96 x 10-6
5 3.00 x 10-6 9.00 x 10-6 5.94 x 10-6
Table 6
https://opentextbc.ca/chemistry/chapter/12-3-rate-laws/
Solution:
The rate law will have the form:
rate=k[NO]m[O3]n
We can determine the values of m, n, and k from the experimental data using the
following three-part process:
1. Determine the value of m from the data in which [NO] varies and [O3] is
constant. In the last three experiments, [NO] varies while [O3] remains constant.
When [NO] doubles from trial 3 to 4, the rate doubles, and when [NO] triples
from trial 3 to 5, the rate also triples. Thus, the rate is also directly proportional
to [NO], and m in the rate law is equal to 1.

278
NOTE: Practice personal hygiene protocols at all times
2. Determine the value of n from data in which [O3] varies and [NO] is
constant. [NO] is constant in the first three experiments while [O3] varies. The
reaction rate changes in direct proportion to the change in [O 3]. When [O3]
doubles from trial 1 and 2, the rate doubles; when [O 3] triples from the trial 1 to
3, the rate increases also triples. Thus, the rate is directly proportional to [O3],
and n is equal to 1.
The rate law is thus:
rate=k[NO]1[O3]1=k[NO][O3]
3. Determine the value of k from one set of concentrations and the corresponding
rate.
𝑟𝑎𝑡𝑒
k = [𝑁𝑂][𝑂
3]

6.60 𝑥 10−5𝑚𝑜𝑙/𝐿/𝑠
k= 10−6 𝑚𝑜𝑙 10−6 𝑚𝑜𝑙
(1.00 𝑥 𝐿
)(3.00 𝑥 𝐿
)

k = 2.20 x 107 L mol-1 s-1


The large value of k tells us that this is a fast reaction that could play an
important role in ozone depletion if [NO] is large enough.
4. The following rate data were obtained at 25°C for the following reaction. What
is the rate law expression? What is the overall order of the reaction?
2A + B → 3C
Trial [A] (mol/L) [B] (mol/L) Initial Rates
1 0.10 0.10 2.00 x 10-4
2 0.30 0.30 6.00 x 10-4
3 0.10 0.30 2.00 x 10-4
4 0.10 0.40 6.00 x 10-4
Table 7
https://www.bscsd.org/site/handlers/filedownload.ashx?moduleinstanceid=374
&dataid=1507&FileName=apchemistrykineticspracticeproblemssolutions2006.pdf
Rate = k[A]m[B]n

Experiments 1 and 3: [A] is constant = 0.10 M


➢ [B] increases by a factor of 3 but the rate of reaction does NOT change
➢ Therefore [B] does not affect the rate
➢ Therefore, the order of reaction with respect to B = y = 0
Experiments 2 and 3: [B] is constant = 0.30 M

279
NOTE: Practice personal hygiene protocols at all times
➢ [A] decreases by a factor of 3 and the rate of the reaction decreases by
a factor of 3
➢ Therefore, the reaction is first order with respect to A = x = 1

Rate law expression: Rate = k[A]1[B]0 = k[A]


The overall order of reaction is (1+0) = 1

Summary of the 3 Types of Order

Order Rate Law Units of k


Zero Rate = k mol/L.s or mol L-1 s-1
1st Rate = k [A] 1/s or s-1
2nd Rate = k [A]2 L/mol . s or L mol-1 s-1
Table 8
http://www.cabrillo.edu/~dscoggin/chem1b/lecturenotes/Ch16Kinetics(a).ppt

Units of Rate
The rate of reaction as concentration of reactant or product divided by time. The
unit of concentration is mole liter-1 (mole L-1 or mole/L) and time is generally expressed
by seconds. So, reaction rate has units of mole L-1 s-1. But time may be given in any
convenient unit second (s), minutes (min), hours (hr), day (d) or possible years.
Therefore, the units of reaction rates may be:
➢ Mole/L s or mole (L-1) (s-1)
➢ Mole/L min or mole (L-1) (min-1)
➢ Mole/L-hr or mole (L-1) (hr-1)
Units of the Rate Constant, k for several overall reaction order
Overall reaction order Units of k (t in seconds)
0 mol/L.s or mol L-1 s-1
1 1/s or s-1
2 L/mol . s or L mol-1 s-1
3 L2 / mol2 .s or L2 mol-2 s-1
Table 9
https://www3.nd.edu/~aseriann/CHAP16.html/chapt16_lecture.ppt

280
NOTE: Practice personal hygiene protocols at all times
Learning Competency:
• Differentiate zero, first-, and second order reactions (STEM_GC11CK-IIIi-j-132)

Activity 1: IDENTIFICATION

Directions: Describe the concept of the rate of reaction by identifying what is being
described below.
1. It is the speed at which chemical reaction proceeds.
2. Give the formula of “answer in number 1”.
3. It is the speed or rate which the chemical reaction occurs.
4. It is the mathematical explanation of the variation of the rate of a chemical
reaction at a function of time.
5. The reaction time depends on ___________.
6. At any specific time, the rate at which a reaction is proceeding is known as
____.
7. The instantaneous rate of a reaction at “time zero,” when the reaction
commences, is its _____.
8. Give the unit rate of first order of reaction
9. Give the unit rate of second-order of reaction
10. Give the unit rate of zero order of reaction

281
NOTE: Practice personal hygiene protocols at all times
Activity 2: HOW AM I DIFFERENT?

Directions: Determine the difference of zero, first, and second order of reaction

Order Rate law Units of k

Zero
First s-1
Rate = k [A]2

1. What are the types of order of reaction?


______________________________________________________________
______________________________________________________________
____________________________________________
2. Describe first-order of reaction.
______________________________________________________________
______________________________________________________________
____________________________________________
3. Describe second-order of reaction.
______________________________________________________________
______________________________________________________________
____________________________________________
4. Describe zero-order of reaction.
______________________________________________________________
______________________________________________________________
____________________________________________
5. Differentiate zero, first and second order of reaction.
______________________________________________________________
______________________________________________________________
____________________________________________

282
NOTE: Practice personal hygiene protocols at all times
Activity 3: FIND ME!

Directions: Answer the following questions. Determine the rate law from the initial
rates.
1. Determine the rate law and the rate constant for the reaction from the
following experimental data:
Acetaldehyde decomposes when heated to yield methane and carbon
monoxide according to the equation:
CH3CHO(g) ⟶ CH4(g)+CO(g)
Trial [CH3CHO] Initial Rate
1 1.75 x 10-3 2.06 x 10-11
2 3.50 x 10-3 8.24 x 10-11
3 7.00 x 10-3 3.30 x 10-10

2. Using the initial rates method and the experimental data, determine the rate law
and the value of the rate constant for this reaction:
2NO(g) + Cl2(g) → 2NOCl(g)
Trial [NO] (mol/L) [Cl2] (mol/L) Initial Rates
1 0.50 0.50 1.14
2 1.00 0.50 4.56
3 1.00 1.00 9.12

3. Consider the table of initial rates for the reaction:


2ClO2 + 2OH1- → ClO31- + ClO21- + H2O
Trial [ClO2] [OH1-] Initial Rates
1 0.050 0.100 5.75 x 10-2
2 0.100 0.100 2.30 x 10-1
3 0.100 0.050 1.15 x 10-1
Order with respect to [ClO2]: _______
Order with respect to [OH1-]: _______
Rate law of this reaction: _______
Value and units for the rate constant: ________

283
NOTE: Practice personal hygiene protocols at all times
4. Consider the table of initial rate for the reaction between hemoglobin (Hb) and
carbon monoxide.
Trial [Hb] [CO] Initial Rates
1 2.21 1.00 0.619
2 4.42 1.00 1.24
3 3.36 2.40 2.26
Order with respect to [Hb]: _______
Order with respect to [CO]: _______
Rate law of this reaction: _______
Value and units for the rate constant: ________

5. Consider the table of initial rates for the reaction:


2ClO2 + 2OH1- → ClO31- + ClO21- + H2O
Trial [ClO2] [OH1-] Initial Rates
1 0.012 0.012 2.07 x 10-4
2 0.024 0.012 8.28 x 10-4
3 0.012 0.024 4.14 x 10-4
4 0.024 0.024 1.66 x 10-3
Order with respect to [ClO2]: _______
Order with respect to [OH1-]: _______
Rate law of this reaction: _______
Value and units for the rate constant: ________

Activity 4: TRUE OR FALSE

Directions: Label the following statements as True or False. If the statement is


correct write True and if the statement is wrong, write False and change it
to make it true.
_____ 1. All of the rate expressions for the various reactants and products must
equal each other to be correct.

284
NOTE: Practice personal hygiene protocols at all times
_____ 2. The rate of the zero-order reaction does vary with increasing nor
decreasing reactants concentrations.
_____ 3. Second order reaction depends on the concentration of one reactants.
_____ 4. The rate of reaction of 1st order is indirectly proportional to the
concentration of reacting substance
_____ 5. The rate of the zero-order reaction does not vary with increasing nor
decreasing reactants concentrations

Activity 5: ORDER OF REACTION

Directions: Identify what order of reaction is being describe


RATE LAW Order of Reaction
1. Rate = k [O3][Cl]

2. Rate = k[H2][Br2]1/2

3. Rate = k [NO2][F2]

4. Rate = k [ICl][H2]

5. Rate = k [N2O5]

Reflection:

1. I learned that _________________________________________________


___________________________________________________________________
_________________________________________________________

2. I enjoyed most on ______________________________________________


___________________________________________________________________
___________________________________________________________________.

3. I want to learn more on __________________________________________


___________________________________________________________________
___________________________________________________________________.

285
NOTE: Practice personal hygiene protocols at all times
References:
Conceptual and Functional Chemistry Modular Approach by Ma. Cristina D.
Padolina, et al
“Rusting of iron.” BYJU’S The Learning App, 2020.
https://byjus.com/chemistry/rusting-iron-prevention/. Accessed August 25, 2020
“The Rate Law: Concentration and Time”. Lumen Boundless Chemistry.
https://courses.lumenlearning.com/boundless-chemistry/chapter/the-rate-law-
concentration-and-time/. Accessed August 28, 2020
https://www.google.com/search?q=initial+rate+of+reaction&sxsrf=ALeKk00eUPiw0F
4g9Fn0d0ih1Tt5Udt0Xw:1598569396012&source=lnms&tbm=isch&sa=X&ved=2ahU
KEwjO0f2Hv7zrAhWryIsBHWrHCowQ_AUoAXoECAwQAw&biw=1366&bih=657#im
grc=ywQ1JdChakRFxM
Laidler, K. J. “Reaction Rate”. Britannica.
https://www.britannica.com/science/reaction-rate. Accessed August 25, 2020
“Chemical Kinetics”.
http://www.berkeleycitycollege.edu/wp/somar/files/2013/01/Chapter_12_Chemical_Ki
netics.ppt. Accessed August 25, 2020
“Chapter 16 Kinetics: Rates and Mechanism of Chemical Reaction”.
https://www3.nd.edu/~aseriann/CHAP16.html/chapt16_lecture.ppt. Accessed August
25, 2020
Song, Xin. “Chemical Kinetics”.
https://users.cs.duke.edu/~reif/courses/molcomplectures/Kinetics/KineticsOverview/
KineticsOverview.ppt. Accessed August 25, 2020
“Reaction Kinetics”. Slideshare.com. https://www.slideshare.net/sathiakumaran/81-
rate-law. Accessed August 25, 2020
“12.3 Rate Laws”. Opentxtbc. https://opentextbc.ca/chemistry/chapter/12-3-rate-
laws/. Accessed August 28, 2020
Mandes. “Kinetics Practice Problems and Solutions”
http://www.delandhs.org/_cache/files/c/1/c10cd6c6-c726-4b39-87ad-
83a9aed8427c/11280D23BF7E8BE93781F873F16905F8.kinetics-ws-3.pdf.
Accessed August 28, 2020
“AP Chemistry: Kinetics Practice Problems”.
https://www.bscsd.org/site/handlers/filedownload.ashx?moduleinstanceid=374&datai
d=1507&FileName=apchemistrykineticspracticeproblemssolutions2006.pdf.
Accessed August 28, 2020

286
NOTE: Practice personal hygiene protocols at all times
ANSWER KEY

Activity 1: IDENTIFICATION
1. Reaction rate
2. Rate = Change in concentration/ change in time
3. Chemical Kinetics
4. Rate laws
5. Concentration of reactants at a fixed temp
6. Instantaneous rate
7. Initial rate
8. s-1
9. M-1s-1
10. M/s

Activity 2: HOW AM I DIFFERENT?


Complete the table.
Order Rate Law Units of k
Zero Rate = k M/s
1st Rate = k [A] s-1
2nd Rate = k [A]2 M-1s-1

1. What are the types of order of reaction?


➢ Zero order of reaction, 1st order of reaction and 2nd order of reaction
2. Describe first-order of reaction.
➢ When the reaction rate depends on the first power of concentration of a
single reactant. Rate = k[A]
3. Describe second-order of reaction.
➢ It depends on the concentration of two reactant species. Rate = k[A]2 or
rate = k[A][B]

287
NOTE: Practice personal hygiene protocols at all times
4. Describe zero-order of reaction.
➢ The rate of the zero-order reaction does not vary with increasing nor
decreasing reactants concentrations. This means that the rate of the
reaction is equal to the rate of constant, k, of that reaction. Rate = k

5. Differentiate zero, first and second order of reaction.


➢ When the rate is equal to the rate of constant, k it is called Zero order of
reaction, when the rate depends on the first power of concentration of
single reactant it is called first zero of reaction while second order of
reaction happens when the concentration is depend on two reactant
species.
Using a rubric
(https://www.westfordk12.us/sites/westfordmaps/files/uploads/science_writing_rubric
.pdf)

Activity 3: WRITING RATE LAW


1. Rate=k[CH3CHO]2 with k = 6.73 × 10−6 L/mol/s
2. Rate = k[NO]2[Cl]2; k = 9.12 L2 mol−2 h−1;
second order in NO; first order in Cl2
3. Order with respect to [ClO2]: 2
Order with respect to [OH1-]: 1
Rate law of this reaction: Rate = k[ClO2]2[OH1-]1
Value and units for the rate constant: k = 230 L2/ mol2 s
4. Order with respect to [Hb]: 1
Order with respect to [CO]: 1
Rate law of this reaction: Rate = k[HB]1[CO]1
Value and units for the rate constant: 0.28 L/mol s
5. Order with respect to [ClO2]: 2
Order with respect to [OH1-]: 1
Rate law of this reaction: Rate = k[ClO2]2[OH-]
Value and units for the rate constant: k = 1.2 x 102 M-2• s-1

288
NOTE: Practice personal hygiene protocols at all times
Activity 4: TRUE OR FALSE
1. True

2. False

3. False

4. False

5. True

Activity 5: ORDER OF REACTION


RATE LAW Order of Reaction
1. Rate = k [O3][Cl] 2ND

2. Rate = k[H2][Br2]1/2 1ST

3. Rate = k [NO2][F2] 2ND

4. Rate = k [ICl][H2] 2ND

5. Rate = k [N2O5] 1ST

Prepared by:

CHERRY JANE D. BASUG


Baggao National Agricultural School-Main

289
NOTE: Practice personal hygiene protocols at all times
GENERAL CHEMISTRY 2

Name: ____________________________ Grade Level: _________


Date: _____________________________ Score: ______________

LEARNING ACTIVITY SHEET


COLLISION THEORY

Background Information for the Learners (BIL)


In previous chapter, you have learned the different factors on reaction rates at
the molecular level. This module will mainly focus on reactions qualitatively in terms
of molecular collision.
A collision happens when 2 or more objects collide or come together. Molecules
collide in order to react.
The basic component for a reaction to occur is that the reactant particles –
atoms or molecules must collide and interact with each other in same way. This is the
main idea of collision model, which is used to explain many of the observations made
about Chemical Kinetics.
Collision theory is a theory proposed independently by Max Trautz, German in
1916 and William Lewin, British in 1918, explain how chemical reactions occur and why
reaction rates are different from each other.

Collision Theory
Collision theory explains why different reactions occur at different rates and
suggest ways to change the rate of reaction. It also the basic requirements in order to
react molecules must collide with one another. Collision theory also tells that reacting
particles often collide without reacting.
There are three basic requirements to have an effective collision: (1) the
reactants must collide with each other; (2) the molecules must have the sufficient
energy to initiate a reaction (activation energy); and (3) the molecules must have a
proper orientation. To have an effective collision is to have a good result in chemical

290
NOTE: Practice personal hygiene protocols at all times
reaction. Even if two molecules collide with sufficient activation energy, there is no
guarantee that the collision will be successful.
Therefore, if one of the three basic requirements are missing, there is no
chemical reaction. The three basic requirements must be present to have a successful
chemical reaction.

Three Basic Requirements in Effective Collision


1. The reactants must collide with each other
A basic principle of collision theory is that, in order to react, molecules
must collide. This fundamental rule guides any analysis of an ordinary reaction
mechanism. Consider a simple bimolecular step:
A + B → Products
If the two molecules A and B are to react, they must approach closely
enough to disrupt some of their existing bonds and to permit the creation of any
new ones that are needed in the products. Such an encounter is called
a collision. The frequency of collisions between A and B in a gas is proportional
to the concentration of each; if [A] is doubled, the frequency of A−B collisions
will double, and doubling [B] will have the same effect. If all collisions lead to
products, then the rate of a bimolecular process is first-order in A and in B, or
second-order overall:
rate=k[A][B]

https://chem.libretexts.org/Courses/University_of_California_Davis/UCD_Chem
_002C/UCD_Chem_2C%3A_Larsen/Text/Unit_4%3A_Chemical_Kinetics/4.07
%3A_Collision_Theory
Illustration of the dependence of molecular collisions frequency with
concentration. Image used with permission (Sadi Carnot; Public Domain).
Figure 1

291
NOTE: Practice personal hygiene protocols at all times
The frequency of collisions between A and B in a gas is proportional to
the concentration of each.
Consider the reaction in the Haber process for making ammonia:
N2(g)+3H2(g)⇌2NH3(g) (Equation1)
The collision theory says that H2 and N2 will only react when they collide.
Hence, the more frequently they collide, the faster the rate of reaction. This can
be achieved easily by either increasing the pressure on the gasses to
bring H2 and N2 closer together on average or by increasing the temperature to
makes molecules move faster.
Molecular collisions – is the more molecules present, the more collisions
will happen. The collision theory says that as more collisions in a system occur,
there will be more combinations of molecules bouncing into each other. If you
have more possible combinations there is a higher chance that the molecules
will complete the reaction.
When two billiard balls collide, they simply bounce off of one another.
This is also the most likely outcome when two molecules, A and B, come in
contact: they bounce off one another, completely unchanged and unaffected.
In order for a collision to be successful by resulting in a chemical reaction, A
and B must collide with sufficient energy to break chemical bonds. This is
because in any chemical reaction, chemical bonds in the reactants are broken,
and new bonds in the products are formed. Therefore, in order to effectively
initiate a reaction, the reactants must be moving fast enough (with enough
kinetic energy) so that they collide with sufficient force for bonds to break. This
minimum energy with which molecules must be moving in order for a collision
to result in a chemical reaction is known as the activation energy.

2. The molecules must have sufficient energy or activation energy


In the Haber process (Equation 1) at 300 K only 1 in 1011 collisions
between H2 and N2 results in a reaction! At 800 K, this increases to 1
in 104collisions resulting in a reaction. Hence, while the collisions are needed
for a reaction, other aspects contribute. Reacting particles can produce
products when they collide with one another provided those collisions have
enough kinetic energy and the correct orientation. Particles that lack the
necessary kinetic energy may collide, but the particles will simply bounce off

292
NOTE: Practice personal hygiene protocols at all times
one another unchanged. Figure 2 shows the difference. In the first collision, the
particles bounce off one another and no rearrangement of atoms has occurred.
The second collision occurs with greater kinetic energy, and so the bond
between the two red atoms breaks. One red atom bond with the other molecule
as one product, while the single red atom is the other product. The first collision
is called an ineffective collision, while the second collision is called
an effective collision.

https://chem.libretexts.org/Courses/University_of_California_Davis/UCD_Che
m_002C/UCD_Chem_2C%3A_Larsen/Text/Unit_4%3A_Chemical_Kinetics/4.
07%3A_Collision_Theory
Figure 2. An ineffective collision (A) is one that does not result in product formation.
An effective collision (B) is one in which chemical bonds are broken and a product is
formed.
For a gas at room temperature and normal atmospheric pressure, there
are about 1033 collisions in each cubic centimeter of space every second. If
every collision between two reactant molecules yielded products, all reactions
would be complete in a fraction of a second. For example, when two billiard
balls collide, they simply bounce off of each other. This is the most likely
outcome if the reaction between A and B requires a significant disruption or
rearrangement of the bonds between their atoms. In order to effectively initiate
a reaction, collisions must be sufficiently energetic (or have sufficient kinetic
energy) to bring about this bond disruption.
A reaction will not take place unless the particles collide with a certain
minimum energy called the activation energy of the reaction. Activation
energy is the minimum energy required to make a reaction occur. This can be

293
NOTE: Practice personal hygiene protocols at all times
illustrated on an energy profile for the reaction. An energy profile for a simple
exothermic reaction is given in the Figure 3.

https://chem.libretexts.org/Courses/University_of_California_Davis/UCD_Che
m_002C/UCD_Chem_2C%3A_Larsen/Text/Unit_4%3A_Chemical_Kinetics/4.
07%3A_Collision_Theory
Figure 3. Exothermic reaction profile
If the particles collide with less energy than the activation energy, nothing
interesting happens. They bounce apart. The activation energy can be thought
of as a barrier to the reaction. Only those collisions with energies equal to or
greater than the activation energy result in a reaction.
Any chemical reaction results in the breaking of some bonds (which
requires energy) and the formation of new ones (which releases energy). Some
bonds must be broken before new ones can be formed. Activation energy is
involved in breaking some of the original bonds. If a collision is relatively gentle,
there is insufficient energy available to initiate the bond-breaking process, and
thus the particles do not react.
Energetic collisions between molecules cause interatomic bonds to
stretch and bend, temporarily weakening them so that they become more
susceptible to cleavage. Distortion of the bonds can expose their associated
electron clouds to interactions with other reactants that might lead to the
formation of new bonds.

294
NOTE: Practice personal hygiene protocols at all times
https://chem.libretexts.org/Courses/University_of_California_Davis/UCD_Chem_002
C/UCD_Chem_2C%3A_Larsen/Text/Unit_4%3A_Chemical_Kinetics/4.07%3A_Collis
ion_Theory
Figure 4. Anatomy of a Collision
Chemical bonds have some of the properties of mechanical springs: their
potential energies depend on the extent to which they are stretched or
compressed. Each atom-to-atom bond can be described by a potential energy
diagram that shows how its energy changes with its length. When the bond
absorbs energy (either from heating or through a collision), it is elevated to a
higher quantized vibrational state (indicated by the horizontal lines) that
weakens the bond as its length oscillates between the extended limits
corresponding to the curve in Figure 4.
When the bond absorbs energy (either from heating or through a
collision), it is elevated to a higher quantized vibrational state (indicated by the
horizontal lines) that weakens the bond.
A particular collision will typically excite a number of bonds in this way.
Within about 10–13 seconds, this excitation is distributed among the other bonds
in the molecule in complex and unpredictable ways that can concentrate the
added energy at a particularly vulnerable point. The affected bond can stretch
and bend farther, making it more susceptible to cleavage. Even if the bond does
not break by pure stretching, it can become distorted or twisted so as to expose

295
NOTE: Practice personal hygiene protocols at all times
nearby electron clouds to interactions with other reactants that might encourage
a reaction.
The Maxwell-Boltzmann Distribution
Because of the key role of activation energy in deciding whether a
collision will result in a reaction, it is useful to know the proportion of the particles
present with high enough energies to react when they collide. In any system,
the particles present will have a very wide range of energies. For gases, this
can be shown on a graph called the Maxwell-Boltzmann distribution, a plot
showing the number of particles with each particular energy.

https://chem.libretexts.org/Courses/University_of_California_Davis/UCD_Che
m_002C/UCD_Chem_2C%3A_Larsen/Text/Unit_4%3A_Chemical_Kinetics/4.
07%3A_Collision_Theory
Figure 5
The area under the curve measures of the total number of particles
present. Remember that for a reaction to occur, particles must collide with
energies equal to or greater than the activation energy for the reaction. The
activation energy is marked on the Maxwell-Boltzmann distribution with a green
line:

296
NOTE: Practice personal hygiene protocols at all times
https://chem.libretexts.org/Courses/University_of_California_Davis/UCD_Chem_002
C/UCD_Chem_2C%3A_Larsen/Text/Unit_4%3A_Chemical_Kinetics/4.07%3A_Collis
ion_Theory
Figure 6
Notice that the large majority of the particles have insufficient energy to
react when they collide. To enable them to react, either the shape of the curve
must be altered, or the activation energy shifted further to the left to lower
energies.

3. The molecules must have a proper orientation


Even if two molecules collide with sufficient activation energy, there is
no guarantee that the collision will be successful. In fact, the collision theory
says that not every collision is successful, even if molecules are moving with
enough energy. The reason for this is because molecules also need to collide
with the right orientation, so that the proper atoms line up with one another, and
bonds can break and re-form in the necessary fashion. However, because
molecules in the liquid and gas phase are in constant, random motion, there is
always the probability that two molecules will collide in just the right way for
them to react.

297
NOTE: Practice personal hygiene protocols at all times
Consider a simple reaction involving a collision between two molecules:
for example, ethene, CH2=CH2, and hydrogen chloride, HCl. These react to
give chloroethane as shown:
H2C=CH2+HCl→CH3CH2Cl (2)
As a product of the collision between the two molecules, the double bond
in ethene is converted into a single bond. A hydrogen atom is now attached to
one of the carbons and a chlorine atom to the other. The reaction can only
happen if the hydrogen end of the H-Cl bond approaches the carbon-carbon
double bond. No other collision between the two molecules produces the same
effect. The two simply bounce off each other.

https://chem.libretexts.org/Courses/University_of_California_Davis/UCD_Che
m_002C/UCD_Chem_2C%3A_Larsen/Text/Unit_4%3A_Chemical_Kinetics/4.07%3
A_Collision_Theory
Figure 7. The collisions display in the diagram, only collision 1 may possibly
lead on to a reaction (if enough kinetic energy is involved). The other three collisions
will not lead to a reaction irrespective of the kinetic energy involved.
With no knowledge of the reaction mechanism, one might wonder why
collision 2 would be unsuccessful. The double bond has a high concentration

298
NOTE: Practice personal hygiene protocols at all times
of negative charge around it due to the electrons in the bonds. The approaching
chlorine atom is also partially negative due to dipole created by the
electronegativity difference between it and hydrogen. The repulsion simply
causes the molecules to bounce off each other. In any collision involving
unsymmetrical species, the way they hit each other is important in determining
whether a reaction occurs.

Summary
Collision theory provides qualitative explanation of chemical reactions and the
rates at which they occur. To have an effective collision, the three basic principles
must be achieved, so there will be a chemical reaction. The three basic principles of
collision theory: (1) reactants must collide with each other; (2) molecules must have a
sufficient energy to initiate the reaction known as activation energy; and (3) molecules
must have proper orientation. If one of the three basic principles is not present, then
there will be no chemical reaction.
The different factors in reaction rates discussed in last chapter affect the
collision theory. Temperature is one of the factors in reaction rates, the greater number
of reactant molecules are more energetic at higher temperatures than at lower
temperatures, thereby making collisions more effective for products to form.
Concentration is also a factor in reaction rates, the more number of reactant
molecules, the greater probability for effective collisions to form products.

Learning Competency:

Explain reactions qualitatively in terms of molecular collisions (STEM_GC11CK-IIIi-j-


136)

299
NOTE: Practice personal hygiene protocols at all times
Activity 1: ARRANGE AND IDENTIFY ME!
Directions: Arrange the following jumbled letters about collision inside BOX X.
Then, find the meaning of the words from BOX Y. In BOX Z column A write the
assembled word and the corresponding letter in column B.

BOX X
1. O C L L I O N I S
2. R O P P E R T A R I E N T I O N
3. T E F F E C I V E L I O N C O N I S
4. C A T T I V A I O N R E N E G Y
5. C I F F U I E N T S Y E N E R G

BOX Y
a. It happens when two or more objects collide or come together
b. To have a good chemical reaction is to have a/an ______
c. To have a right orientation is to have ______
d. It is sometimes called activation energy
e. A reaction will not take place unless the particles collide with a certain
minimum energy called the ______

BOX Z
A B
1. _____________________________ = ______
2. _____________________________ = ______
3. _____________________________ = ______
4. _____________________________ = ______
5. _____________________________ = ______

300
NOTE: Practice personal hygiene protocols at all times
Activity 2: TRUE OR FALSE

Directions: Label the following statements as True or False. If the statement is


false, change it to make it true.

_____ 1. Collision theory provides quantitatively explanation of chemical reaction


and the rates at which they occur.
_____ 2. A collision happens when two or more objects collide or come together.
_____ 3. The three basic principles of collision theory are: (1) reactants must collide
with each other; (2) reactants must have sufficient energy to initiate a reaction or
activation energy; and (3) particles must be in proper orientation.
_____ 4. To have an effective collision is to result a chemical reaction.
_____ 5. There will be a good chemical reaction even if there are 2 requirements
present.

301
NOTE: Practice personal hygiene protocols at all times
Activity 3: ESSAY

Directions: Answer the following questions briefly.

1. Chemical reaction occurs when reactants collide. What are the factors that may
prevent from producing chemical reaction?
______________________________________________________________
______________________________________________________________
____________________________________________

2. Will there be a chemical reaction without activation energy?


______________________________________________________________
______________________________________________________________
____________________________________________

3. How does it affect the factors of rate reaction to the collision?


______________________________________________________________
______________________________________________________________
____________________________________________

4. What happens after reactant molecules collide?


______________________________________________________________
______________________________________________________________
____________________________________________

5. Do you agree that if there are molecules to collide, there will be more
chemical reaction to happen?
______________________________________________________________
______________________________________________________________
____________________________________________

302
NOTE: Practice personal hygiene protocols at all times
Activity 4: CROSSWORD PUZZLE
Directions: Reveal the word in the box by answering the following questions.

Across
1. It explains the different reactions occur a different rate and suggest ways to
change the rate of reaction.
2. ______ energy is the minimum energy with which reactants must collide in
order for a reaction to occur
3. One of the basic principles of collision theory is to have a proper ________.
Down
1. This happens when 2 or more objects collide or come together.
4. To have a good chemical reaction is to have an ______ collision.
5. Who is the other person aside from William Levin to proposed Collision Theory?

303
NOTE: Practice personal hygiene protocols at all times
Activity 5: SPILL THE MESSAGE!

Directions: Answer the following questions. Find your answer in the table below.
Your answer corresponds with the letter. Then, transfer the numbered letters on the
answer box to form a beautiful phrase. The first one is done for you.

“___ __ __ __ __ __ __ __ __ __ __ __ __ __ __ __ __ __
13 11 16 9 10 13 13 10 2 4 3 10 8 4 1 8 1 9
__ F __ __ __ __ __ __ __ __ __ __ __ __ __ __ __
10 5 12 14 3 3 4 12 12 2 14 13 11 1 13 6 4 11

__ __ __ __ __ __ __ __ __ __ __ __ __ __ __ __ F
13 11 6 13 10 2 4 3 10 8 4 1 8 1 9 10 5

__ __ __ __ __”
15 1 7 14 4
ALBERT EINSTEIN

Questions:

1. It is the study of rates of chemical reactions or how fast reaction occur.


2. Who is the other person aside from Max Trautz to propose Collision Theory?
3. From where is William Levin?
4. What year did William Levin proposed Collison Theory?
5. From where is Max Trautz?
6. What year did Max Trautz proposed Collison Theory?
7. One of the basic principles of Collision Theory is: (3) The molecules must
have _______
8. (2) The molecules must have sufficient energy to initiate a reaction or ___
9. (1) The reactants must ____ with each other.
10. What happens if proper orientation is not present in the collision?
11. Is activation energy important in the 3 basic principles of collision?
12. Is activation energy more important than proper orientation?
13. What is the 1st basic principle of Collision Theory?
14. How many basic principles are there in Collision Theory?
15. To have an activation energy is to have ______
16. The more molecules present, the more collision will happen

304
NOTE: Practice personal hygiene protocols at all times
Answer Letter
1. Activation energy A
2. Collide B
3. The reactants must collide with each other C
4. 3 E
5. Molecular Collision F
6. Sufficient energy H
7. Chemical Kinetics L
8. William Levin M
9. British N
10. German O
11. 1918 R
12. 1916 S
13. Proper Orientation T
14. No Chemical Reaction U
15. Yes V
16. No Y

Reflection:

1. I learned that _______________________________________________________


___________________________________________________________________
_________________________________________________________

2. I enjoyed most on ___________________________________________________


___________________________________________________________________
_______________________________________________________________.

3. I want to learn more on _______________________________________________


___________________________________________________________________
_____________________________________________________________.

305
NOTE: Practice personal hygiene protocols at all times
References:
Encarta Encyclopedia @ Microsoft corporation
Kanthasamy, V. “Collision and Transition State Theory”. Slideshare.net.
https://www.slideshare.net/vksprasath/trasition-and-collision-theory. Accessed July
2, 2020
“4.7: Collision Theory”. Chemistry Libretexts, 2019.
https://chem.libretexts.org/Courses/University_of_California_Davis/UCD_Chem_002
C/UCD_Chem_2C%3A_Larsen/Text/Unit_4%3A_Chemical_Kinetics/4.07%3A_Collis
ion_Theory. Accessed July 2, 2020
“The Collision Theory”. Lumen Introduction to Chemistry.
https://courses.lumenlearning.com/introchem/chapter/the-collision-theory/. Accessed
July 2, 2020
Ogena, E. B. et al. “Chemical Kinetics: Reaction Rates and Collision Theory”.
Commission on Higher Education. Philippine Normal University.
https://files.catbox.moe/z73zoe.pdf. Accessed July 1, 2020
“Rate of Reaction”. Rader’s Chem4kids.com.
http://www.chem4kids.com/files/react_rates.html#:~:text=The%20collision%20theory
%20says%20that,molecules%20will%20complete%20the%20reaction. Accessed
July 3, 2020
“12.1 Chemical Reaction Rates”. Openstax. https://openstax.org/books/chemistry-
2e/pages/12-exercises. Accessed July 3, 2020
https://www.google.com/search?q=inspirational+success+albert+einstein+quotes&tb
m=isch&hl=en&hl=en&ved=2ahUKEwjWxK_b2YrqAhXVzIsBHS9eAWcQrNwCKAJ6
BQgBEKIC&biw=1349&bih=657#imgrc=-008L0o438JTmM. Accessed July 1, 2020

306
NOTE: Practice personal hygiene protocols at all times
ANSWER KEY

Activity 1: ARRANGE AND IDENTIFY ME!


BOX Z
1. COLLISION =a
2. PROPER ORIENTATION =c
3. EFFECTIVE COLLISION =b
4. ACTIVATION ENERGY =e
5. SUFFICIENT ENERGY =d

Activity 2: TRUE OR FALSE


1. FALSE, Qualitatively
2. TRUE
3. TRUE
4. TRUE
5. FALSE, 3 basic requirements present

Activity 3: ESSAY
1. Chemical reaction occurs when reactants collide. What are the factors that may
prevent from producing chemical reaction?
The reactants are either moving too slowly to have kinetic energy to exceed
activation energy for the reaction or the orientation of the molecules when they
collide may prevent the reaction occurring.

2. Will there be a chemical reaction without activation energy?


There will be no chemical reaction after the reactants collide because the
reactants do not have sufficient energy to initiate a reaction.

307
NOTE: Practice personal hygiene protocols at all times
3. How does it affect the factors of rate reaction to the collision?

The rate of chemical reaction is affected by several parameters. Examples is


the Temperature is one of the factors in reaction rates, the greater number of
reactant molecules are more energetic at higher temperatures than at lower
temperatures, thereby making collisions more effective for products to form.
Concentration is also a factor in reaction rates, the more number of reactant
molecules, the greater probability for effective collisions to form products.

4. What happens after reactant molecules collide?


Assuming the sufficient energy and proper orientation are present after the
reactant molecules collide, then there will be a chemical reaction. But, if proper
orientation is not present even though there is sufficient energy to initiate the
reaction, then there will be no chemical reaction or simply the molecules bounce
off after they collide

5. Do you agree that if there are molecules to collide, there will be more
chemical reaction to happen?
I agree because according to the collision theory says that as more collisions
in a system occur, there will be more combinations of molecules bouncing into
each other. If you have more possible combinations there is a higher chance
that the molecules will complete the reaction.

308
NOTE: Practice personal hygiene protocols at all times
Activity 4: CROSSWORD PUZZLE

Activity 5: SPILL THE MESSAGE

“T R Y N O T T O B E C O M E A M AN
13 11 16 9 10 13 13 10 2 4 3 10 8 4 1 8 1 9
O F S UCCE SS BU T R AT H ER
10 5 12 14 3 3 4 12 12 2 14 13 11 1 13 6 4 11

T R Y T O B E C O M E A M AN OF
13 11 6 13 10 2 4 3 10 8 4 1 8 1 9 10 5

V A L U E”
15 1 7 14 4

ALBERT EINSTEIN

Prepared by:

CHERRY JANE D. BASUG


Baggao National Agricultural School-Main

309
NOTE: Practice personal hygiene protocols at all times
GENERAL CHEMISTRY 2

Name: _______________________________ Grade Level: _____________


Date: _______________________________ Score: __________________

LEARNING ACTIVITY SHEET


ACTIVATION ENERGY AND CATALYST IN CHEMICAL
REACTIONS

Background Information for the Learners (BIL)


In the previous modules, you learned that the collision theory is very useful in
explaining many characteristics of chemical reactions. No reaction can take place
between two particles if they are far apart. They must come in contact so that they
may be able to break bonds, exchange atoms, and form new bonds.

Figure 1 Molecules of different conditions under Collision Theory


Photo taken from:
https://saintschemistry10.weebly.com/uploads/5/1/9/3/51932861/gw500h283_orig.jpg

What should happen for a reaction to take place? The reactant molecules must
collide. But mere collision is not enough. The reactant particles must possess energy

310
NOTE: Practice personal hygiene protocols at all times
equal to or higher than a critical value. This critical amount of energy required for a
reaction to take place is called activation energy.

What is activation energy?


There is an easy way to think about the meaning of activation energy that could
be analogous to that of a mountain. If you want to go over a mountain through a steep
road, your vehicle should have sufficient power, regardless of how easy it will be to go
down on the other side of the mountain. Try to look at Figure 2 for an analogy to Greek
mythology
All molecules possess a
certain minimum amount of energy.
The energy can be in the form of
kinetic energy or potential energy.
When molecules collide, the kinetic
energy of the molecules can be used
to stretch, bend, and ultimately break Figure 2: In Greek mythology Sisyphus was
bonds, leading to chemical reactions. punished by being forced to roll an immense
boulder up a hill, only to watch it roll back down,
If molecules move too slowly with little and to repeat this action forever. If this were a
kinetic energy, or collide with chemical reaction, then it would never be
observed since the reactants must overcome the
improper orientation, they do not react energy barrier to get to the other side (products).
and simply bounce off each other. Photo taken from:
https://chem.libretexts.org/@api/deki/files/5133
However, if the molecules are moving 0/sisyphus1.jpg?revision=1
fast enough with a proper collision
orientation, such that the kinetic energy upon collision is greater than the minimum
energy barrier, then a reaction occurs.

311
NOTE: Practice personal hygiene protocols at all times
For the reaction to take place, some, or all the chemical bonds in the reactants
must be broken so that new bonds, those of the products, can form. To get the bonds

Figure 3 A typical energy diagram showing the transition state


Photo taken from: https://cdn.britannica.com/28/4728-050-C4ABAC20/activation-energy-
equation-Arrhenius-reactants-products-reaction.jpg

into a state that allows them to break, the molecule must be contorted (deformed, or
bent) into an unstable state called the transition state. The transition state is a high-
energy state, and some amount of energy – the activation energy – must be added for
the molecule to reach it. Because the transition state is unstable, reactant molecules
do not stay there long, but quickly proceed to the next step of the chemical reaction.
Most chemical reactions that take place in cells are like the hydrocarbon
combustion example: the activation energy is too high for the reactions to proceed
significantly at ambient temperature. At first, this seems like a problem; after all, you
cannot set off a spark inside of a cell without causing damage. Fortunately, it is
possible to lower the activation energy of a reaction, and to thereby increase reaction
rate. The process of speeding up a reaction by reducing its activation energy is known
as catalysis, and the factor that is added to lower the activation energy is called
a catalyst.

Catalysts and the Rates of Chemical Reactions


A catalyst is a substance that speeds up the rate of a chemical reaction but is
not consumed during the course of the reaction. A catalyst will appear in the steps of
a reaction mechanism, but it will not appear in the overall chemical reaction (as it is
not a reactant or product). Generally, catalysts alter the mechanism of the reaction in

312
NOTE: Practice personal hygiene protocols at all times
a substantial way such that the new barriers along the reaction coordinate are
significantly lower. By lowering the activation energy, the rate constant is greatly
increased (at the same temperature) relative to the uncatalyzed reaction.
Four criteria must be satisfied for something to be classified as catalyst.
• Catalysts increase the rate of reaction.
• Catalysts are not consumed by the reaction.
• A small quantity of catalyst should be able to affect the rate of reaction
for a large amount of reactant.
• Catalysts do not change the equilibrium constant for the reaction.
The first criterion provides the basis for defining a catalyst as something that
increases the rate of a reaction. The second reflects the fact that anything consumed
in the reaction is a reactant, not a catalyst. The third criterion is a consequence of the
second; because catalysts are not consumed in the reaction, they can catalyze the
reaction over and over again. The fourth criterion results from the fact that catalysts
speed up the rates of the forward and reverse reactions equally, so the equilibrium
constant for the reaction remains the same.

Effect of catalysts
The effect of a catalyst is that it lowers the activation energy for a reaction.
Generally, this happens because the catalyst changes the way the reaction happens
(the mechanism). We can visualize this for a simple reaction coordinate in the
following way.

Figure 4 Energy diagram for catalyzed and noncatalyzed reaction


Photo taken from: http://ch302.cm.utexas.edu/kinetics/catalysts/catalysts-all.php

313
NOTE: Practice personal hygiene protocols at all times
How do catalysts work?
Many catalysts work in the same way. They provide a means for the reactant
molecules to break bonds and then form temporary bonds with the catalyst. This
means the catalyst must be somewhat reactive, but not too reactive (since we do not
want these bonds to be permanent). For example, platinum (Pt) metal serves as a
catalyst for many reactions involving hydrogen gas or oxygen gas. This is because
the Pt surface allows the H2 or O2 to break their bonds then form atomic species that
are "bonded" to the Pt. However, these new bonds can be weak enough that the

Figure 5 Reaction between ethene and hydrogen gas with Platinum which serves as
a catalyst
Photo taken from: http://ch302.cm.utexas.edu/kinetics/catalysts/catalysts-all.php

atomic species can then react with other molecules and leave the surface. In this way,
the Pt metal returns to its pristine state after the reaction. For example, the cartoon
below depicts the reaction of ethene and hydrogen gas. The hydrogen lands on the
surface and breaks its bond to form H atoms bonded to the surface (2). The double
bond of the ethene is also broken and the two carbon atoms also bond to the surface
(3). Then the H atoms can migrate until they collide with the bound carbon species
and react (4) to form ethane which can then leave the surface (5).

Enzymes are biological catalysts. They are proteins that fold into particular
conformations such that they can help speed up very particular chemical
reactions. For biochemical reactions, the reactant is typically called the

314
NOTE: Practice personal hygiene protocols at all times
substrate. The substrate is converted into the product. The mechanisms for many
enzymes are very similar. The substrate(s) and the enzyme bind into a complex. The
physical location on the enzyme in which the substrate binds is called the "active
site". Once bound this complex can then weaken particular bonds in the substrate
such that chemistry occurs to form the product. The product is weakly bound to the
substrate such that it now dissociates and the enzyme is free to bind another substrate
molecule.

Since activation energy and catalyst were already discussed in the first part of
this module, there are different learning activities which were prepared to test your
understanding with regards to the topic. Let us get started!

Learning Competency:
Explain activation energy and how a catalyst affects the reaction. (STEM-GC11CK-
IIIi-j137)

Activity 1: SELF CHECK WITH THE CONCEPT

Directions: Answer the following questions completely.


1. Chemical reactions occur when reactants collide. What are two factors that
may prevent a collision from producing a chemical reaction?
______________________________________________________________
______________________________________________________________
______________________________________________________________
2. What is the activation energy of a reaction, and how is this energy related to
the activated complex of the reaction?
______________________________________________________________
______________________________________________________________
______________________________________________________________

315
NOTE: Practice personal hygiene protocols at all times
3. Describe the relationship between activation energy and the rate of a chemical
reaction.
______________________________________________________________
______________________________________________________________
______________________________________________________________
4. What happens when a catalyst is used in a reaction?
______________________________________________________________
______________________________________________________________
______________________________________________________________
5. What effect does a catalyst have on the stoichiometry of a reaction?
______________________________________________________________
______________________________________________________________
_____________________________________________________

Activity 2: ACTIVATING IMAGINATION!


Directions: Make your own illustration of activation energy like that in Figure 2 (Greek
Mythology analogy). And below the illustration, explain how the analogy works. Refer
to the Rubrics below on how your illustration will be graded.

316
NOTE: Practice personal hygiene protocols at all times
Rubrics
CRITERIA LEVEL 1 LEVEL 2 LEVEL 3 RATING
(2 points) (3 points) (5 points)
The work is The work is
The work appears
presented in a neat presented in a neat,
NEATNESS AND unorganized. It is
and organized clear and organized
ORGANIZATION not visually
fashion that is fashion that is easy
appealing.
appealing. to visualize.
Many parts of the Few parts of the The whole
COMPLETION OF illustration were still illustration were illustration was
THE ILLUSTRATION not finished and still not finished and finished and
furnished. furnished. furnished.
Several of the
One or two of the graphics used on the
No graphics made graphics used in the illustration reflect an
GRAPHICS-
by the students are illustration reflect exceptional degree
ORIGINALITY
included. students’ creativity of students’
in their creation. creativity in their
creation.
The illustration
The illustration does The illustration
somewhat
not accurately accurately portrays
CONTENT accurately portrays
portrays an analogy an analogy with that
ACCURACY an analogy with
with that of an of an activation
that of an activation
activation energy energy
energy
Analogy is
ANALOGY
Analogy is not somewhat Analogy is accurately
EXPLANATION
accurately explained accurately explained
ACCURACY
explained
Total:

317
NOTE: Practice personal hygiene protocols at all times
Activity 3: WHERE IS THE CATALYSTS?
Directions: Identify the catalyst in the following statements. Write your answer on the
space provided after each statement.
1. Hydrogen peroxide will decompose into water and oxygen gas. Two molecules
of hydrogen peroxide will produce two molecules of water and one molecule of
oxygen. Adding potassium permanganate to the hydrogen peroxide will cause
a reaction that produces a lot of heat, and water vapor will shoot out.
________________
2. The catalytic converter in a car contains platinum, which change carbon
monoxide, which is toxic, into carbon dioxide. _________________________
3. If you light a match in a room with hydrogen gas and oxygen gas, there will be
an explosion and most of the hydrogen and oxygen will combine to create water
molecules. _______________
4. Through normal cell processes, living things produce hydrogen peroxide in their
cells. But hydrogen peroxide is a poison so the cells need a way to break it
down very quickly. Cells contain catalase, which breaks down hydrogen
peroxide at a very fast rate. _____________________
5. Protein is broken down in the stomach by the enzyme pepsin. ______________

318
NOTE: Practice personal hygiene protocols at all times
Activity 4: TANGLED CATALYSTS PROCESS
Directions: The diagrams and statements below show how a catalyst speeds up this
reaction but they are in the wrong order. Cut them out, then match up the statements
to the diagrams so they describe what is happening. Then, stick them down in the
correct order. Use arrows to indicate the sequence and a separate sheet for your
output.

Molecules of reactant A form bonds The product leaves the surface


with the active sites on the catalyst. of the catalyst.
The bonds in between the atoms of
the molecule start to break.
Molecules of reactant A come close
to active sites on surface of the
Molecules of reactant B align itself catalyst.
alongside reactant A.
Bonds between the atoms in
reactant B start to break as new
ones are formed between the atoms
of reactant A and B.

319
NOTE: Practice personal hygiene protocols at all times
Activity 5: BIOLOGICAL CATALYSTS TO RESCUE!

Directions: Enzymes are proteins that act as catalysts and some help biochemical
reactions occur at a rapid rate. You already learned this in your General Biology 1.
Give at least five enzymes found inside the body and provide the function or role of
the given enzyme in the body. (Note: Two (2) points for the enzyme and three (3)
points for the function/role)

Enzymes Function(s)/Role(s) in the Body

1. 1.

2. 2.

3. 3.

4. 4.

5. 5.

320
NOTE: Practice personal hygiene protocols at all times
Reflection:
1. I learned that _____________________________________________________
________________________________________________________________
________________________________________________________________

2. I enjoyed most on _________________________________________________


________________________________________________________________
________________________________________________________________

3. I want to learn more on _____________________________________________


________________________________________________________________
________________________________________________________________

321
NOTE: Practice personal hygiene protocols at all times
References:

Collision Theory. Retrieved from https://saintschemistry10.weebly.com/collision-


theory.html
The Arrhenius Law: Activation Energies. Retrieved May 19, 2020 from
https://chem.libretexts.org/Bookshelves/Physical_and_Theoretical_Chemistry_Textb
ook_Maps/Supplemental_Modules_(Physical_and_Theoretical_Chemistry)/Kinetics/
Modeling_Reaction_Kinetics/Temperature_Dependence_of_Reaction_Rates/The_Ar
rhenius_Law/The_Arrhenius_Law%3A_Activation_Energies
CK-12 Foundation. Activation Energy. Retrieved from
https://www.ck12.org/chemistry/activation-energy/lesson/Activation-Energy-MS-PS/
Catalyst. Retrieved from http://ch302.cm.utexas.edu/kinetics/catalysts/catalysts-
all.php
Activation Energy. https://www.khanacademy.org/science/high-school-biology/hs-
energy-and-transport/hs-enzymes/a/activation-energy
Chemistry: Catalyst Worksheet. http://smart-learning.co.uk/teachers-club/wp-
content/uploads/2014/09/Chemistry-Catalysts-worksheet.pdf

322
NOTE: Practice personal hygiene protocols at all times
ANSWER KEY
Activity 1: Self-Check with the Concept
1. The reactants either may be moving too slowly to have enough kinetic energy
to exceed the activation energy for the reaction, or the orientation of the
molecules when they collide may prevent the reaction from occurring.
2. The activation energy is the minimum amount of energy necessary to form the
activated complex in a reaction. It is usually expressed as the energy necessary
to form one mole of activated complex.
3. If the activation energy is high, the rate of a chemical reaction is slow and if the
activation energy is low, the rate of a chemical reaction is fast
4. A catalyst changes the reaction mechanism, in the process lowering the
activation energy.
5. A catalyst has no effect on the balanced reaction equation and stoichiometry of
a reaction.
Activity 2: Activating your Imagination
• Students’ output may vary. See attached Rubrics below the activity for scoring
purposes.
Activity 3: Where is the Catalyst?
1. Potassium permanganate
2. Platinum
3. Lighting a match (heat)
4. Enzyme catalase
5. Enzyme pepsi

323
NOTE: Practice personal hygiene protocols at all times
Activity 4: Tangled Catalyst Process

1 2

Molecules of reactant A come close to Molecules of reactant A form bonds with


active sites on surface of the catalyst. the active sites on the catalyst. The
bonds in between the atoms of the
molecule start to break.

3
4

Bonds between the atoms in reactant B Molecules of reactant B align itself


alongside reactant A.
start to break as new ones are formed
between the atoms of reactant A and B.

The product leaves the surface


of the catalyst.

324
NOTE: Practice personal hygiene protocols at all times
Activity 5: Biological Catalyst to the Rescue!
(Sample Answers)

Enzymes Function(s)/Role(s) in the Body

1. Lipases 1. A group of enzymes that help digest fats in the gut.

2. Amylase 2. Helps change starches into sugars. Amylase is


found in saliva.

Prepared by:

GENEVIE VINAGRERA
Licerio Antiporda Sr National High School- Main

325
NOTE: Practice personal hygiene protocols at all times
GENERAL CHEMISTRY 2

Name: _______________________________ Grade Level: _____________


Date: _______________________________ Score: __________________

LEARNING ACTIVITY SHEET


CATALYSIS AND TYPES OF CATALYSTS

Background Information for the Learners (BIL)

In chemistry, a catalyst is a substance that speeds up the rate of a reaction


without itself being consumed in the reaction. Any reaction that makes use of a
catalyst is termed catalysis. Be careful about this distinction when reading
chemistry material; a catalyst (plural "catalysts") is a physical substance, but
catalysis (plural "catalyses") is a process.
Catalysts and their associated catalytic reactions come in two main types:
homogeneous catalysts, and heterogeneous catalysts. Some sources consider
biocatalysts (usually called enzymes) as the third type of catalyst. But in this module,
we are going to cite and differentiate the two types of catalyst: homogeneous and
heterogeneous catalysts.

Types of Catalyst
Catalysts may be classified generally according to their physical state, their
chemical nature, or the nature of the reactions that they catalyze.
Catalysts can be divided into two types, homogeneous or heterogeneous,
depending on the reaction phase that they occupy. Homogeneous catalysts are those
that occupy the same phase as the reaction mixture (typically liquid or gas), while
heterogeneous catalysts occupy a different phase.

Homogeneous Catalysis
When the catalyst and the reacting substances are present together in a single
state of matter, usually as a gas or a liquid, it is customary to classify the reactions as
326
NOTE: Practice personal hygiene protocols at all times
cases of homogeneous catalysis. Oxides of nitrogen serve as catalysts for the
oxidation of sulfur dioxide in the lead chamber process for producing sulfuric acid, an
instance of homogeneous catalysis in which the catalyst and reactants are gases.
This most often happens with gaseous catalyst-reactant pairs. Types of
homogeneous catalysts include organic acids in which the donated hydrogen atom
is replaced by a metal, several compounds blending carbon and metal elements in
some form, and carbonyl compounds. An example of this type of catalysis involving
liquids is the conversion of persulfate and iodide ions to sulfate ion and iodine:
S2O82- + 2 I- → 2 SO42- + I2
This reaction would have a difficult time proceeding on its own despite the
favorable energetics, because both reactants are negatively charged and therefore
their electrostatic qualities are in opposition to their chemical qualities. But if iron
ions, which carry a positive charge, are added to the mix, the iron "distracts" the
negative charges and the reaction moves forward quickly.

Examples of Homogeneous Catalysts


Acid catalysis, organometallic catalysis, and enzymatic catalysis are examples
of homogeneous catalysis. Most often, homogeneous catalysis involves the
introduction of an aqueous phase catalyst into an aqueous solution of reactants. In
such cases, acids and bases are often very effective catalysts, as they can speed up
reactions by affecting bond polarization.
An advantage of homogeneous catalysis is that the catalyst mixes into the
reaction mixture, allowing a very high degree of interaction between catalyst and
reactant molecules. However, unlike with heterogeneous catalysis, the homogeneous
catalyst is often irrecoverable after the reaction has run to completion.
Homogeneous catalysts are used in variety of industrial applications, as they
allow for an increase in reaction rate without an increase in temperature.

Heterogeneous Catalysis
A heterogeneous catalyst is a catalyst that is present in a different phase
(usually a solid) than the reactants. Such catalysts generally function by furnishing an
active surface upon which a reaction can occur. Gas and liquid phase reactions
catalyzed by heterogeneous catalysts occur on the surface of the catalyst rather than
327
NOTE: Practice personal hygiene protocols at all times
within the gas or liquid phase. Heterogeneous catalysis is most often involves gaseous
reactants being adsorbed on the surface of a solid catalyst. Adsorption refers to the
collection of one substance on the surface of another substance; absorption refers to
the penetration of one substance into another. Water is absorbed by the sponge.

Mechanism
Heterogeneous catalysis has at least four steps:
1. Adsorption of the reactant onto the surface of the catalyst
2. Activation of the adsorbed reactant
3. Reaction of the adsorbed reactant
4. Diffusion of the product from the surface into the gas or liquid phase
(desorption).
Any one of these steps may be slow and thus may serve as the rate determining
step. In general, however, in the presence of the catalyst, the overall rate of the
reaction is faster than it would be if the reactants were in the gas or liquid phase.

Figure 1 illustrates the


steps that chemists
believe to occur in the
reaction of compounds
on a catalyst.
Source:
http://www.knockhardy.
org.uk/catalyst.htm

Advantages and Disadvantages of Heterogeneous Catalysis

Heterogeneous catalysis has several benefits. For one, heterogeneous


catalysts can be separated from a reaction mixture in a straightforward manner, such
as by filtration. In this way, expensive catalysts can be easily and effectively recovered,
which is an important consideration for industrial manufacturing processes.

328
NOTE: Practice personal hygiene protocols at all times
However, one limitation of heterogeneous catalysis has to do with the available
surface area of the catalyst. Once the surface of the catalyst is completely saturated
with reactant molecules, the reaction cannot proceed until products leave the surface,
and some space opens again for a new reactant molecule to adsorb or attach. It is for
this reason that the adsorption step in a heterogeneously catalyzed reaction is
oftentimes the rate-limiting step. Despite this, the overall benefits of heterogeneous
catalysis often outweigh its disadvantages, in that the catalyzed reaction is still much
faster than the uncatalyzed reaction.

Adsorption of ethene on a solid catalyst surface

Photo taken from: https://courses.lumenlearning.com/boundless-chemistry/chapter/catalysis/

One example of a heterogeneous catalyst is the catalytic converter in gasoline


or diesel-fueled cars. Catalytic converters contain transition metal catalysts embedded
on a solid phase support. The solid-phase catalyst comes into contact with gases from
the car's exhaust stream, increasing the rate of reactions to form less toxic products
from pollutants in the exhaust stream such as carbon monoxide and unburnt fuel.
329
NOTE: Practice personal hygiene protocols at all times
Other significant industrial processes that involve the use of heterogeneous
catalysts include the preparation of sulfuric acid, the preparation of ammonia, the
oxidation of ammonia to nitric acid, and the synthesis of methanol, CH 3OH.

Enzymes

Enzymes are proteins that function as catalysts in living systems. These


enzymes have components called substrate binding sites, or active sites, where the
molecules involved in the reaction under catalysis become attached. The component
parts of all proteins are amino acids, and each of these individual acids has an
uneven charge distribution from one end to the other. This property is the main
reason enzymes possess catalytic capabilities.
In general, when more substrate and more of an enzyme are present, the
reaction will proceed more quickly. But if more and more substrate is added without
adding more enzyme as well, all of the enzymatic binding sites become saturated,
and the reaction has reached its maximum rate for that enzyme concentration. Each
reaction catalyzed by an enzyme can be represented in terms of the intermediate
products formed owing to the presence of the enzyme.
Enzymes, although classified as category of catalyst distinct from those listed
above, can be either homogeneous or heterogeneous.

ENZYME STRUCTURE AND FUNCTION

The study of enzymes is an important interconnection between biology and


chemistry. Enzymes are usually proteins (polypeptides) that help to control the rate of
chemical reactions between biologically important compounds, particularly those that
are involved in cellular metabolism. Different classes of enzymes perform a variety of
functions, as shown in Table 1.

330
NOTE: Practice personal hygiene protocols at all times
Table 1. Classes of Enzymes and Their Functions
Class Function
Oxidoreductases Redox reactions
Transferases Transfer of functional groups
Hydrolases Hydrolysis reactions
Lyases Group elimination to form double bonds
Isomerases Isomerization
Ligases Bond formation with ATP hydrolysis

Table taken from: https://www.researchgate.net/profile/A_Klos-


Witkowska/publication/276309566/figure/tbl1/AS:614271008382984@1523465029046/Si
x-classes-of-enzymes-and-their-functions-used-in-the-detection-of-analytes.png

Enzyme molecules possess an active site, a part of the molecule with a shape
that allows it to bond to a specific substrate (a reactant molecule), forming an enzyme-
substrate complex as a reaction intermediate. There are two models that attempt to
explain how this active site works. The most simplistic model is referred to as the lock-
and-key hypothesis, which suggests that the molecular shapes of the active site and
substrate are complementary, fitting together like a key in a lock. The induced fit
hypothesis, on the other hand, suggests that the enzyme molecule is flexible and
changes shape to accommodate a bond with the substrate. This is not to suggest that
an enzyme’s active site is completely malleable, however. Both the lock-and-key
model and the induced fit model account for the fact that enzymes can only bind with

Figure 2. (a) According to the lock-and-key model, the shape of an enzyme’s active site
is a perfect fit for the substrate. (b) According to the induced fit model, the active site is
somewhat flexible, and can change shape to bond with the substrate.
Photo taken from: https://s3-us-west-2.amazonaws.com/courses-images/wp-
content/uploads/sites/1941/2017/05/30163032/648px-Induced_fit_diagram.svg_.png

331
NOTE: Practice personal hygiene protocols at all times
specific substrates, since in general a particular enzyme only catalyzes a particular
reaction (Figure 2).

Learning Competency:
Cite and differentiate the types of catalysts. (STEM-GC11CK-IIIi-j138)

Activity 1: Heterogeneous vs. Homogeneous: Which is more Interesting?


Directions: Using the Graphic Organizer below, compare and contrast the two types
of catalyst by filling-up the boxes. Refer to the Rubrics below on how your Graphic
Organizer will be graded.
First Type of Catalyst: Second Type of Catalyst:

Different Features: Similar Features: Different Features:

Conclusion:

332
NOTE: Practice personal hygiene protocols at all times
Rubrics

CRITERIA Full Credit Partial Credit Limited Credit Minimal Credit RATIN
(10 points) (7 points) (5 points) (2 points) G
- Compares & - Compares and - Compares and
- Compares or
contrasts items contrasts clearly, contrasts clearly,
contrasts, but
clearly but supporting but supporting
Purpose & does not do both
- Only includes information is information is
Supporting - No supporting
relevant and general incomplete.
Details information, or
accurate - Only includes - May include
incomplete
information relevant irrelevant
information
information information

- Whole-to-whole - Breaks
- Breaks
similarities information into
information into
- Whole-to-whole one of the
structure, but - Many details are
differences structures
Organizatio some information not in logical
- Similarities-to- - Does not follow
n& is in wrong order
differences consistent order
Structure section - Little sense that the
- Consistent when comparing
- Some details writing is organized
order when
are not in logical
discussing the
or expected order
comparison

- No errors in - 1-2 errors in - 3-4 errors in - Excessive (5 or


grammar grammar or grammar and more) errors in
or spelling spelling that spelling that grammar and
Grammar &
distract the distract the spelling that distract
Spelling
reader reader the
reader from the
content

- Moves smoothly - Moves from one


from idea to the next,
one idea to the but with little
next variety
- Uses - Uses - Some
comparison and comparison transitions
- Transitions are
contrast transition and contrast work well, but
Transitions unclear or
words to show transition words to connections
nonexistent
relationships show between other
- There are relationships ideas are unclear
variety of between ideas
sentence
structures &
transitions

TOTAL:

Rubrics base from: https://www.pinterest.ph/pin/297519119103384580/

333
NOTE: Practice personal hygiene protocols at all times
Activity 2: Self-Check with the Concept
Directions: Answer the following questions completely.
1. Identify each of the following as examples of the action of homogeneous or
heterogeneous catalysts:
a. Rhodium and platinum metals are used in an automobile catalytic converter to
convert exhaust gases into safer gases. _____________________________
b. Gaseous chlorofluorocarbons (CFCs) have been shown to catalyze the
breakdown of ozone in the upper atmosphere. ________________________
c. Aqueous sulfuric acid catalyzes the decomposition of aqueous formic acid to
carbon monoxide and water. ______________________________________
d. Powdered TiCl4 is used in the formation of polyethylene polymer from gaseous
ethylene. _____________________________________________________

2. What are the key stages in heterogeneous catalysis?


____________________________________________________________
____________________________________________________________
____________________________________________________________
____________________________________________________________
____________________________

3. Many gas reactions are catalyzed by solids, and the process starts with the gas
molecules being adsorbed on to the metal surface. Explain the difference between the
words adsorbed and absorbed.
___________________________________________________________________
___________________________________________________________________
___________________________________________________________________
___________________________________________________________________
4. Silver and tungsten are not very good catalysts, but platinum and nickel can be
excellent catalysts. Explain why this is in terms of the strength of the adsorption of
gases to their surfaces.
__________________________________________________________________________________
__________________________________________________________________________________

334
NOTE: Practice personal hygiene protocols at all times
__________________________________________________________________________________
__________________________________________________________________________________
5. A heterogeneous catalyst works by interacting with a reactant in a process
called adsorption. What occurs during this process? Explain how this can lower the
activation energy.
___________________________________________________________________
___________________________________________________________________
___________________________________________________________________
___________________________________________________________________

335
NOTE: Practice personal hygiene protocols at all times
Activity 3: Operation: Word Search!
Directions: Locate and encircle the term being described in each item below the grid.
The words can be found horizontally, vertically, horizontally or in reverse. Write also
the term being described on each item at the space provided.
Word Search Puzzle

1. These speeds up the rate of a chemical reaction in a living organism. ______________


2. This refers to the collection of one substance on the surface of another
substance. __________________
3. A heterogeneous catalyst is a catalyst that is present in a different phase which is
usually in what state of matter than the reactants? ___________________
4. It is termed as any reaction that makes use of a catalyst. _________________

336
NOTE: Practice personal hygiene protocols at all times
5. Enzyme molecules possess this part of the molecule with a shape that allows it to
bond to a specific substrate (a reactant molecule), forming an enzyme-substrate
complex as a reaction intermediate. ____________________
6. It is the term used to describe when the products were released from the surface
of the catalyst. ___________________
7. It is a type of catalysis wherein the catalyst and the reacting substances are present
together in a single state of matter. ____________________
8. It is a substance that speeds up the rate of a reaction without itself being
consumed in the reaction. ___________________
9. Enzymes are usually ___________ that help to control the rate of chemical reactions
between biologically important compounds, particularly those that are involved in cellular
metabolism.
10. When the catalyst and the reacting substances are present together in a single state of
matter, usually as a ______ or a ________, it is customary to classify the reactions as
cases of homogeneous catalysis.

Activity 4: More, more, more Catalyst!


In this activity, you will be finding examples of heterogeneous and homogeneous
catalysts or catalytic reactions in the real world.
Directions: Identify three examples of each type of catalyst by completing the table
below. Give a brief description for each examples. A sample is given below.

Types of Sample Example 1 Example 2 Example 3


Catalyst

a. Homogeneous Oxidation of
Sulfur dioxide
into Sulfur
trioxide with
dioxygen in
the presence
of oxides of
nitrogen as
the catalyst in
the lead
chamber
process. The
reactants,
337
NOTE: Practice personal hygiene protocols at all times
sulfur dioxide
and oxygen,
and the
catalyst, nitric
oxide, are all
in the same
phase.
b. Heterogeneous Oxidation of
Sulfur dioxide
into Sulfur
trioxide in the
presence of
Pt. The
reactant is in
gaseous
state while
the catalyst is
in the solid
state.

Rubrics

CRITERIA Excellent Good Fair Needs RATING


(5 points) (3 points) (2 points) Improvement
(1 point)
Demonstrates Demonstrates Demonstrates
Do not
full knowledge enough a little
demonstrates
by providing knowledge by knowledge by
any knowledge
Content three providing ttwo providing one
by providing no
examples with examples with example with
examples with
explanation explanation explanation
explanation and
and and elaboration and
elaboration
elaboration elaboration
Obviously Has logical and Organization
controlled appropriate attempted but Inconsistent
Organization
and/or organization of unclear or organization of
& Structure
intelligent ideas inappropriate ideas
organization of ideas
ideas
- No errors in - 1-2 errors in - 3-4 errors - Excessive
grammar grammar or that distract errors that
Grammar &
or spelling spelling that the reader distract the
Spelling
distract the reader from the
(5 points)
reader content

TOTAL:

Rubrics based from: http://www.studentwritingintensive.com/writing-rubric-for-esl-students.html

338
NOTE: Practice personal hygiene protocols at all times
Activity 5: Catalytic Ozone Destruction
Directions: Read and analyze the article below. Make a Reaction Paper on the
importance of having knowledge about catalyst in the environment. The Reaction
Paper should tackle the role of catalyst in the formation of ozone layer and how human
activities paved its destruction. A Rubric is prepared that will be used as a basis on
how your output will be graded. If there are questions or clarifications, it is better to
ask me directly.

Mario J. Molina: The Man behind the Discovery of Ozone Layer

The 1995 Nobel Prize in Chemistry was shared by Paul J. Crutzen, Mario
J. Molina (Figure 3), and F. Sherwood Rowland “for their work in atmospheric

Figure 3. Mexican chemist Mario Molina (a) shared the Nobel Prize in Chemistry in 1995
for his research on the Antarctic ozone hole (b).

chemistry, particularly concerning the formation and decomposition of ozone.” Molina,


a Mexican citizen, carried out most of his work at the Massachusetts Institute of
Technology (MIT).
In 1974, Molina and Rowland published a paper in the journal Nature (one of
the major peer-reviewed publications in the field of science) detailing the threat of
chlorofluorocarbon gases to the stability of the ozone layer in earth’s upper
atmosphere. The ozone layer protects earth from solar radiation by absorbing
ultraviolet light. As chemical reactions deplete the amount of ozone in the upper
atmosphere, a measurable “hole” forms above Antarctica, and an increase in the
amount of solar ultraviolet radiation— strongly linked to the prevalence of skin
cancers—reaches earth’s surface. The work of Molina and Rowland was instrumental

339
NOTE: Practice personal hygiene protocols at all times
in the adoption of the Montreal Protocol, an international treaty signed in 1987 that
successfully began phasing out production of chemicals linked to ozone destruction.
Molina and Rowland demonstrated that chlorine atoms from human-made chemicals
can catalyze ozone destruction in a process similar to that by which NO accelerates
the depletion of ozone. Chlorine atoms are generated when chlorocarbons or
chlorofluorocarbons—once widely used as refrigerants and propellants—are
photochemically decomposed by ultraviolet light or react with hydroxyl radicals. A
sample mechanism is shown here using methyl chloride:
CH3Cl + OH → Cl + other products
Chlorine radicals break down ozone and are regenerated by the following
catalytic cycle:
Cl + O3 → ClO + O2
ClO + O → Cl+ O2
Overall Reaction: O3 + O → 2O2
A single monatomic chlorine can break down thousands of ozone molecules.
Luckily, most of atmospheric chlorine exists as the catalytically inactive forms Cl2 and
ClONO2.
Since receiving his portion of the Nobel Prize, Molina has continued his work in
atmospheric chemistry at MIT.
Source: “The Nobel Prize in Chemistry 1995,” Nobel Prize.org, accessed February 18,
2015, http://www.nobelprize.org/nobel prizes/chemistry/laureates/1995/.

Rubrics
CRITERIA LEVEL 1 LEVEL 2 LEVEL 3 RATING
(1 point) (3 points) (5 points)
The paper is
The paper is well
organized,
organized, uses
makes good use
The paper is poorly transition
of transition
organized and the statements
Organization statements and
ideas were difficult appropriately
in most
to follow. and follows a
instances
logical
follows a logical
progression.
progression.
Almost all parts The whole paper
Most part of the
of the paper was complete
Completion paper was not
were finished on and finished on
finished on time.
time. time.

340
NOTE: Practice personal hygiene protocols at all times
More than 5 errors 3-5 errors in None to 2 errors
Grammar &
in punctuation and punctuation and in punctuation
Spelling
spelling. spelling. and spelling.
Content
Content indicates
Shows some
indicates synthesis of
thinking and
original ideas, in depth
reasoning but most
thinking and analysis and
Level of Content ideas are
develops ideas evidence,
underdeveloped
with sufficient original
and
and firm thought and
unoriginal.
evidence. support for the
topic.
Information is
Information is
gathered from Information is
gathered from
limited gathered from
Information multiple electronic
electronic and electronic or
Gathering and non-electronic
non-electronic non-electronic
sources and cited
sources sources only
properly
TOTAL:

Rubrics based from: https://www.thegraidenetwork.com/blog-all/2018/8/1/rubrics-friend-or-foe

Reflection:
1.I learned that _______________________________________________________
________________________________________________________________
_______________________________________________________.

2.I enjoyed most on ___________________________________________________


________________________________________________________________
_________________________________________________________.

3.I want to learn more on _______________________________________________


________________________________________________________________
____________________________________________________________.

341
NOTE: Practice personal hygiene protocols at all times
References

Catalysis. Retrieved from https://www.britannica.com/science/catalysis/Classification-of-


catalysts

Catalyst in Chemistry. Retrieved from


https://www.sciencenewsforstudents.org/article/explainer-catalyst-chemistry

Types of Catalysts. Retrieved from https://www.khanacademy.org/science/chemistry/chem-


kinetics/arrhenius-equation/a/types-of-catalysts

Catalysis. Retrieved from


https://courses.lumenlearning.com/chemistryformajors/chapter/catalysis/

Catalysis. Retrieved from


https://press.rebus.community/introductorychemistry/chapter/catalysis/

Types of Catalyst. Retrieved from https://sciencing.com/different-types-enzymes-


4968363.html

342
NOTE: Practice personal hygiene protocols at all times
ANSWER KEY

Activity 1: Compare & Contrast Two Types of Catalyst


• (Possible answer)

First Type of Catalyst: Second Type of Catalyst:


Homogeneous Heterogeneous

Different Features: Similar Features: Different Features:



• The catalyst and • Both speeds up • A catalyst that is
the reacting chemical reaction. present in a
• substances are different phase
present together in (usually a solid)
a single state of
• matter
than the reactants
• Easy catalyst
• Difficult catalyst separation
• separation

Conclusion:
In chemistry, a catalyst is a substance that speeds up the rate of a reaction
without itself being consumed in the reaction. Any reaction that makes use of a catalyst
is termed catalysis. An overview of each of the types of catalysts is a helpful starting
point in learning analytical chemistry and understanding what happens at the molecular
level when you mix substances together and a reaction occurs. Catalysts and their
associated catalytic reactions come in two main types: homogeneous catalysts and
heterogeneous catalysts.

ACTIVITY 2: Self-Check with the Concept


1.(a) Heterogeneous Catalyst (b) Homogeneous catalyst (c) Homogeneous catalyst
(d) Heterogeneous catalyst
2.In heterogeneous catalysis, the reacting and catalyst are in different states of the
matter. The most important steps in this process are;
– Adsorption of reactant molecules activation centre.
– Formation of activation complex at the centre.
– This complex decomposes to give products.
343
NOTE: Practice personal hygiene protocols at all times
– Desorption of products from the surface of the catalyst.
3.Adsorption happens when something (a molecule from a gas or liquid, for example)
becomes stuck to the surface of a solid. Absorption happens when one substance is
taken up inside another one.
4.Solid catalysts often work by first adsorbing something on their surface. A reaction
then takes place, and the products are desorbed from the surface (break away into
the gas or liquid again). That frees up the surface for the next lot of reactants to
become adsorbed.
5.In adsorption, a reactant binds tightly to a surface. Because intermolecular
interactions between the surface and the reactant weaken or break bonds in the
reactant, its reactivity is increased, and the activation energy for a reaction is often
decreased.

Activity 3: Testing your Understanding!

Activity 4: Real World Examples


• Students’ output may vary. See attached Rubrics below the activity for scoring
purposes.

344
NOTE: Practice personal hygiene protocols at all times
Activity 5: Catalytic Ozone Destruction

• Students’ output may vary. See attached Rubrics below the activity for scoring
purposes.

Prepared by:

GENEVIE VINAGRERA
Licerio Antiporda Sr National High School- Main

345
NOTE: Practice personal hygiene protocols at all times
346
NOTE: Practice personal hygiene protocols at all times

You might also like